Pneumo and Critical Care Medicine

¡Supera tus tareas y exámenes ahora con Quizwiz!

Pneumo 44 A 67-year-old man is evaluated in the surgical intensive care unit. He underwent laparotomy and diverting colostomy for a ruptured diverticulum 72 hours ago, and now has a temperature of 40.0 °C (104.0 °F) and a heart rate of 135/min. In the past 3 hours his mean arterial blood pressure has dropped to 58 mm Hg despite three 1-L boluses of normal saline; urine output was only 15 mL in the past hour. The patient's oxygen saturation is 85% on 100% oxygen by nonrebreather mask. Platelet count is 42,000/µL (42 × 109/L). A portable chest radiograph shows bilateral alveolar infiltrates. A central venous catheter is placed; invasive mechanical ventilation and broad-spectrum antibiotic therapy are begun. Which of the following is most appropriate next step in the management of this patient? A Activated protein C B Colloid fluid infusion C Insulin drip to maintain plasma glucose less than 110 mg/dL (6.1 mmol/L) D Low-dose dopamine

A Activated protein C has been shown to improve survival in patients with severe sepsis with an APACHE score of 25 or greater. Activated protein C (drotrecogin alfa activated) is a time-sensitive intervention that can improve survival in patients with severe sepsis at high risk of death. Improved survival has been demonstrated in patients with severe sepsis who have an APACHE score of 25 or greater. Patients with either a single failing organ system or an APACHE score less than 25 do not appear to benefit and are at risk of bleeding complications. Although activated protein C is an anticoagulant, when administered to patients with a platelet count between 30,000/µL (30 × 109/L) and 50,000/µL (50 × 109/L), there was a relative risk reduction in mortality of more than 30% in the Phase III PROWESS trial. Platelet counts below 30,000/µL are considered a relative contraindication. The patient is more than 12 hours out of surgery, with no ongoing active bleeding, a platelet count of 42,000/µL (42 × 109/L), and a high risk of death; therefore, activated protein C is an excellent consideration. Hyperglycemia is associated with poor clinical outcomes in critically ill patients. However, the benefit of tight glycemic control (≤110 mg/dL [6.1 mmol/L]) is controversial in critically ill postsurgical patients, and no benefit has been shown in critically ill medical patients. Vasopressors are part of early goal-directed therapy if the mean arterial pressure is less than 65 mm Hg after initial adequate fluid resuscitation. The most commonly used vasopressor for septic shock is norepinephrine, a potent peripheral vasoconstrictor that reverses the endotoxin-induced vasodilation that is the hallmark of septic shock. Dopamine is also acceptable but is associated with more tachycardia and arrhythmia. Low-dose dopamine, however, is not indicated. A randomized controlled trial showed that there is no benefit from "renal doses" of dopamine on renal or other clinical outcomes in early renal dysfunction. The goals of fluid resuscitation are a central venous pressure of 8 to 12 mm Hg, mean arterial pressure greater than 65 mm Hg, urine output greater than 0.5 mL/kg/h, and central venous oxygen saturation greater than 70%. Randomized controlled trials have shown no benefit to the use of colloid compared with crystalloid fluids.

Pneumo 98 A 22-year-old woman is evaluated in the intensive care unit 5 days after sustaining multiple long-bone fractures. Her clinical course was complicated by the acute respiratory distress syndrome, and she was intubated and has been receiving mechanical ventilation for the past 3 days. Her medications are hydromorphone and lorazepam. Over the past 24 hours, the FiO2 and positive end-expiratory pressure have been decreased to 0.4 and 5 cm H2O, respectively, while maintaining an oxygen saturation of 95%. Chest radiograph shows improvement in her bilateral alveolar infiltrates. Sedation is interrupted and she undergoes a spontaneous breathing trial. After 60 minutes, she is diaphoretic, the pulse rate is 120/min, the respiration rate is 30/min, tidal volume is 250 mL, and oxygen saturation is 88%. Full ventilatory support is restarted. Which of the following is most likely to expedite this patient's weaning from the ventilator? A Daily reduction of sedation followed by a spontaneous breathing trial B Extubation and institution of noninvasive positive-pressure ventilation C Increasing FiO2 to 0.6 D Repeating spontaneous breathing trial in 48 hours E Tracheostomy

A Daily spontaneous breathing trials and daily sedation reduction both reduce the duration of mechanical ventilation, and combining the two is superior to daily breathing trials alone. Increasingly, protocol-driven care is being integrated into the routine management of patients in the intensive care unit. This approach has proven to be especially effective in weaning patients from mechanical ventilation. A randomized controlled trial found a protocol that paired sedation reduction and spontaneous breathing trials resulted in fewer days of mechanical ventilation, earlier discharge from the intensive care unit, and earlier hospital discharge compared with daily spontaneous breathing trials alone. One randomized study found early extubation of 50 patients with exacerbation of chronic obstructive pulmonary disease directly to noninvasive positive-pressure ventilation shortened stays in the intensive care unit and improved survival. However, these benefits were not duplicated in subsequent studies, and this approach is suitable only for highly selected patients with acute-on-chronic respiratory failure rather than this patient with acute respiratory failure. Increasing supplemental oxygen beyond 50% will not expedite weaning. Patients requiring more than 50% supplemental oxygen are generally not considered candidates for weaning. Daily, rather than every other day, trials of spontaneous breathing reduce the duration of mechanical ventilation. This patient requires modest ventilator support, and her condition is improving. The anticipated duration of invasive mechanical ventilation is brief, and tracheostomy is not routinely indicated in such cases. Limited evidence suggests early tracheostomy can improve outcomes in patients receiving invasive mechanical ventilation, but there is no consensus on the selection criteria and timing of early tracheostomy.

Pneumo 58 A 77-year-old woman is evaluated for a 1-year history of progressive dyspnea and dry cough. She has had previous evaluations for these symptoms and has had abnormal chest radiographs and high-resolution CT scans. She also has a history of heart failure, systolic hypertension, Sjögren syndrome, and monoclonal gammopathy of unknown significance; her medications are metoprolol, lisinopril, furosemide, hypromellose artificial tears, potassium chloride, and a multivitamin. Three weeks ago, the patient had an open lung biopsy that showed polyclonal plasma-cell infiltration of the pulmonary interstitium within the alveolar septa with fibrotic changes, including rare honeycombing consistent with lymphocytic interstitial pneumonia. High-resolution CT scan before the biopsy showed bilateral mid- and lower-lung abnormalities, including scattered ground-glass opacification, rare subpleural reticular changes, and scattered tiny pulmonary nodules with rare cystic air spaces. On physical examination, she is afebrile, the pulse rate is 78/min, the respiration rate is 16/min, and the blood pressure is 130/78 mm Hg. On cardiac examination, there is no evidence of jugular venous distention and an S4 is present, but there are no murmurs. There are bilateral lower-lung crackles and rare squeaks. There is 1+ edema at the ankles but no digital clubbing. Lung function studies show a mild restrictive pattern. Which of the following is the most likely diagnosis? A Connective tissue disease-related lymphocytic interstitial pneumonia B Idiopathic lymphocytic interstitial pneumonia C Idiopathic pulmonary fibrosis D Primary pulmonary lymphoma

A Lymphocytic interstitial pneumonia is detected at open lung biopsy in approximately one third of patients with Sjögren syndrome. The diagnostic open lung biopsy specimen in this patient with Sjögren syndrome shows the histopathologic features of lymphocytic interstitial pneumonia (LIP), a rare form of interstitial lung disease (ILD) that may occur in one third of patients with ILD and Sjögren syndrome. Affected patients may develop ILD with known connective tissue disease, or ILD may be the presenting finding in a patient with undiagnosed connective tissue disease. Sjögren syndrome-associated ILD is not limited to LIP but may also present as usual interstitial pneumonia, nonspecific interstitial pneumonia, organizing pneumonia, and interstitial amyloidosis at open lung biopsy. In addition, finding LIP at open lung biopsy is not specific for Sjögren syndrome. Treatment is generally directed at the underlying connective tissue disease, but the severity of ILD is important in making treatment decisions. Because this patient has Sjögren syndrome, the diagnosis of idiopathic LIP or any other idiopathic ILD, including idiopathic pulmonary fibrosis, is incorrect. Lymphoma, which develops in up to 5% of patients with Sjögren syndrome, is a rare cause of pulmonary interstitial infiltration in patients with Sjögren syndrome. The diagnosis of primary pulmonary lymphoma requires the pathologic demonstration of a monoclonal population of lymphocytes.

Pneumo 42 A 38-year-old man is evaluated for a 1-year history of shortness of breath with exertion and wheezing. He has smoked intermittently since the age of 18 years. On physical examination, vital signs are normal; BMI is 25. Breath sounds are diminished bilaterally, and there is wheezing posteriorly. Spirometry shows an FEV1 of 55% of predicted and an FEV1/FVC ratio of 61% of predicted. Chest radiograph shows bilateral basilar lucency. Plasma α1-antitrypsin concentration is 25 mg/dL (4.6 µmol/L). The patient is counseled to stop smoking and referred to a smoking cessation program. Standard supportive inhaled bronchodilator and long-acting β2-agonist therapy is started. Which of the following therapies is most reasonable to consider for this patient at this time? A Danazol B Intravenous infusion of human α1-antitrypsin C Lung transplantation D Lung volume reduction surgery

B Intravenous augmentation therapy with an α1-antitrypsin (AAT) is the most direct and efficient means of elevating AAT levels in plasma and lung interstitium. α1-Antitrypsin (AAT) deficiency is associated with early-onset pulmonary emphysema, liver disease, and, rarely, skin disease. In suitable candidates, intravenous infusion of pooled human AAT is the most direct and efficient method of elevating AAT levels in plasma and lung interstitium. Augmentation therapy is most effective in patients with an FEV1 35% to 60% of predicted and an FEV1/FVC ratio 30% to 65%. The selection criteria include: * High-risk phenotype (Pi*Z [protease inhibitor Z]) * Plasma AAT levels below 50 to 80 mg/dL (0.5-0.8 g/L) * Nonsmoker or ex-smoker * Likely adherence to the protocol * Airflow obstruction with spirometry * Age at least 18 years Estimated mean annual cost for patients receiving therapy is $40,000. Efficacy data are derived from observational (not randomized, controlled) trials that suggest that therapy retards the rate of FEV1 decline. Pending further studies, expert groups recommend augmentation therapy for properly selected individuals. Aerosolized AAT augmentation is being studied and has many potential advantages compared with intravenous therapy. Augmentation therapy is not, however, recommended for patients without emphysema. Lung transplantation is an option for patients with severe end-stage disease but is not indicated in this mildly symptomatic patient. Certain hormonal drugs, such as danazol, tamoxifen, and progesterone, have been found to augment endogenous production of AAT. However, these drugs have had very limited success in increasing AAT levels to the point of having a clinical effect and are inferior to exogenous augmentation with AAT. The National Emphysema Therapeutic Trial of lung volume reduction surgery reported mixed results for patients with emphysema. Overall, lung volume reduction surgery appears to alleviate dyspnea and increase exercise capacity, lung function, and quality of life. Survival appears to be improved only in patients with upper lobe-predominant emphysema and low exercise capacity.

Pneumo 100 A 59-year-old man is evaluated in the emergency department in early June for severe bronchospasm. The patient is a farmer, has no significant medical history, and takes no medications. On physical examination, he is anxious, agitated, and disoriented to time and place. He is diaphoretic with a large amount of oral and respiratory secretions. The blood pressure is 80/60 mm Hg, the pulse rate is 50/min, and the respiration rate is 26/min. He has diffuse expiratory wheezing. Laboratory studies show normal electrolytes and complete blood count and a respiratory alkalosis. Which of the following is the most appropriate management? A Amyl nitrite B Pralidoxime C Sodium nitrite D Sodium thiosulfate

B Pralidoxime (2-PAM) reactivates acetylcholinesterase and can reverse the muscle weakness, paralysis, and respiratory depression of organophosphate toxicity. This patient likely has organophosphate poisoning. Organophosphates are a diverse group of chemicals used in both domestic and industrial settings that include insecticides, nerve gases (soman, sarin, tabun, VX), ophthalmic agents (echothiophate, isoflurophate), and antihelmintics (trichlorfon). Exposure to organophosphate insecticides may occur by dermal, gastrointestinal, inhalational, and intravenous routes. Organophosphate insecticides inhibit both cholinesterase and pseudocholinesterase activities. The inhibition of cholinesterase activity leads to accumulation of acetylcholine at synapses, causing overstimulation and disruption of neurotransmission in both the central and peripheral nervous systems. Signs and symptoms of organophosphate poisoning include muscarinic effects, nicotinic effects, and central nervous system effects peripherally at muscarinic receptors and nicotinic receptors. Muscarinic manifestations include excessive salivation, diarrhea, vomiting, hypersalivation, respiratory distress with bronchorrhea and bronchospasm, abdominal pain, depressed level of consciousness, and muscle fasciculations. The muscarinic signs of organophosphate poisoning can be recalled with the help of the mnemonic DUMBELS—Defecation, Urination, Miosis, Bronchorrhea/Bronchospasm/Bradycardia, Emesis, Lacrimation, Salivation. Nicotinic manifestations include increased muscle weakness, skeletal muscle fasciculations, and respiratory failure secondary to diaphragmatic paralysis. Central nervous system effects include altered mental status and seizures. Organophosphate insecticide poisoning is a serious condition that requires rapid diagnosis and treatment. Pralidoxime (2-PAM) reactivates acetylcholinesterase and can reverse the muscle weakness, paralysis, and respiratory depression of organophosphate toxicity. Although atropine treats the bronchorrhea, it does not reverse the muscle weakness or respiratory depression. Amyl nitrite, sodium nitrite, and sodium thiosulfate are used to treat cyanide toxicity and are ineffective in the treatment of organophosphate poisoning. The most common early clinical manifestations of cyanide poisoning include headache and confusion, coma, tachypnea, tachycardia, and skin flushing. Excessive salivation, wheezing, and bronchorrhea do not occur. An anion-gap metabolic acidosis typically occurs as well as a low arterial-venous oxygen gradient as a result of hyperoxygenation of the venous blood.

Pneumo 29 A 54-year-old woman is evaluated after a 1.5-cm nodule was detected in the right lower lobe of the lung on a chest radiograph done to evaluate new-onset dyspnea on exertion. The nodule was not present on a chest radiograph done 8 years ago. The patient lives in Missouri and has not traveled recently. She has a 30-pack-year history of cigarette smoking but quit smoking 3 years ago. Her medical history includes hypertension and hyperlipidemia, and her medications are hydrochlorothiazide and atorvastatin. There is no family history of lung cancer. On physical examination, the temperature is 36.8 °C (98.0 °F), the blood pressure is 124/72 mm Hg, the pulse rate is 64/min, and the respiration rate is 14/min. The lungs are clear, the skin is normal, and there is no lymphadenopathy. Pulmonary function testing shows mild airway obstruction and no acute response to a bronchodilator. Contrast-enhanced CT scan of the chest shows the nodule to be uncalcified on thin-section images, and there was no significant enhancement (10 Hounsfield units) on dynamic contrast study. Histoplasma serology is negative. Which of the following is the most appropriate management for this patient? A Bronchoscopy and biopsy of the lesion B CT-guided transthoracic biopsy of the lesion C Repeat CT scan in 3 months D Video-assisted thoracoscopic surgery to remove the nodule E No further evaluation

C A nodule that is nonenhancing on dynamic CT contrast study is likely benign, and observation is appropriate. The attenuation coefficient of a pulmonary nodule is a measure of its density and is expressed in Hounsfield units. Enhancement after contrast reflects nodule vascularity and is an indicator of malignancy or active inflammation. A nodule that shows less than 15 Hounsfield units of enhancement on dynamic contrast study is highly likely to be benign; a multicenter study showed a 97% negative predictive value at a cutoff of 15 Hounsfield units. Reevaluation of the nodule at 3 months would be appropriate given the low likelihood that this is a malignant nodule. Immediate evaluation with biopsy or surgical removal can be avoided because of the low likelihood that this is a malignant nodule. Lack of enhancement is not the same as a tissue diagnosis, and follow-up is appropriate whether or not the patient has risk factors for lung cancer.

Pneumo 46 A 36-year-old woman is evaluated in the office after having had a small (less than 2 cm) spontaneous pneumothorax treated conservatively in the emergency department. The patient had a 6-month history of worsening dyspnea accompanied by a dry cough before the pneumothorax. She has never smoked and has a history of bilateral breast augmentation. On physical examination, there is no digital clubbing or crackles. High-resolution chest CT scan is shown . Which of the following is the most likely diagnosis? A Emphysema B Idiopathic pulmonary fibrosis C Lymphangioleiomyomatosis D Pulmonary Langerhans cell histiocytosis

C A pulmonary presentation including pneumothorax and abnormal chest imaging is the primary event leading to the diagnosis of lymphangioleiomyomatosis in most patients. This patient presents with subacute respiratory complaints and high-resolution CT findings of diffuse cysts in the clinical context of a young woman with a spontaneous pneumothorax who is otherwise healthy and has never smoked. The diagnosis is most likely lymphangioleiomyomatosis (LAM), a rare disease that accounts for less than 1% of all cases of interstitial lung diseases and occurs nearly exclusively in women in their childbearing years. The histopathologic feature of pulmonary LAM is proliferation of atypical smooth muscle-like cells associated with cystic changes. LAM may occur in patients with multiorgan hamartomas and a diagnosis of the autosomal dominant neurocutaneous syndrome called tuberous sclerosis complex. LAM also may present as a sporadic form in which renal and lymphatic hamartomas occur without diagnostic criteria for the tuberous sclerosis complex. The sporadic form of LAM presents with pulmonary complications and occurs only in women. Spontaneous pneumothorax is the initial cause for presentation in 36% of cases. Physical examination is often normal. Crackles and digital clubbing are usually absent. Many young women with LAM are initially thought to have emphysema. However, patients with LAM are much younger than those with emphysema and present with respiratory complications 86% of the time, which include pneumothorax, as in this case, and chylothorax. Although the HRCT appearance of emphysema may mimic LAM, the cystic air spaces that occur in LAM are more uniform in appearance and distribution. Testing for α1-antitrypsin deficiency may be appropriate to exclude emphysema in young, nonsmoking patients. Idiopathic pulmonary fibrosis has a chronic or subacute onset, but typically occurs in older patients. HRCT findings typically show bibasilar reticular changes with honeycombing and an absence of ground-glass opacification. Pulmonary Langerhans cell histiocytosis can present with spontaneous pneumothorax and nonspecific respiratory complaints. However, the HRCT findings, cysts accompanied by nodules, are distinct from those in LAM. In addition, pulmonary Langerhans cell histiocytosis occurs nearly exclusively in patients with a significant smoking history.

Pneumo 91 A 61-year-old woman is evaluated for a 3-month history of progressive loss of sensation in her legs accompanied by intermittent pain and paresthesias; she has not had shortness of breath. She can no longer walk unaided and uses a wheel chair. She has lost 9 kg (20 lb) during this time. The patient has smoked a pack of cigarettes a day for 30 years. She had been previously healthy and takes no medications. On physical examination, the temperature is 36.5 °C (97.7 °F), the blood pressure is 126/54 mm Hg, the pulse rate is 82/min, the respiration rate is 22/min, and the BMI is 23. There is generalized reduction in touch sensation in the legs, worse below the knees and greater on the right. Motor strength is intact. Gait is broad-based and ataxic. Romberg sign is present. There is no lymphadenopathy. Complete blood count; serum creatinine, glucose, calcium, and liver enzymes; and serum protein electrophoresis are normal. ANNA-1 (anti Hu) antibody is detected on a paraneoplastic antibody panel. Electromyography reveals a sensory neuropathy. Chest radiograph shows a left hilar mass. CT scan shows a 5-cm mass with associated left paratracheal and subcarinal lymphadenopathy. What is the most likely cause of the patient's symptoms? A Adenocarcinoma B Carcinoid tumor C Large cell carcinoma D Small cell carcinoma

D The ANNA-1 antibody is the most common antibody present in a sensory neuropathy due to small cell lung carcinoma. Paraneoplastic neurologic disorders are associated with systemic malignancy but are not related to direct tumor involvement or to other toxic or metabolic, infectious, or vascular complications of cancer. Paraneoplastic sensory neuropathy is commonly associated with lung carcinoma (particularly small cell carcinoma) but may develop in patients with other malignancies. The neurologic syndrome precedes the diagnosis of cancer in most patients. Paraneoplastic sensory neuropathy manifests with progressive paresthesias, profound sensory ataxia, and multimodality sensory loss. Such patients usually have limited-stage small cell lung carcinoma at presentation. The ANNA-1 antibody is the most common antibody present in a sensory neuropathy associated with small cell carcinoma. Paraneoplastic neurologic syndromes are uncommonly associated with the other cell types of lung cancer or other tissue malignancies.

Pneumo 31 A 52-year-old man is evaluated for a 7-month history of progressive dyspnea, initially with vigorous exertion; now, even walking slowly causes immediate severe dyspnea and dizziness. The symptoms subside when he is at rest. He has had two syncopal episodes, both while he was walking at a brisk pace. He does not have cough, chest pain, or wheezing. He has no other significant medical history and takes no medications. On physical examination, the temperature is 37.0 °C (98.6 °F), the blood pressure is 105/60 mm Hg, the pulse rate is 102/min at rest and 120/min after the patient walks across the room, the respiration rate is 20/min, and the BMI is 32. Lung expansion is normal during deep breathing. Jugular venous distention is present. Lungs are clear to auscultation with no wheezes or crackles. There is fixed splitting of S2 with an increased pulmonic component. There is a grade 1-2/6 holosystolic murmur at the left sternal border near the fourth rib that increases with inspiration. The lower extremities are edematous. There is no cyanosis or clubbing. Complete blood count and resting arterial blood gases are normal. Electrocardiography shows a rightward QRS axis and large R waves in V1. Spirometry and plethysmography are normal. The chest radiograph shows no infiltrates or masses. Which of the following is the best next step in the evaluation of this patient? A Bronchoscopy and transbronchial lung biopsy B Methacholine challenge test C Right-heart catheterization and pulmonary angiography D Transthoracic echocardiography

D In patients with suspected pulmonary hypertension, transthoracic echocardiography can suggest the presence of pulmonary hypertension and evaluate cardiac causes of elevated pulmonary artery pressure. The patient's progressive dyspnea, hemodynamic symptoms during exercise, and physical findings suggest right ventricular dysfunction and pulmonary hypertension. Transthoracic echocardiography can confirm the presence of pulmonary hypertension and right ventricular dysfunction. Echocardiography is also useful to rule out left-sided heart disease and congenital heart disease as a cause of pulmonary hypertension. A ventilation/perfusion scan can also rule out potential causes. Typically, the ventilation/perfusion scan in pulmonary arterial hypertension is either normal or shows a scattered, "moth-eaten" perfusion pattern in the peripheral lung zones. The patient has no evidence of bronchospasm. Exercise-induced asthma is unlikely because the symptoms begin immediately during mild exertion and subside rapidly upon rest. Furthermore, exercise-induced bronchospasm cannot explain the patient's clinical findings of pulmonary hypertension. Therefore, a methacholine challenge test is not indicated. Right-heart catheterization and pulmonary angiography might be necessary to confirm the diagnosis of pulmonary arterial hypertension but are not indicated before less invasive screening tests for pulmonary hypertension are done. Bronchoscopy and transbronchial lung biopsy may be indicated in patients with diffuse parenchymal lung disease, but this patient's chest radiograph is normal, making parenchymal lung disease unlikely.

Pneumo 22 A 60-year-old man is evaluated during routine follow-up in November. The patient has severe chronic obstructive pulmonary disease, with dyspnea on minimal exertion and a chronic cough. He has a 40-pack-year history of cigarette smoking, but he quit smoking 3 years ago. His medications are albuterol as needed, inhaled corticosteroids, and tiotropium. On physical examination, the patient is afebrile, blood pressure is 140/88 mm Hg, pulse rate is 90/min, and respiration rate is 20/min. Oxygen saturation with the patient at rest and breathing ambient air is 86%. Jugular venous distention and a loud P2 are present. The chest is hyperinflated and breath sounds are diminished. There is 1+ pedal edema. Hemoglobin concentration is 16.5 g/dL (165 g/L). Arterial blood gases show pH 7.35, PCO2 55 mm Hg, and PO2 55 mm Hg on ambient air. Spirometry shows an FEV1 of 25% of predicted. Chest radiograph shows hyperinflation but no infiltrates. Which of the following is the most appropriate therapy for this patient? A Continuous oxygen B Nocturnal oxygen C Oxygen as needed D Oxygen during exercise

A In a patient with severe chronic obstructive pulmonary disease, at-rest oxygen saturation less than or equal to 88% is an indication for long-term continuous oxygen therapy. The long-term administration of oxygen for more than 15 hours per day to patients with chronic obstructive pulmonary disease (COPD) increases survival, and may also improve hemodynamics, hematologic characteristics, exercise capacity, lung mechanics, and mental status. Indications for continuous long-term oxygen therapy for patients with COPD include: * PO2 less than or equal to 55 mm Hg or oxygen saturation less than or equal to 88% * Po2 less than or equal to 59 mm Hg or oxygen saturation less than or equal to 89% if there is evidence of cor pulmonale, right heart failure, or erythrocytosis (hematocrit greater than 55%). This patient's resting oxygen saturation is 86% and his PO2 is 55 mm Hg, and, therefore, continuous long-term oxygen therapy is indicated. Chronic hypoxemia leading to the development of cor pulmonale portends a poor prognosis. Nocturnal oxygen therapy is better than no oxygen therapy at all, but continuous therapy is better than nocturnal therapy in severely hypoxemic patients with erythrocytosis, elevated pulmonary artery pressures, and respiratory acidosis. No study has shown a survival benefit when oxygen is prescribed for exercise-induced oxygen desaturation or when used as needed for symptoms of breathlessness.

Pneumo 69 A 21-year-old man was brought to the emergency department obtunded with a temperature of 38.9 °C (102.0 °F) and a blood alcohol level of 0.3 g/dL (65 mmol/L). He was intubated and placed on mechanical ventilation with a tidal volume of 10 mL/kg ideal body weight. His oxygen saturation was 97% on positive end-expiratory pressure (PEEP) of 5 cm H2O and FiO2 0.4. Chest radiograph showed an infiltrate in the right lower lobe, and therapy with ceftriaxone and clindamycin was started. Now, 48 hours after admission, he develops increased respiratory distress. He is afebrile; blood pressure is 120/60 mm Hg, the pulse rate is 115/min, and the respiration rate is 32/min (14/min 24 hours ago). Oxygen saturation is 90% on FiO2 0.7 and PEEP of 12 cm H2O. The patient is sedated and has inspiratory crackles bilaterally. Jugular venous pressure is 7 cm H2O. There are minimal endotracheal secretions, and no focal neurologic findings. A new chest radiograph shows a normal cardiac silhouette and no effusions. The right lower lobe infiltrate has improved, but there are new bilateral infiltrates. Which of the following is the most appropriate management for this patient? A Adjust tidal volume to 6 mL/kg ideal body weight B Change current antibiotics to vancomycin, levofloxacin, and meropenem C Extubate the patient and start noninvasive positive-pressure ventilation D Insert a pulmonary artery catheter

A A lung-protective ventilator strategy employing low tidal volumes reduces mortality in patients with the acute respiratory distress syndrome. The patient has developed the acute respiratory distress syndrome (ARDS) complicating aspiration pneumonia. A lung-protective ventilator strategy should be used to reduce the risk of ventilator-induced lung injury by minimizing alveolar overdistention. A randomized controlled trial in patients with acute lung injury compared ventilation with tidal volumes of 6 mL/kg and 12 mL/kg ideal body weight and found an absolute mortality reduction of 8% in the low-tidal-volume group. The cause of the patient's bilateral infiltrates is ARDS rather than poorly responding pneumonia. His fever curve and right lower lobe infiltrate have improved, and he has minimal secretions. A change in antibiotic therapy is not indicated. Noninvasive positive-pressure ventilation (NPPV) would not be appropriate. He is sedated and has rapidly deteriorating gas exchange. Furthermore, ARDS is not rapidly reversible, and previous trials of NPPV in patients with ARDS have not shown a mortality benefit. Recent trials have found no advantage to using pulmonary artery catheters compared with central venous catheters for fluid management in acute lung injury.

Pneumo 86 A 47-year-old woman is evaluated in the intensive care unit for sepsis. She was admitted with severe lower lobe pneumonia and is in septic shock despite receiving 8 L of fluid. She has a history of stage IIA colon cancer treated with resection. She is receiving vasopressors and mechanical ventilation; neurologic examination shows no focal abnormalities. Her oxygen saturation is 88% on an FiO2 of 0.75 with a positive end-expiratory pressure of 12 cm H2O. Her extremities are becoming cold and show signs of livedo reticularis. The nasogastric return reveals more than 250 mL of bright red blood. She receives an additional 2 L of fluid, but her blood pressure remains 72/48 mm Hg on norepinephrine. Laboratory studies: Hemoglobin 9 g/dL (90 g/L) Leukocyte count 15,500/µL (15.5 × 109/L) Platelet count 50,000/µL (50 × 109/L) INR 2.1 Which of the following would be a contraindication to therapy with activated protein C in this patient? A Active gastrointestinal bleeding B INR of 2.1 C Platelet count of 50,000/µL (50 × 109/L) D Resected colon cancer in remission

A Activated protein C has anticoagulant properties, and evidence of active bleeding is a contraindication to its use in patients with sepsis. All the contraindications for therapy with activated protein C (APC, drotrecogin alfa activated) therapy refer to bleeding risk. Heme-positive upper gastrointestinal aspirate or fecal occult blood testing in the absence of active bleeding (often present in intensive care unit patients) is not a contraindication to APC, but this patient's moderate upper gastrointestinal bleeding is a contraindication. As a rule of thumb, when systemic heparin is contraindicated, APC is also contraindicated. A low platelet count (or elevated INR) is evidence of coagulopathy, which is a good reason to give APC (relative risk reduction of greater than 30%) as long as the platelet count is greater than 30,000/µL (30 × 109/L) and the INR is 3.0 or less. An active malignancy that would predict less than 1 month of survival was an exclusion in the PROWESS study of the efficacy and safety of APC, and such patients were not investigated because they often have do-not-resuscitate orders early in severe life-threatening acute illness such as sepsis. In this case, a history of colon cancer in remission would not be a contraindication.

Pneumo 34 A 19-year-old woman is evaluated in the emergency department for low-grade fever, muscle aches, cough, and progressively severe shortness of breath of 1 week's duration. She requires intubation and mechanical ventilation. The patient recently moved to the United States from Japan. She started smoking cigarettes 3 weeks ago and smokes a pack a day. On physical examination, the temperature is 37.2 °C (99.0 °F), the blood pressure is 128/60 mm Hg, the pulse rate is 110/min, and the respiration rate is 22/min on mechanical ventilation. Cardiac examination is normal. There are bilateral crackles posteriorly. The rest of the general physical examination is normal. The hemoglobin is 14.3 g/dL (143 g/L); the leukocyte count is 16,888/µL (16.9 × 109/L) with 52% neutrophils, 4% monocytes, 20% lymphocytes, and 24% eosinophils; and the platelet count is 345,000/µL (345 × 109/L). Chest CT scan with intravenous contrast shows bilateral focal areas of consolidation with scattered ground-glass opacification, small bilateral pleural effusions, and no evidence of pulmonary embolism. Bronchoscopy with bronchoalveolar lavage shows no organisms on Gram stain or fungal stain; cell count is elevated, and the differential shows 30% neutrophils, 6% macrophages, 4% lymphocytes, and 60% eosinophils. Blood and sputum cultures are negative after 2 days. Which of the following is the most likely diagnosis? A Acute eosinophilic pneumonia B Churg-Strauss syndrome C Idiopathic acute interstitial pneumonia D Mycoplasma pneumonia

A Acute eosinophilic pneumonia may present as hypoxemic respiratory failure after 1 to 2 weeks of low-grade fever and systemic symptoms. This patient's dramatic presentation is typical of acute eosinophilic pneumonia, which must be differentiated from other causes of idiopathic acute respiratory distress syndrome. Treatment of respiratory failure due to acute eosinophilic pneumonia with intravenous corticosteroids is efficacious, with clinical improvement over 12 to 24 hours. If a patient does not respond to corticosteroid therapy, another diagnosis should be considered. Acute eosinophilic pneumonia is idiopathic, but it has been associated with inhaled environmental antigens. Recently, initiation of cigarette smoking has been linked to acute eosinophilic pneumonia in a Japanese population and in military recruits. This patient does not have asthma or evidence of systemic vasculitis, making Churg-Strauss syndrome unlikely. Acute interstitial pneumonia, which occurs over 1 to 3 weeks, progresses to hypoxemic respiratory failure and is associated with fever, nonproductive cough, headache, myalgia, and a flu-like malaise. Chest radiograph shows diffuse, bilateral air-space infiltrates. Pathologically, a pattern of organizing diffuse alveolar damage is seen. Treatment is usually supportive, with intravenous pulse corticosteroids frequently used but of unproven benefit. Idiopathic acute interstitial pneumonia is a diagnosis of exclusion and would not be appropriate for this patient with eosinophilic pneumonia. Mycoplasma pneumonia may be culture-negative and present with respiratory symptoms. However, these symptoms rarely progress to respiratory failure and would not explain this patient's eosinophilia.

Pneumo 2 A 73-year-old woman is evaluated in the emergency department for a 2-week history of worsening dyspnea and a dry cough. She has not had fever or any recent travel. Idiopathic pulmonary fibrosis was diagnosed 2 years ago by open lung biopsy. She also has a history of hypertension and gastroesophageal reflux disease. Her medications are prednisone, diltiazem, hydrochlorothiazide, and omeprazole. On physical examination, she is afebrile; the blood pressure is 142/86 mm Hg, the pulse rate is 97/min, the respiration rate is 28/min, and the BMI is 27. Oxygen saturation with the patient breathing oxygen, 10 L/min by face mask, is 90%. There are dry crackles at the lung bases extending half way up the chest bilaterally. Cardiac and abdominal examinations are normal. Gram stain of sputum is negative; culture is pending. CT scan of the chest is negative for pulmonary embolism but shows new areas of alveolar infiltrates and consolidation superimposed on previous basilar, reticular, and honeycomb changes. Which of the following is the most appropriate next test in the evaluation of this patient? A Bronchoscopy with bronchoalveolar lavage B Fungal serologies C Right-heart catheterization D Swallowing evaluation

A Bronchoalveolar lavage is the diagnostic procedure to exclude opportunistic infection in an apparent acute exacerbation of idiopathic pulmonary fibrosis. The two immediate diagnostic considerations in this patient are respiratory infection and an acute exacerbation of pulmonary fibrosis. Both diagnostic possibilities may be evaluated by bronchoalveolar lavage with studies to detect bacterial organisms, opportunistic pathogens (for example, Pneumocystis jirovecii), and viral pathogens. Routine sputum evaluation for Gram stain and culture is not sensitive enough to detect opportunistic infectious organisms. Diagnostic criteria for an acute exacerbation of pulmonary fibrosis include exclusion of opportunistic respiratory infections via endotracheal aspiration or bronchoalveolar lavage as well as exclusion of pulmonary embolism, left ventricular failure, and other causes of acute lung injury. The incidence of an acute exacerbation of idiopathic pulmonary fibrosis is not certain but likely ranges between 5% and 40%. In patients with pulmonary fibrosis admitted to the intensive care unit for respiratory failure, the incidence may be as high as 60% with a reported mortality rate between 80% and 100%. No therapy has been shown to be beneficial. Fungal serologies may be helpful to diagnose opportunistic infection in this patient. However, bronchoalveolar lavage is more sensitive, and results more readily available for detecting other opportunistic pathogens that need to be excluded in this immunosuppressed patient. Right-heart catheterization is not part of the initial evaluation of patients with a suspected acute exacerbation of idiopathic pulmonary fibrosis. Aspiration may cause acute lung injury and may trigger an acute exacerbation of idiopathic pulmonary fibrosis, but swallowing evaluation is unlikely to be diagnostic in this patient with no history of aspiration.

Pneumo 19 A 24-year-old woman with persistent asthma, which is well controlled on low-dose fluticasone and albuterol as needed, became pregnant 2 months ago and asks for advice about asthma therapy during her pregnancy. Before she started fluticasone therapy, she had frequent asthma symptoms and occasional exacerbations requiring emergency department treatment. Since she became pregnant, her asthma has remained under good control. The physical examination is unremarkable, and spirometry is normal. Which of the following is the most appropriate management for this patient? A Continue the current regimen B Stop fluticasone; add theophylline C Stop fluticasone; add salmeterol D Stop fluticasone; add inhaled cromolyn

A Clinical experience has shown that inhaled corticosteroids are safe and effective in pregnant patients with asthma. Asthma during pregnancy follows the rule of thirds: the condition improves in one third of patients, worsens in one third, and remains unchanged in one third. Uncontrolled asthma has significantly worse impact on pregnancy outcome than the potential risk of medications during pregnancy. Short-acting β-agonists are regarded as safe during pregnancy. Budesonide has been studied in pregnancy and been shown to be safe. There are fewer data on other inhaled corticosteroids, such as fluticasone, which is a U.S. Food and Drug Administration pregnancy risk category C drug (studies of safety in pregnancy are lacking but the potential benefit of the drug may justify the potential risk). The inhaled corticosteroids are, however, believed from clinical experience to be safe during pregnancy, and, therefore, it is generally recommended to keep the patient on the regimen that has been effective for control of asthma. Theophylline and aminophylline are pregnancy risk category C drugs also, but extensive clinical experience suggests that they are safe during pregnancy. However, the metabolism of these agents may be altered in pregnancy, requiring increased drug level monitoring. Also, inhaled corticosteroids are as effective as theophylline with fewer side effects in pregnant patients. Cromolyn is also considered safe in pregnancy but no safer than inhaled corticosteroids and less effective in persistent asthma. The National Asthma Education and Prevention Program (NAEPP) expert panel guidelines in 2007 affirmed the recommendation of adding long-acting β-agonists to patients whose asthma is not controlled with an inhaled corticosteroid but advised against using long-acting β-agonists as a single controller therapy. There is no need to add a long-acting β-agonist to this patient's asthma regimen because her symptoms are well controlled and substituting the long-acting β-agonist for inhaled fluticasone may result in loss of symptom control and possible increased risk of asth

Pneumo 20 A 64-year-old woman is evaluated for a 6-week history of dyspnea, dry cough, fever, chills, night sweats, and fatigue, which have not responded to treatment with azithromycin and levofloxacin; she has lost 2.2 kg (5 lb) during that time. The patient had a thorough examination 6 months ago while she was asymptomatic that included routine laboratory studies, age- and sex-appropriate cancer screening, and a chest radiograph; all results were normal. The patient has never smoked, has had no known environmental exposures, and has not traveled recently or been exposed to anyone with a similar illness. Her only medications are aspirin and a multivitamin. On physical examination, temperature is 37.8 °C (100.0 °F); other vital signs are normal. Cardiac examination is normal. There are scattered crackles in the mid-lung zones with associated rare expiratory wheezes. There is no digital clubbing. Musculoskeletal and skin examinations are normal. Chest radiograph is shown . Which of the following is the most likely diagnosis? A Cryptogenic organizing pneumonia B Idiopathic pulmonary fibrosis C Lymphocytic interstitial pneumonia D Nonspecific interstitial pneumonia

A Cryptogenic organizing pneumonia most often presents with subacute disease progression and bilateral opacities on chest radiograph. This nonsmoker without any exposure history has acute to subacute development of nonspecific systemic and respiratory symptoms with a dominant alveolar (opacification) process on chest radiograph. The tempo of the disease process is the key to differentiating cryptogenic organizing pneumonia (COP) from other interstitial lung diseases. COP (formerly called idiopathic bronchiolitis obliterans organizing pneumonia) is often acute or subacute, with symptom onset occurring within 2 months of presentation in three fourths of patients. The presentation is so suggestive of an acute or subacute lower respiratory tract infection that patients have almost always been treated with and failed to respond to one or more courses of antibiotics before diagnosis. Idiopathic pulmonary fibrosis (IPF), nonspecific interstitial pneumonia (NSIP), or lymphocytic interstitial pneumonia (LIP) typically follows a prolonged course with evidence of respiratory symptoms and radiographic findings that progress slowly over months or years. Radiographic findings in COP are also distinct from those in IPF, NSIP, and LIP. A dominant alveolar opacification process is typically present in patients with COP. The opacities are almost always bilateral with varied distribution. One of the key radiographic features of COP is the tendency for COP opacities to "migrate" or involve different areas of the lung on serial examinations. Although the radiographic findings of IPF, NSIP, and LIP are varied, they all have a dominant interstitial (reticular) pattern with or without opacities. LIP (which is very rare) is one of the few interstitial lung diseases that can present with cystic changes on high-resolution CT.

Pneumo 8 A 70-year-old man is evaluated in the emergency department for a 2-day history of dyspnea with exertion, orthopnea, and paroxysmal nocturnal dyspnea. He has ischemic heart disease with left ventricular dysfunction and had coronary artery bypass graft surgery 6 weeks ago. His medications include aspirin, nitroglycerin, metoprolol, lisinopril, and furosemide. On physical examination, the patient is sitting upright and breathing with difficulty; the temperature is 37 °C (98.6 °F), the blood pressure is 150/85 mm Hg, the pulse rate is 105/min and regular, and the respiration rate is 28/min. Oxygen saturation is 89% on ambient air. There are fine crackles at the lung bases bilaterally, and breath sounds are diminished at the right base. There is a regular tachycardia and an S3 at the apex. There is no jugular venous distention or peripheral edema. Hemoglobin is 12.5 g/dL (125 g/L), and the leukocyte count is 10,500/µL (10.5 × 109/L). Chest radiograph shows cardiomegaly and small bilateral pleural effusions, greater on the right than the left. Thoracentesis is performed, and pleural fluid analysis shows: Nucleated cell count: 450/µL with 3% neutrophils, 70% lymphocytes, 10% macrophages, 15% mesothelial cells, and 2% eosinophils. Pleural fluid to serum total protein ratio: 0.54 Lactate dehydrogenase (LDH): 125 U/L Pleural fluid to upper limits of normal serum LDH ratio 0.52 Glucose: 80 mg/dL (4.44 mmol/L) Total protein: 3.7 g/dL (37 g/L) pH : 7.45 Albumin: 1.5 g/dL (15 g/L) Cholesterol: 35 mg/dL (0.9 mmol/L) The serum-pleural fluid albumin gradient is 1.7. Which of the following is the most likely diagnosis? A Heart failure B Parapneumonic effusion C Post-cardiac injury syndrome D Pulmonary embolism

A Diuretic therapy for heart failure can result in either a protein- or lactate dehydrogenase-discordant exudative pleural effusion and, rarely, a concordant exudate. The patient's pleural fluid analysis shows a protein discordant exudate (an exudate by protein criterion only) with a pleural fluid to serum total protein ratio of 0.54 and a pleural fluid lactate dehydrogenase (LDH) to upper limits of normal serum LDH ratio of 0.52. Pleural fluid findings may have exudative characteristics in patients with heart failure who are receiving diuretics. A serum-pleural fluid albumin gradient greater than 1.2 suggests a transudate in cases where the pleural fluid to serum total protein ratio or pleural fluid to serum LDH ratio and pleural fluid LDH to upper limits of normal serum LDH ratio suggest an exudate, but the clinical findings suggest a transudate. The increased pleural fluid to serum total protein ratio is the result of a diuretic effect, with more efficient clearance of pleural liquid than pleural protein. Patients with post-cardiac injury syndrome typically present 3 weeks (range 3 days to 1 year) after coronary artery bypass graft surgery; they usually have pleuritic chest pain and typically dyspnea, pleural or pericardial friction rub, fever, left lower lobe infiltrates, leukocytosis, and an increased erythrocyte sedimentation rate. This patient's findings are not compatible with post-cardiac injury syndrome. The absence of chest pain would be highly unlikely with a pulmonary embolism-induced pleural effusion. No consolidation was detected on chest radiograph, making pneumonia unlikely. Furthermore, a parapneumonic effusion is typically a concordant exudate (both protein and LDH in the exudate range) with a neutrophil predominance, and a low pleural fluid LDH is typically not associated with an acute parapneumonic effusion.

Pneumo 50 A 51-year-old woman is evaluated after a pulmonary nodule was detected in the emergency department where she was evaluated for substernal chest pain. Serial electrocardiograms and measurements of cardiac enzymes were negative for ischemia or infarction. A subsequent exercise stress test showed no ischemia. CT scan of the chest showed a 2.6-cm lobulated right upper lobe lung nodule and no lymphadenopathy. The patient smoked a pack of cigarettes a day for 18 years but stopped smoking 4 years ago. She is otherwise healthy. Her father died of lung cancer at age 63 years. On physical examination, vital signs are normal and the lungs are clear, there is no palpable lymphadenopathy, and the abdomen is soft and without organomegaly. Positron emission tomography-CT shows that the nodule is metabolically active with significant tracer uptake in the nodule as well as a 1-cm right hilar lymph node and a 9-mm right paratracheal lymph node. Which of the following is the most appropriate management for this patient? A Bronchoscopy with ultrasonography-guided node sampling B Chemotherapy C CT-guided needle biopsy of the lung nodule D Surgical staging with mediastinoscopy

A Endobronchial ultrasonography-guided and endoscopic ultrasonography-guided node sampling are less invasive alternatives to surgical staging of the mediastinum with mediastinoscopy in a patient with suspected lung cancer. The patient likely has non-small cell lung cancer, and the likely clinical stage is IIIA because positron emission tomography (PET)-CT findings support mediastinal lymph node involvement despite the normal size of the nodes on CT. However, PET-CT positivity is not equivalent to a histologic diagnosis, and tissue is needed; therefore, referral for chemotherapy would be premature. The selection of a biopsy site should take into consideration the ability of the results to establish a tissue diagnosis and to stage the disease. The site for biopsy should be a lesion that has the potential to establish the highest disease stage in order to decrease the number of procedures for the patient. Therefore, the next step in management is to establish a diagnosis and pathologic stage with mediastinal lymph node sampling. Endobronchial ultrasonography-guided node sampling is a less invasive means than mediastinoscopy (a surgical procedure) and has a high yield. A positive lymph node biopsy establishes advanced-stage disease, eliminates the need for surgical staging, and prompts referral to medical oncology. Endoscopic ultrasonography-guided sampling through the esophagus has shown similar results for accessible nodes and may be more available in some centers. A negative result at bronchoscopy or endoscopy would result in surgical staging of the mediastinum by mediastinoscopy or thoracotomy. CT-guided needle aspiration of the nodule would likely establish the diagnosis but not the stage, and a second procedure would be needed to stage the mediastinum.

Pneumo 55 A 60-year-old man is evaluated in the emergency department for a 3-hour history of shaking chills and cough productive of purulent sputum. He has a 35-pack-year history of cigarette smoking but has no other significant medical history. On physical examination, the temperature is 38.5 °C (101.4 °F), the blood pressure is 150/90 mm Hg, the pulse rate is 115/min and regular, and the respiration rate is 28/min. There is increased dullness to percussion at the right base, crackles in the right mid-lung field, and anterior wheezes. Chest radiograph shows a right lower lobe posterior infiltrate and a moderate-sized pleural effusion; diagnostic thoracentesis reveals a turbid pleural fluid. Analysis of the fluid shows: Erythrocyte count 750/µL Nucleated cell count 18,000/µL (18 × 109/L) with 87% neutrophils, 5% lymphocytes, 4% macrophages, and 4% mesothelial cells Total protein 4.5 g/dL (45 g/L) Lactate dehydrogenase 1000 U/L pH 7.20 Glucose 50 mg/dL (2.8 mmol/L) Ultrasonography at the time of thoracentesis showed complex septations in the pleural fluid. Cytology is pending. Therapy with ceftriaxone, azithromycin, and inhaled bronchodilators is started. Which of the following is the most appropriate management for this patient? A Admit to the hospital and perform pleural cavity drainage B Admit to the hospital and repeat chest radiograph in 2 days C Admit to the hospital for video-assisted thoracoscopic surgery D Treat as an outpatient

A Immediate pleural space drainage, with or without fibrinolytic therapy, is indicated in a patient with acute bacterial pneumonia and a parapneumonic effusion with septations. Pleural infections often resolve with antibiotic therapy alone, but fibrinous organization and lung entrapment require invasive treatment in about 10% of parapneumonic effusions. Effusions at risk for loculation are called complicated parapneumonic effusions. Because clinical prediction is unreliable, thoracentesis should be done to assess the need for invasive treatment provided that the effusion can be clearly visualized on ultrasonography. This patient with presumed underlying chronic obstructive pulmonary disease had typical symptoms of an acute bacterial pneumonia with development of a parapneumonic effusion with septations, low pleural fluid pH, low glucose, and elevated lactate dehydrogenase level. All of these factors suggest that the patient will have a poor outcome without immediate pleural space drainage with either thoracostomy tube placement or a radiologically guided small-bore catheter. The evidence for use of fibrinolytic agents is still unclear, but in this situation, pleural space drainage is imperative and, with the ultrasonographic findings, a trial of a fibrinolytic agent is warranted before performing video-assisted thoracoscopic surgery (VATS). Intrapleural administration of fibrinolytic agents does not cause systemic thrombolysis. Early VATS or thoracotomy is the generally preferred approach for patients who are candidates for surgery and who have persistent sepsis and loculation. The patient should not be discharged; he needs close follow-up for possible escalation of therapy. He should not be sent for surgery immediately without a trial of chest tube drainage with or without fibrinolytic therapy. If clinical improvement is not obvious by 3 days, surgical consultation should be initiated.

Pneumo 3 A 65-year-old woman is evaluated in a follow-up examination for dyspnea, chronic cough, and mucoid sputum; she was diagnosed with chronic obstructive pulmonary disease 3 years ago. The patient has a 40-pack-year history of cigarette smoking, but quit smoking 1 year ago. She is otherwise healthy, and her only medication is inhaled albuterol as needed. On physical examination, vital signs are normal. Breath sounds are decreased, but there is no edema or jugular venous distention. Spirometry shows an FEV1 of 62% of predicted and an FEV1/FVC ratio of 65%. Chest radiograph shows mild hyperinflation. Which of the following is the most appropriate therapy for this patient? A Add a long-acting β2-agonist B Add an inhaled corticosteroid C Add an oral corticosteroid D Add theophylline and montelukast E Continue current albuterol therapy

A In patients with moderate chronic obstructive pulmonary disease, therapy with a long-acting β2-agonist or a long-acting anticholinergic agent improves quality of life and pulmonary function compared with therapy with short-acting bronchodilators alone. This patient has stage II chronic obstructive pulmonary disease (COPD) as defined by the guidelines of the Global Initiative for Obstructive Lung Disease (GOLD). GOLD stage II disease is defined by a postbronchodilator FEV1/FVC ratio less than 70% and an FEV1 less than 80% but more than 50% of predicted with or without chronic symptoms. In patients with GOLD stage II disease, maintenance treatment with one or more long-acting bronchodilators such as a long-acting β2-agonist (salmeterol or formoterol) is recommended, along with as-needed albuterol. Pulmonary rehabilitation can be considered in addition to medical treatment in symptomatic patients. Starting or adding a long-acting anticholinergic agent would also be appropriate. Inhaled corticosteroids, oral corticosteroids, theophylline, and montelukast would be inappropriate for this patient. Theophylline's narrow therapeutic window and poor bronchodilator effect make it a poor choice. Oral corticosteroids are not recommended routinely in COPD because of their systemic side effects. Montelukast is used as a third-line agent in asthma but has not been shown to be efficacious in COPD. The GOLD guidelines recommend consideration of inhaled corticosteroids in patients whose lung function is less than 50% and who experience recurrent exacerbations. When inhaled corticosteroids are combined with a long-acting β2-agonist in such patients, the rate of decline in quality of life and health status is significantly reduced and the frequency of acute exacerbations is reduced by 25%; lung function is also improved and dyspnea alleviated. Continuing current therapy with albuterol would not be appropriate because using a short-acting bronchodilator alone does not afford effective therapy for GOLD stage II disease and requires more frequent treatments. Two recent long-term studies of more than 11,000 patients have evaluated treatment for patients with GOLD stages II to IV. The TORCH study and the UPLIFT study showed that in patients with COPD, therapy with a long-acting β2-agonist or a long-acting anticholinergic agent in addition to a short-acting bronchodilator improved quality of life and pulmonary function without significant adverse effects.

Pneumo 99 A 71-year-old woman is brought to the emergency department from a nursing home because of confusion, fever, and flank pain. Her temperature is 38.5 °C (101.3 °F), blood pressure is 82/48 mm Hg, pulse rate is 123/min, and respiration rate is 27/min. Mucous membranes are dry, and there is costovertebral angle tenderness, poor skin turgor, and no edema. Hemoglobin concentration is 10.5 g/dL (105 g/L), leukocyte count is 15,600/µL (15.6 × 109/L); urinalysis reveals 50 to 100 leukocytes/hpf and many bacteria/hpf. The patient has an anion gap metabolic acidosis. A central venous catheter is placed, and antibiotic therapy is started. Which of the following additional interventions is most likely to improve survival for this patient? A Aggressive fluid resuscitation B Hemodynamic monitoring with a pulmonary artery catheter C Maintaining hemoglobin concentration above 12 g/dL (120 g/L) D Maintaining PCO2 below 50 mm Hg

A In patients with severe sepsis, early goal-directed therapy within the first 6 hours to maintain a central venous or mixed venous oxygen saturation of greater than 70% and to resolve lactic acidosis improves survival compared with more delayed resuscitation attempts. The patient has severe sepsis presumptively from pyelonephritis. Aggressive fluid resuscitation with resolution of lactic acidosis within 6 hours would have a beneficial effect on this patient's survival. Resuscitation of the circulation should target a central venous oxygen saturation (SCVO2) or mixed venous oxygen saturation (SVO2) of at least 70%. Other reasonable goals include a central venous pressure of 8 to 12 mm Hg, a mean arterial pressure of at least 65 mm Hg, and a urine output of at least 0.5 mL/kg/h. In patients such as the one presented, this often translates into administration of 5 to 6 L of fluid. Timing of resuscitation matters to survival. In a landmark study by Rivers and colleagues, early goal-directed therapy that included interventions within the first 6 hours to maintain a SCVO2 of greater than 70% and to resolve lactic acidosis resulted in higher survival rates than more delayed resuscitation attempts. Over the first 72 hours, patients in the control arm received the same quantity of fluid for resuscitation, but they had a significantly higher likelihood of dying by discharge or at 60 days. Crystalloid is given much more frequently than colloid, and there are no data to support routinely using colloid in lieu of crystalloid. Blood transfusion may be part of resuscitation for anemic patients in shock, but maintaining hemoglobin levels above 12 g/dL (120 g/L) is not supported by evidence. In stable patients who are not in shock, a transfusion threshold of 7 g/dL (70 g/L) is an acceptable conservative approach. There are no data to support that maintaining a lower PCO2 or using a pulmonary artery catheter would help to increase survival in this patient.

Pneumo 28 A 55-year-old man with a 7-year history of severe chronic obstructive pulmonary disease is evaluated after being discharged from the hospital following an acute exacerbation; he has had three exacerbations over the previous 18 months. He is a long-term smoker who stopped smoking 1 year ago. He adheres to therapy with albuterol as needed and inhaled salmeterol and tiotropium and has demonstrated proper inhaler technique. On physical examination, vital signs are normal. Breath sounds are decreased bilaterally; there is no edema or cyanosis. Oxygen saturation after exertion is 92% on ambient air. Spirometry shows an FEV1 of 32% of predicted and an FEV1/FVC ratio of 40%. Chest radiograph done in the hospital 3 weeks ago showed no active disease. Which of the following should be added to this patient's therapeutic regimen? A An inhaled corticosteroid B Ipratropium C N-acetylcysteine D Oral prednisone

A Inhaled corticosteroids may offer significant benefit in patients with severe chronic obstructive pulmonary disease, with the benefit generally greater when an inhaled corticosteroid is combined with a long-acting β2-agonist. Regular use of inhaled corticosteroids in patients with chronic obstructive pulmonary disease (COPD) is associated with a reduction in the rate of exacerbations from 1.3 to 0.9 per year, and patients who have frequent exacerbations with more severe COPD benefit most. In six placebo-controlled trials in 1741 patients over 6 months, inhaled corticosteroids reduced exacerbations by 24%. Therefore, the GOLD guidelines recommend consideration of inhaled corticosteroids in patients whose lung function is less than 50% and those who have exacerbations. When inhaled corticosteroids are combined with a long-acting β2-agonist, the rate of decline in quality of life and health status is significantly reduced and acute exacerbations are reduced by 25%; lung function is also improved and dyspnea is alleviated. There does not appear to be a dose response to inhaled corticosteroids in COPD, and the effects of combination therapy on mortality are uncertain. Anticholinergic agents in COPD are especially useful when combined with short-acting or long-acting β2-agonists. Tiotropium is effective in patients with stable COPD for up to 24 hours and should not be combined with short-acting anticholinergic agents, such as ipratropium. Mucolytic agents have little effect on lung function. The antioxidant N-acetylcysteine, a drug with both mucolytic and antioxidant action, did not reduce the number of exacerbations of COPD in a large prospective 3-year trial. Oral corticosteroids are not recommended for regular use in a long-term maintenance program because their use is not associated with superior outcomes compared with standard therapy and is associated with increased side effects.

Pneumo 18 A 65-year-old man is admitted to the intensive care unit for gram-negative sepsis. The patient's medical history is significant only for hyperthyroidism for which he takes methimazole. On day 2 in the intensive care unit, he undergoes rapid sequence intubation with propofol and succinylcholine for worsening hypoxemic respiratory failure resulting from the acute respiratory distress syndrome. The patient receives intermittent lorazepam and fentanyl boluses intravenously for sedation. Several hours later, the patient becomes febrile (temperature 40 °C [104 °F]), hypertensive, and tachycardic. On examination, he is diaphoretic and has muscular rigidity. Arterial blood gas analysis shows a metabolic and respiratory acidosis, and laboratory results are significant for an elevated serum creatine kinase level. Which of the following is the most likely cause of the patient's clinical deterioration? A Malignant hyperthermia B Neuroleptic malignant syndrome C Serotonin syndrome D Thyroid storm

A Malignant hyperthermia is an inherited skeletal muscle disorder characterized by a hypermetabolic state precipitated by exposure to volatile inhalational anesthetics and the depolarizing muscle relaxants. This patient most likely has malignant hyperthermia, which is an inherited skeletal muscle disorder characterized by a hypermetabolic state precipitated by exposure to volatile inhalational anesthetics (halothane, isoflurane, enflurane, desflurane, sevoflurane) and the depolarizing muscle relaxants succinylcholine and decamethonium. It usually occurs on exposure to the drug but can occur several hours after the initial exposure and can develop in patients who were previously exposed to the drug without effect. Increased intracellular calcium leads to sustained muscle contractions, with skeletal muscle rigidity and masseter muscle spasm, tachycardia, hypercarbia, hypertension, hyperthermia, tachypnea, and cardiac arrhythmias. Rhabdomyolysis and acute renal failure can develop. Malignant hyperthermia should be suspected in patients with a family history of problems during anesthesia. The neuroleptic malignant syndrome is a life-threatening disorder caused by an idiosyncratic reaction to neuroleptic tranquilizers (dopamine D2-receptor antagonists) and some antipsychotic drugs. The most common offending neuroleptic agents are haloperidol and fluphenazine. The syndrome occurs with all drugs that cause central dopamine receptor blockade, usually soon after starting a new drug or with dose escalation. It has been reported in patients with Parkinson disease who abruptly discontinue levodopa or anticholinergic therapy. Most patients with the syndrome develop muscle rigidity, hyperthermia, cognitive changes, autonomic instability, diaphoresis, sialorrhea, seizures, arrhythmias, and rhabdomyolysis within 2 weeks after initiating the drug. Because this patient did not receive a neuroleptic agent, neuroleptic malignant syndrome is unlikely. In critical care patients receiving both neuroleptic tranquilizers and depolarizing muscle relaxants, malignant hyperthermia can be differentiated from neuroleptic malignant syndrome by the presence of a mixed (metabolic and respiratory) acidosis in the former condition. Like the neuroleptic malignant syndrome, the serotonin syndrome presents with high fever, muscle rigidity, and cognitive changes. Findings unique to the serotonin syndrome are shivering, hyperreflexia, myoclonus, and ataxia. The serotonin syndrome is caused by the use of selective serotonin reuptake inhibitors, a category of drug that this patient has not been exposed to. Thyroid storm is a potential cause of hyperthermia in hospitalized patients, but thyroid storm does not cause muscle rigidity or elevations of the creatine kinase level.

Pneumo 94 A 60-year-old man with an 8-year history of chronic obstructive pulmonary disease is evaluated for a 3-month history of weight loss of 4.4 kg (10 lb), increased fatigue, and decreased exercise capacity. He has had multiple hospitalizations for exacerbations of his disease, including one 2 months ago. He stopped smoking 1 year ago and recently completed a 6-month pulmonary rehabilitation program without much benefit. The patient also has hypertension, and his medications are inhaled albuterol as needed; salmeterol; tiotropium; an inhaled corticosteroid; theophylline; oxygen, 2 L/min by nasal cannula; and enalapril. On physical examination, vital signs are normal; BMI is 20. Breath sounds are decreased, and heart sounds are distant. There is 1+ bilateral pitting edema, and the 6-minute walk distance is 90 meters. Spirometry shows an FEV1 of 18% of predicted and an FEV1/FVC ratio of 34%. DLCO is 16% of predicted, and total lung capacity is 108% of predicted. Arterial blood gases with the patient breathing oxygen are pH 7.37, PCO2 47 mm Hg, and PO2 78 mm Hg. There is no evidence of nocturnal oxygen desaturation. Chest radiograph shows hyperinflation, and chest CT scan shows homogeneous emphysema. Electrocardiography shows right ventricular hypertrophy and right atrial enlargement. Echocardiography shows normal left ventricular function; pulmonary artery pressure is slightly increased. Which of the following is the most appropriate management for this patient? A Lung transplantation B Lung volume reduction surgery C Nocturnal assisted ventilation D Repeat pulmonary rehabilitation

A Patients with homogenous emphysema, an FEV1 not greater than 20% of predicted, and DLCO not greater than 20% of predicted have a median survival of 3 years and should be considered for lung transplantation. This patient is a candidate for transplantation. Functional capacity and quality of life are improved after lung transplantation, but the effect on overall survival is mixed. The National Emphysema Therapeutic Trial of lung volume reduction surgery reported mixed results on patient outcomes. For patients with very reduced lung function (FEV1 less than 20%), low DLCO (less than 20%), and diffuse disease, the mortality rate was greater with surgery than with medical therapy. However, in patients with predominant upper lobe emphysema and low baseline exercise tolerance, functional capacity and survival improved. Patients with homogeneous emphysema, FEV1 less than 20% of predicted, and DLCO less than 20% of predicted have high mortality rates with lung volume reduction surgery. This patient did not benefit from 6 months of pulmonary rehabilitation and is unlikely to benefit from additional rehabilitation. Patients with severe chronic obstructive pulmonary disease, daytime hypercapnia, and nocturnal oxygen desaturation despite supplemental oxygen might benefit from the use of nocturnal assisted ventilation, but this patient does not have nocturnal desaturation.

Pneumo 63 A 42-year-old man who is a new patient is evaluated for a 6-month history of mild shortness of breath, which occurs primarily with exertion, and also occasional wheezing. He has smoked a half pack of cigarettes daily since the age of 18 years. He is otherwise healthy and takes no medications. He works in an automobile repair shop. His father, a cigarette smoker, died of emphysema at the age of 55 years. On physical examination, vital signs are normal; BMI is 25. Breath sounds are diminished bilaterally, and there is occasional wheezing posteriorly. Spirometry shows an FEV1 of 58% of predicted and an FEV1/FVC ratio of 65%. Chest radiograph shows bilateral basilar lucency. Which of the following is the most appropriate next step in the management of this patient? A Measure plasma α1-antitrypsin B Measure sweat chloride C Obtain a flow-volume loop D Order high-resolution CT scan of the chest

A Patients with severe α1-antitryspin deficiency are predisposed to early-onset chronic obstructive pulmonary disease, especially panacinar emphysema, which involves the lung bases. This patient may have α1-antitrypsin (AAT) deficiency, a clinically underdiagnosed disorder that primarily affects the lungs but also the liver and, rarely, the skin. AAT protects against proteolytic degradation of elastin, a protein that promotes elasticity of connective tissue. The normal plasma concentration of AAT is 150 to 350 mg/dL (1.5 to 3.5 g/L). Patients with plasma levels lower than 50 to 80 mg/dL (0.5 to 0.8 g/L) have severe deficiency. In the lungs, severe deficiency of AAT predisposes to early-onset chronic obstructive pulmonary disease, especially panacinar emphysema, which involves the lung bases. This patient is younger than 45 years and has bilateral basilar emphysema, and, therefore, AAT deficiency must be ruled out. High-resolution CT scan is not helpful in the diagnosis of AAT deficiency, although it may be useful in evaluating the extent of the disease. The sweat chloride test is a screening test for cystic fibrosis. Nearly 10% of patients diagnosed with cystic fibrosis are older than 18 years. Of these patients, gastrointestinal symptoms and infertility are the most common presenting problems. In cystic fibrosis lung disease, chest radiography typically shows hyperinflation and accentuated bronchovascular markings, appearing first in the upper lobes, followed by bronchiectasis and cyst formation. This patient's age, presenting symptoms, and chest radiograph findings make cystic fibrosis unlikely. A flow-volume loop, which includes forced inspiratory and expiratory maneuvers, is indicated for patients with unexplained dyspnea and can detect upper airway obstruction that cannot be diagnosed with spirometry. However, this patient has no physical findings suggestive of upper airway obstruction (for example, stridor), and even if such findings were present, they would not explain the patient's findings on chest radiography.

Pneumo 12 A 74-year-old man is evaluated for a 5-year history of gradually progressive dyspnea and dry cough without wheezing or hemoptysis. For the past 2 years he has had pain and occasional swelling in both knees. He has not had fever or lost weight. He smoked one pack of cigarettes a day from the age of 18 to 60 years. He worked as an insulator for 40 years. Physical examination shows no digital clubbing or cyanosis. Auscultation of the lungs reveals bilateral end-inspiratory crackles. Pulmonary function testing shows: Total lung capacity 67% of predicted Residual volume 72% of predicted FVC 65% of predicted FEV1 75% of predicted FEV1/FVC ratio 89% DLCO 52% of predicted His chest radiograph is shown . Which of the following is the most likely diagnosis? A Asbestosis B Idiopathic pulmonary fibrosis C Rheumatoid interstitial lung disease D Pulmonary sarcoidosis

A Pleural plaques are focal, often partially calcified, fibrous tissue collections on the parietal pleura and are a marker of asbestos exposure. The diagnosis of asbestosis is based on a convincing history of asbestos exposure with an appropriately long latent period (10 to 15 years) and definite evidence of interstitial fibrosis without other likely causes. This patient worked as an insulator when asbestos exposure was still widespread and is at risk for asbestos-related lung disease. The most specific finding on chest radiograph is bilateral partially calcified pleural plaques. Pleural plaques are focal, often partially calcified, fibrous tissue collections on the parietal pleura and are considered a marker of asbestos exposure. Rheumatoid lung disease has many manifestations, including an interstitial lung disease, which is most common in patients with severe rheumatoid arthritis. This patient's occasional swelling in both knees is not compatible with the diagnosis of rheumatoid arthritis. Sarcoidosis occurs most commonly in young and middle-aged adults, with a peak incidence in the third decade. More than 90% of patients with sarcoidosis have lung involvement. The chest radiograph may show hilar lymphadenopathy alone, hilar lymphadenopathy and reticular opacities predominantly in the upper lung zone, or reticular opacities without hilar lymphadenopathy. Pulmonary function tests may reveal a restrictive pattern and reduction in DLCO, or may be normal. The patient's age, predominantly lower lobe involvement, occupational history, and pleural plaques argue against pulmonary sarcoidosis. Idiopathic pulmonary fibrosis presents with slowly progressive dyspnea and a chronic, nonproductive cough. The chest radiograph is almost always abnormal at the time of presentation, with decreased lung volumes and basal reticular opacities. Almost all patients have a physiologic restrictive process (decreased forced vital capacity, total lung capacity, functional residual capacity) as well as impaired gas exchange with a decreased DLCO. However, asbestosis is a much more likely diagnosis in a patient with a positive exposure history and radiographic evidence of pleural plaques.

Pneumo 83 A 65-year-old man is being evaluated for clearance to undergo an elective repair of an abdominal aortic aneurysm. The patient has a 5-year history of moderate chronic obstructive pulmonary disease with chronic cough productive of mucoid sputum. He smokes a half pack of cigarettes a day. His medical history also includes hypertension, and he is adherent to therapy with albuterol as needed, an inhaled corticosteroid, salmeterol, and lisinopril. On physical examination, vital signs are normal; BMI is 22. The chest is normal with no wheezes or rhonchi; there is no jugular venous distention or edema. Spirometry done 6 months ago showed an FEV1 of 55% of predicted and an FEV1/FVC ratio of 55%. Spirometry done 1 week ago showed minimal change but no response to a bronchodilator. Surgery is scheduled for next week. Which of the following would most effectively reduce the patient's risk for postoperative pulmonary complications? A Incentive spirometry B Leukocyte-depleted blood transfusions C Preoperative systemic corticosteroids D Right-heart catheterization E Smoking cessation

A Routine lung expansion with incentive spirometry and deep-breathing exercises has been shown to prevent postoperative pulmonary complications. There is good evidence supporting routine lung expansion with incentive spirometry and deep-breathing exercises to prevent postoperative pulmonary complications in patients with chronic obstructive pulmonary disease. No particular pulmonary expansion modality has been found to be clearly superior to the others. The data suggest that there is a moderate increase in risk for postoperative pulmonary complications among current smokers. There is also evidence suggesting that at least 2 months of smoking cessation reduces postoperative pulmonary risk. A brief period of abstinence does not improve perioperative pulmonary outcomes, and smokers who tried to decrease cigarette use shortly before surgery are more likely to develop a postoperative pulmonary complication than those who continued smoking. The preoperative consultation can certainly be used to reinforce the benefits of smoking cessation and even to consider delaying elective surgery in patients who are ready to quit, but smoking cessation 1 week prior to the planned surgery is unlikely to be helpful in preventing postoperative pulmonary complications. Right-heart catheterization has not been found to prevent postoperative pulmonary complications. Evidence on prophylactic systemic corticosteroids before surgery for the prevention of postoperative pulmonary complications is insufficient to make a recommendation. Using leukocyte-depleted blood in transfusions has not been found to prevent postoperative pulmonary complications.

Pneumo 41 A 67-year-old man is evaluated in the emergency department for confusion and agitation secondary to malignant hypertension. Initial blood pressure is 230/130 mm Hg, and funduscopic examination reveals papilledema. He is admitted to the intensive care unit, and therapy with nitroprusside by continuous infusion is begun; the therapy is titrated over the next 3 days. The encephalopathy and papilledema resolve with control of the blood pressure. However, he becomes more confused and lethargic. The physical examination is normal. He is afebrile, with a blood pressure of 100/70 mm Hg and a pulse rate of 60/min. He is oriented only to person. There are no focal findings on the neurologic examination and no evidence of nuchal rigidity. Arterial blood gases reveal: pH 7.2; PCO2 20 mm Hg; PO2 90 mm Hg (on ambient air). Venous PO2 is 72 mm Hg. Serum electrolyte panel shows: sodium 140 meq/L (140 mmol/L); potassium 3.8 meq/L (3.8 mmol/L); chloride 90 meq/L (90 mmol/L), and bicarbonate 9 meq/L (9 mmol/L). Which of the following is the most likely cause of the patient's findings? A Cyanide toxicity B Delirium tremens C Hepatic encephalopathy D Hypoxic-ischemic encephalopathy E Wernicke encephalopathy

A Sodium nitroprusside when used in high doses or over a period of days can produce toxic blood concentrations of cyanide. Cyanide may cause toxicity through parenteral administration, smoke inhalation, oral ingestion, or dermal absorption. Sodium nitroprusside, when used in high doses or over a period of days, can produce toxic blood concentrations of cyanide. In most patients, cyanide release from sodium nitroprusside is slow enough that the body's innate detoxification mechanisms can eliminate the cyanide before it interferes with cellular respiration. However, patients with low thiosulfate reserves (for example, malnourished or postoperative patients) are at increased risk for developing symptoms, even with therapeutic dosing. A severe anion gap metabolic acidosis, combined with a reduced arterial-venous oxygen gradient (less than 10 mm Hg due to venous hyperoxia), suggests the diagnosis of cyanide toxicity. Apnea may result in a combined metabolic and respiratory acidosis. The treatment of cyanide poisoning is empiric because laboratory confirmation can take hours or days. Treatment includes administration of both sodium thiosulfate and hydroxocobalamin. Hepatic encephalopathy can cause confusion, respiratory alkalosis, and mild hypoxemia. Hypoxic-ischemic encephalopathy typically follows an obvious anoxic event such as cardiac arrest or drowning. This patient has no history of such a precipitating event. Wernicke encephalopathy is defined by confusion, ataxia, and ophthalmoplegia, but the full triad of findings is frequently absent. The first symptoms of alcohol withdrawal occur within 6 hours of the last drink and include tremors, diaphoresis, anxiety, headache, and gastrointestinal upset. None of these conditions are associated with an anion gap metabolic acidosis and a reduced arterial-venous oxygen gradient and are therefore unlikely causes of the patient's findings.

Pneumo 116 A 78-year-old woman is evaluated in the intensive care unit for disorientation. The patient recently developed the acute respiratory distress syndrome secondary to community acquired pneumonia, and mechanical ventilation was started 2 days ago. She lives alone and functions well independently. The patient is on a ventilator; she has received small doses of lorazepam over the past 48 hours and appears comfortable. She has recently become disoriented, is not interacting as well with her family as she had before, and has had fluctuations in mental status over the past 24 hours. On physical examination, pulse rate is 92/min, but vital signs are otherwise normal. Neurologic examination shows no focal abnormalities, and cranial nerve examination is normal. She is calm and awake but cannot follow directions to do the "random letter A test" by squeezing the examiner's hand only on hearing the letter "A"; she also cannot organize her thinking to answer simple questions. When asked whether she is seeing things or hearing things that are not there, she shakes her head "No." Laboratory studies show hemoglobin of 9.9 g/dL (99 g/L) and a leukocyte count of 11,000/µL (11 × 109/L) with a normal differential. Metabolic panel reveals plasma glucose of 180 mg/dL (10.0 mmol/L); serum total thyroxine and thyroid-stimulating hormone levels are normal. Which of the following is the most likely diagnosis? A Delirium B Dementia C Psychosis D Stroke

A The cardinal features of delirium are (1) acute onset or fluctuations in mental status over a 24-hour period, (2) inattention, (3) disorganization of thinking, and (4) an altered level of consciousness at the time of the evaluation. This patient has the most common manifestation of delirium in the intensive care unit (ICU), which is hypoactive or "quiet" delirium. Delirium is a form of acute brain dysfunction that occurs in 50% to 80% of ventilated patients in the ICU. It is associated with a threefold higher rate of death by 6 months, much longer lengths of ICU and hospital stay, higher costs, and a 10-fold higher rate of chronic cognitive deficits after survival. It can be diagnosed quickly using the Confusion Assessment Method for the Intensive Care Unit (CAM-ICU), which takes less than a minute in most patients. The four cardinal features of the diagnosis are (1) acute onset or fluctuations in mental status over a 24-hour period, (2) inattention, (3) disorganization of thinking, and (4) an altered level of consciousness at the time of the evaluation. Patients are defined as delirious if they are positive for features 1 and 2 and either 3 or 4. This patient has features 1, 2, and 3: she is having fluctuations in her mental status as evidenced by the Richmond Agitation Sedation Scale, is inattentive as evidenced by her inability to do the random letter A test, and cannot correctly answer simple questions that require organization of her thinking. Hallucinations may be a symptom of delirium, but they are not required for the diagnosis. She is not hyperactive or in "distress," which is also not required for the diagnosis of delirium. The patient has no signs of acute focal neurologic findings, which are characteristic of stroke. Dementia is an acquired chronic impairment of memory and other aspects of intellect that impedes daily functioning and is not compatible with this patient's acute fluctuating mental status. Psychosis is a disturbance in the perception of reality, evidenced by hallucinations, delusions, or thought disorganization. The patient denies hearing or seeing things that are not there, making psychosis unlikely.

Pneumo 40 A 70-year-old man is evaluated for the insidious onset of dyspnea. He does not have chest pain, cough, hemoptysis, or fever. The patient has advanced adenocarcinoma of the lung, and 1 month ago a left-sided malignant pleural effusion was diagnosed. The effusion was small, and observation was recommended. On physical examination, the patient is thin and shows minimal increased work of breathing at rest on ambient air. The temperature is 36.9 °C (98.5 °F), the blood pressure is 140/75 mm Hg, the pulse rate is 96/min and regular, and the respiration rate is 22/min; oxygen saturation is 91% on ambient air. There is dullness to percussion in the left base with decreased fremitus and egophony above the level of dullness. There is digital clubbing. Chest radiograph shows left hilar and mediastinal lymphadenopathy and a moderate-sized pleural effusion occupying 50% of the hemithorax with minimal contralateral mediastinal shift. Thoracentesis was terminated because of the patient's coughing and severe anterior chest pain. The lung did not expand after thoracentesis. Analysis of the pleural fluid shows: Erythrocyte count 10,000/µL (10 × 109/L) Nucleated cell count 2800/µL (2.8 × 109/L) with 7% neutrophils, 61% lymphocytes, 15% macrophages, 10% mesothelial cells, and 7% eosinophils Total protein 3.8 g/dL (38 g/L) Lactate dehydrogenase 250 U/L pH 7.18 Glucose 38 mg/dL (2.1 mmol/L) Pleural fluid cytology is positive for adenocarcinoma. Which of the following is the most appropriate management for this patient? A Placement of an indwelling pleural catheter B Radiation therapy C Talc pleurodesis through a small-bore catheter D Video-assisted thoracoscopic surgery with pleural abrasion

A The development of severe, anterior chest pain during therapeutic thoracentesis is virtually diagnostic of an unexpandable lung with the development of significant negative intrapleural pressure. The development of severe, anterior chest pain during therapeutic thoracentesis is virtually diagnostic of an unexpandable lung with the development of significant negative intrapleural pressure. The anterior chest pain is quickly relieved by allowing air entry into the pleural space through the thoracentesis needle or catheter. In this situation, the patient is best managed with an indwelling catheter. The patient and his family are instructed to drain the pleural fluid when breathlessness ensues and to discontinue drainage immediately when anterior chest pain develops. This patient has two distinct pathophysiologic causes of his pleural effusion: (1) fluid produced by the malignant involvement of the pleura primarily due to increased levels of vascular endothelial growth factor and (2) an unexpandable lung due to tumor involvement of the visceral pleura. Therefore, an indwelling catheter removes the "malignant fluid," and when this volume of fluid has been removed, further fluid removal (due to the unexpandable lung) results in a significant decrease in pleural pressure causing anterior chest pain. Talc pleurodesis will not be completely effective because all of the lung cannot expand to the chest wall to promote pleurodesis. A surgical procedure in a patient with advanced malignancy would not be the appropriate initial treatment, and furthermore, pleural abrasion would not promote effective pleurodesis with an unexpandable lung, which would require a decortication that would not be appropriate for this patient. Outpatient thoracentesis could be appropriate; however, it requires frequent trips to the hospital or clinic, which might be problematic for some patients. Radiation therapy is not indicated for lung entrapment from adenocarcinoma.

Pneumo 24 A 40-year-old man is evaluated for shortness of breath and left-sided chest discomfort without cough, fever, or hemoptysis. He had a contusion to the left side of his chest and back 1 week ago in an automobile accident. Chest radiograph immediately after the accident showed no fracture of the spine or ribs, but he had severe contusions on his back and on the left side of the chest. The patient has a history of lymphoma. Examination of the chest shows dullness to percussion and decreased breath sounds on the left side. Chest radiograph shows a moderate-sized, left-sided pleural effusion without a pneumothorax. Serum protein is 5.8 g/dL (58 g/L), cholesterol 200 mg/dL (5.2 mmol/L), and triglycerides 100 mg/dL (1.13 mmol/L). Thoracentesis yields 500 mL of pleural fluid, and analysis shows: Cell count Erythrocytes 300/µL; leukocytes 890/µL (0.89 × 109/L) with 65% lymphocytes, 22% neutrophils, 8% mesothelial cells, and 4% eosinophils Total protein 3.5 g/dL (35 g/L) Lactate dehydrogenase 250 U/L pH 7.50 Amylase 25 U/L Triglycerides 145 mg/dL (1.6 mmol/L) Cholesterol 38 mg/dL (0.98 mmol/L) Cytology, Gram stain, acid-fast bacilli stain, and bacterial culture are negative. Which of the following is the most likely diagnosis? A Chylothorax B Lymphomatous pleural effusion C Parapneumonic effusion D Tuberculous pleural effusion

A The most common causes of chylothorax are cancer and trauma; other causes are pulmonary tuberculosis, chronic mediastinal infections, sarcoidosis, lymphangioleiomyomatosis, and radiation fibrosis. Chylothorax is drainage of lymphatic fluid into the pleural space secondary to disruption or blockage of the thoracic duct or one of its lymphatic tributaries. Malignancy is the most common cause of chylothorax, but trauma is the second most common cause. Chylothorax can present about 2 to 10 days after penetrating or non-penetrating trauma to the chest. The pleural fluid in chylothorax is usually milky but may also be serous or serosanguineous in malnourished patients with little fat intake. The pleural fluid triglyceride concentration in a chylothorax is typically greater than 110 mg/dL (1.24 mmol/L) and occurs in association with a low pleural fluid cholesterol concentration. If the triglyceride level is less than 50 mg/dL (0.56 mmol/L), chylothorax is unlikely. When the pleural fluid triglyceride concentration is between 50 and 110 mg/dL (0.56 and 1.24 mmol/L), a lipoprotein analysis should be done and the presence of chylomicrons would confirm the diagnosis in such cases. Chylothorax can also occur in association with pulmonary tuberculosis and chronic mediastinal infections, sarcoidosis, lymphangioleiomyomatosis, and radiation fibrosis. A lymphomatous pleural effusion is always a consideration in patients with a history of lymphoma; however, a lymphomatous pleural effusion typically has an elevated lactate dehydrogenase level (often greater than 1000 U/L). Parapneumonic effusion is usually associated with a neutrophilic pleocytosis. Patients with tuberculous pleural effusion usually present with a nonproductive cough, chest pain, and fever. Chest radiograph usually shows a small to moderate effusion.

PNeumo 72 A 63-year-old woman is evaluated for a 3-week history of gradually increasing dyspnea and a mild productive cough with intermittent fever and severe fatigue. The patient was diagnosed with chronic myeloid leukemia 2 months ago; she has no previous history of lung disease. Her medications include imatinib mesylate and a multivitamin. On physical examination, the temperature is 37.8 °C (100.0 °F), the blood pressure is 110/78 mm Hg, the pulse rate is 96/min, and the respiration rate is 22/min. Oxygen saturation is 92% with the patient breathing ambient air. Laboratory studies show mild anemia with leukocytosis and eosinophilia. Chest radiograph shows new interstitial and alveolar infiltrates. Chest CT scan shows a pattern of ground-glass attenuation and irregular reticular lines distributed in the middle and upper lung zones bilaterally. Bronchoscopy with bronchoalveolar lavage performed 5 days ago showed no evidence for bacterial, fungal, or viral pathogens on smear and culture. Transbronchial lung biopsy performed during bronchoscopy revealed a pathologic diagnosis of organizing pneumonia. Which of the following is the most appropriate management for this patient? A Discontinue imatinib mesylate B Start broad-spectrum antibiotic C Start ribavirin D Start intravenous heparin

A The most important management step in treatment of drug-induced lung disease is immediate withdrawal of the offending agent. This patient has drug-induced lung disease caused by imatinib mesylate. The most important management step in treatment of drug-induced lung disease is immediate withdrawal of the offending agent. Diagnosis of drug-induced lung disease requires a high index of suspicion. This patient developed new-onset respiratory symptoms and radiologic changes (mixed alveolar and interstitial pattern) that were temporally associated with initiation of imatinib mesylate to treat chronic myeloid leukemia. Exclusion of underlying infectious causes by bronchoalveolar lavage in this patient is an important diagnostic step. The findings of peripheral blood eosinophilia and organizing pneumonia on bronchoscopic lung biopsy are nonspecific but support drug-induced lung injury caused by imatinib mesylate. This patient has no evidence for respiratory infection on bronchoalveolar lavage, and antibiotic or antiviral therapy is not needed. Deep venous thrombosis and pulmonary embolism are always a possibility in patients with underlying malignancy. However, this patient's CT chest findings and biopsy results are most consistent with drug-induced lung disease, and initiation of heparin anticoagulation would not be appropriate.

Pneumo 17 A 28-year-old man is evaluated in the emergency department for a 2-day history of worsening dyspnea and wheezing in conjunction with an upper respiratory tract infection. The patient has a history of asthma, and his medications are inhaled mometasone and albuterol. In the emergency department, the patient is anxious and is using accessory muscles to breathe; he cannot speak in full sentences. The oxygen saturation is 90% while he is breathing ambient air. Breath sounds are reduced bilaterally, with faint diffuse expiratory wheezes. He is given albuterol by nebulizer, and use of accessory muscles is reduced. Bedside spirometry shows an FEV1 of 35% of predicted; he is given two more treatments of nebulized albuterol. After treatment, the patient is alert with slight use of accessory muscles; he can speak in short full sentences. Vital signs are stable; oxygen saturation is 98% with the patient receiving oxygen, 2 L/min. Breath sounds are louder than on initial examination, and wheezing is more intense. Spirometry shows an FEV1 of 50% of predicted. Which of the following is the most appropriate next step in the management of this patient? A Admit the patient to a regular medicine ward B Discharge the patient on his baseline asthma treatment regimen C Intubate and admit the patient to the intensive care unit D Monitor the patient in the intensive care unit

A The response to inhaled bronchodilators is more predictive of the clinical course in a patient with asthma than initial physical examination and findings. This patient presented with signs of a severe asthma exacerbation. Decreased breath sounds, accessory muscle use, sternocleidomastoid or suprasternal retractions, inability to speak in full sentences, and paradoxical pulse greater than 15 mm Hg are associated with severe airflow obstruction, although the absence of these findings does not necessarily exclude the presence of a high-risk exacerbation. However, the initial physical examination and findings are less predictive of the clinical course in a patient with asthma than the response to bronchodilators. This patient has responded well to bronchodilators, with improved ability to speak and reduced accessory muscle use. Wheezing may become more prominent in the early stages of recovery owing to improved airflow through narrowed airways. According to the newest National Asthma Education and Prevention Program's guidelines, admission to the intensive care unit is recommended for symptomatic patients with even mild carbon dioxide retention (PCO2 greater than 42 mm Hg) or severely decreased lung function despite aggressive bronchodilator treatment (persistent FEV1 or peak expiratory flow less than 40% of predicted). This patient does not meet the criteria for admission to the intensive care unit or intubation and mechanical ventilation at this time. The best disposition for this patient would be admission to the hospital ward; his FEV1 has not improved enough to warrant discharge.

Pneumo 71 A 70-year-old man with a 7-year history of chronic obstructive pulmonary disease is evaluated for increased fatigue. The patient has daily cough and has had two exacerbations in the past 8 months. He has smoked a pack of cigarettes a day for the past 55 years and has recently stopped smoking. His medications are albuterol as needed, salmeterol, and tiotropium. On physical examination, vital signs are normal. Breath sounds are decreased, and heart sounds are distant. Oxygen saturation is 93% with the patient breathing ambient air. Spirometry shows an FEV1 of 39% of predicted and an FEV1/FVC ratio of 60%. Chest radiograph shows hyperinflation. The patient is referred for pulmonary rehabilitation. Which of the following additional therapies would be indicated for this patient? A Inhaled corticosteroids B Montelukast C Oral corticosteroids D Theophylline

A Treatment of GOLD stage III chronic obstructive pulmonary disease includes short-acting bronchodilators as needed and regular treatment with one or more long-acting bronchodilators with an inhaled corticosteroid and pulmonary rehabilitation. This patient has severe (GOLD stage III) chronic obstructive pulmonary disease (COPD) defined as FEV1/FVC <70% and FEV1 30% to 50% of predicted with or without chronic symptoms (cough, sputum production). Treatment for this patient includes support for his smoking cessation, short-acting bronchodilators as needed, and regular treatment with one or more long-acting bronchodilators, as well as an inhaled corticosteroid, along with pulmonary rehabilitation. Oral corticosteroids are not recommended for regular use in a long-term maintenance program because there has been no consistent evidence of efficacy or superiority compared with other agents that have fewer adverse effects. Low-dose theophylline reduces exacerbation in patients with COPD but does not increase post-bronchodilator lung function significantly. High-dose theophylline is an effective bronchodilator but due to its potential toxicity inhaled bronchodilators are again preferred. Leukotriene modifiers, such as montelukast, are not recommended in COPD. Many studies have evaluated leukotriene modifiers in COPD, but the improvement in lung function and symptoms with these agents is modest at best. Two recent long-term studies of more than 11,000 patients have evaluated treatment for patients with GOLD stages II to IV. The TORCH study and the UPLIFT study showed that in patients with COPD, combination therapy with bronchodilators and an inhaled corticosteroid improved quality of life and pulmonary function without significant adverse effects. Patients enrolled in pulmonary rehabilitation programs benefit with respect to both exercise tolerance and symptoms of dyspnea and fatigue. The minimum length of an effective rehabilitation program is 2 months, but the longer the program continues, the more effective the results are.

Pneumo 11 A 71-year-old woman is evaluated for a 3-week history of mild pain in the shoulders and thighs and weakness when rising from a seated position and getting out of bed. She also has a new rash on her hands. Eight months ago she was evaluated for dyspnea and new interstitial infiltrates that resulted in a lung biopsy and a diagnosis of idiopathic nonspecific interstitial pneumonia. She was treated with prednisone, 60 mg/d, for 1 month; the dose was then tapered to 10 mg/d. Her symptoms had been stable on that dose until her new complaints. On physical examination, there are swelling and discoloration of the eyelids and an erythematous scaly rash over the extensor surfaces of interphalangeal joints of both hands. There is symmetric weakness of the proximal hip flexors and shoulder girdle muscles; hand strength is normal. Laboratory studies show antinuclear antibodies positive at a titer of 1:1280 (previously negative), serum creatine kinase 1270 U/L, and erythrocyte sedimentation rate 60 mm/h; serum electrolytes and complete blood count are normal. Chest radiograph shows bilateral reticular and alveolar abnormalities in the lower- and mid-lung zones. Which of the following is the most appropriate management for this patient? A Electromyography and muscle biopsy B Repeat lung biopsy C Skin biopsy D Taper prednisone dosage

A Up to 30% of patients with dermatomyositis and polymyositis present with single-organ involvement of the lungs indistinguishable from idiopathic interstitial lung disease. This patient's symmetric proximal muscle weakness and pain with an erythematous, scaly rash over the interphalangeal joints (Gottron sign), coupled with a positive antinuclear antibody titer and elevated creatine kinase level, suggest dermatomyositis. Electromyography and muscle biopsy will establish the diagnosis of inflammatory myopathy. Inflammatory myopathy must be distinguished from corticosteroid-induced myopathy because treatment of dermatomyositis requires increased corticosteroids, whereas corticosteroid-induced myopathy is treated with withdrawal of prednisone. Corticosteroid-induced myopathy is not associated with elevated antinuclear antibodies, creatine kinase, or erythrocyte sedimentation rate. Up to 30% of patients with dermatomyositis and polymyositis present without muscle, skin, or joint involvement and have single-organ involvement of the lungs indistinguishable from idiopathic interstitial lung disease. Interstitial lung disease associated with inflammatory myopathy often occurs in the context of antisynthetase antibodies (for example, anti-Jo-1) and the antisynthetase syndrome (acute onset, constitutional symptoms, Raynaud phenomenon, "mechanic's hands," arthritis, and interstitial lung disease). Repeat lung biopsy is unlikely to yield new diagnostic information in this patient with stable chest radiograph and no new respiratory symptoms. A skin biopsy is likely to reveal nonspecific findings that will not help diagnose the muscle symptoms.

Pneumo 61 A 30-year-old woman develops bilateral pulmonary infiltrates and hypoxemia 48 hours after undergoing repair of multiple long-bone fractures. Her initial arterial blood gases are pH 7.48, PCO2 30 mm Hg, and PO2 45 mm Hg on 100% oxygen by nonrebreather mask. The patient is intubated and placed on mechanical ventilation. The temperature is 38.3 °C (101.0 °F), the pulse rate is 100/min, the respiration rate is 28/min, and the blood pressure is 120/60 mm Hg. The patient weighs 60 kg (132.3 lb). Oxygen saturation is 83%. She is sedated, hemodynamically normal, and not using accessory muscles to breathe. There are bilateral inspiratory crackles. The ventilator settings are as follows: volume control mode, respiration rate 26/min, tidal volume 360 mL, positive end-expiratory pressure (PEEP) 5 cm H2O, FiO2 0.8. Arterial blood gases are pH 7.45, PCO2 33 mm Hg, and PO2 50 mm Hg. Which of the following is the most appropriate next step in the management of this patient? A Increase FiO2 to 1.0 B Increase PEEP to 10 cm H2O C Increase respiration rate to 32/min D Increase tidal volume to 700 mL E Start vecuronium infusion

B Increasing positive end-expiratory pressure is first-line therapy in acute respiratory distress syndrome patients with severe hypoxemia. The patient has severe hypoxemic respiratory failure from the acute respiratory distress syndrome (ARDS). The primary cause of hypoxemia is shunt, which will not correct with breathing 100% oxygen. Increasing positive end-expiratory pressure (PEEP) improves gas exchange by recruiting flooded and collapsed alveoli. Recruitment may also reduce subsequent ventilator-induced lung injury. However, excess PEEP can cause lung injury by overdistending alveoli and reducing cardiac output. The National Institutes of Health ARDS Network ALVEOLI study randomized patients to high levels of PEEP (starting at 12 cm H2O) versus low levels (starting at 5 cm H2O). Despite greater acute improvements in oxygenation with higher levels of PEEP, there was no difference in the mortality rate between the groups. In another trial, the high-PEEP group, in which the PEEP was uniformly titrated up to plateau pressure of 28 to 30 cm H2O, spent fewer days on the ventilator than the low-PEEP group but overall mortality was not decreased. In this patient, oxygenation is inadequate, and it would be appropriate to increase PEEP. Increasing the respiration rate is inappropriate because increasing ventilation does not correct hypoxemia resulting from shunt. Increasing the tidal volume is not as effective as increasing PEEP, and the use of higher tidal volumes is associated with increased mortality in patients with acute lung injury. Use of paralytic agents is generally reserved for ARDS patients with hypoxemia refractory to maximal ventilator support or temporarily in patients with marked ventilator dyssynchrony. Paralytic agents can improve oxygenation by reducing oxygen consumption but pose a risk of intensive care unit-acquired weakness and require aggressive sedation.

Pneumo 51 A 22-year-old woman is evaluated in the emergency department for an acute exacerbation of asthma. The patient has a history of multiple hospitalizations for asthma, including one episode requiring invasive mechanical ventilation. Her medications are prednisone, 10 mg/d; inhaled salmeterol; montelukast; and nebulized albuterol every 4 hours as needed. She has used four doses of albuterol in the past 24 hours. On physical examination, she is alert, diaphoretic, using accessory muscles to breathe, and speaking in short sentences. The respiration rate is 32/min, and oxygen saturation on ambient air is 89%. Breath sounds are decreased bilaterally with a few end-expiratory wheezes. Arterial blood gases are pH 7.48, PCO2 30 mm Hg, and PO2 59 mm Hg. Chest radiograph shows hyperinflation but is otherwise normal. Therapy with supplemental oxygen and continuous nebulized albuterol is started. One hour after her presentation, the patient's examination findings and symptoms are unchanged. Repeat arterial blood gases are pH 7.39, PCO2 40 mm Hg, and PO2 95 mm Hg with the patient on an unspecified amount of supplemental oxygen. Which of the following is the most appropriate next step in the management of this patient? A Administer lorazepam B Admit the patient to the intensive care unit C Admit the patient to a regular medicine ward D Discharge the patient on her current medications E Perform laryngoscopy

B A normal, slightly elevated, or rising Pco2 may indicate impending respiratory arrest in patients with an asthma exacerbation. Blood gas results must be correlated with a patient's clinical course. This patient's initial arterial blood gas results are consistent with acute respiratory alkalosis, which is typical in patients with an exacerbation of asthma. The return of Pco2 to normal on the repeat blood gas measurement may reflect rapid improvement after effective bronchodilator therapy but could also indicate impending respiratory failure. The patient's lack of symptomatic and objective improvement indicates the latter is the case, and she is at risk of rapid progression to severe respiratory acidosis. At this juncture, the patient is a suitable candidate for intubation and should be managed in the intensive care unit. Patients with vocal cord dysfunction can present with wheezing and be misdiagnosed with an asthma exacerbation. However, vocal cord dysfunction is unlikely in this patient given her history of prior intubation for an asthma exacerbation, and direct laryngoscopy, which is used to diagnose vocal cord dysfunction, could precipitate respiratory arrest in the patient. Benzodiazepines and other sedating medications should be used cautiously during asthma exacerbations because of the risk of precipitating respiratory arrest.

Pneumo 38 A 20-year-old man with a history of severe asthma is brought to the emergency department obtunded and in severe respiratory acidosis from an acute exacerbation of his asthma. He is intubated in the emergency department, and 30 minutes later he is evaluated in the intensive care unit. On physical examination, the patient is sedated and unresponsive; he is afebrile, and the blood pressure is 80/40 mm Hg, pulse rate is 140/min, and respiration rate is 24/min. Oxygen saturation is 98%. There are diffusely decreased breath sounds with faint bilateral expiratory wheezes. Cardiac examination reveals distant regular tachycardic rhythm but is otherwise normal. Ventilator settings include volume control mode with a set rate of 24, tidal volume 800 mL (12 mL/kg ideal body weight), positive end-expiratory pressure 5 cm H2O, and FiO2 0.6. Arterial blood gases are pH 7.30, PCO2 40 mm Hg, and PO2 180 mm Hg. Chest radiograph shows hyperinflation and a properly placed endotracheal tube. Which of the following would most rapidly improve the patient's hypotension? A Administer thrombolytic therapy B Disconnect the patient from the ventilator for 30 seconds C Perform pericardiocentesis D Place chest tubes bilaterally

B Auto-positive end-expiratory pressure is relieved by allowing sufficient time for full expiration and aggressive treatment of airflow obstruction. The patient's hypotension is the result of severe auto-positive end-expiratory pressure (PEEP). Patients with severe airflow obstruction are particularly vulnerable to developing auto-PEEP when receiving mechanical ventilation or bag-mask ventilation. This patient's high set respiration rate and tidal volume do not allow sufficient time for complete expiration before the next breath is delivered. The resulting elevation in intrathoracic pressure impairs venous return to the right heart and increases pulmonary vascular resistance, thereby reducing cardiac output and producing hypotension. Briefly disconnecting the patient from the ventilator allows for complete expiration, reduction in auto-PEEP, and recovery of cardiac output. The use of low tidal volumes and rates may result in acute respiratory acidosis, but this poses a lower risk to the patient than sustained severe auto-PEEP. This ventilator strategy is known as permissive hypercapnia. The elevated airway pressures associated with auto-PEEP can also cause tension pneumothorax. However, the presence of bilateral breath sounds and absence of pneumothorax on chest radiograph effectively rule out tension pneumothorax. Therefore, inserting chest tubes would not be appropriate. Cardiac tamponade and massive pulmonary embolism would be much less likely than severe auto-PEEP in a patient with severe airflow obstruction receiving mechanical ventilation with these ventilator settings. Therefore, thrombolysis and pericardiocentesis are not appropriate in this patient.

Pneumo 15 A 28-year-old man is evaluated for a 9-month history of daily cough productive of yellow sputum and intermittent low-grade fever. He has had three episodes of pneumonia during that time; the symptoms improve with antibiotic therapy but return when therapy is discontinued. The patient does not have a history of aspiration, asthma, or sinusitis, and he takes no medications. He has never smoked. On physical examination, the temperature is 37.4 °C (99.3 °F), the pulse rate is 88/min, the respiration rate is 18/min, the blood pressure is 116/58 mm Hg, and the BMI is 24. Breath sounds are reduced in the right base; the lungs are otherwise clear. Laboratory tests are normal. Two chest radiographs 3 months apart have shown an infiltrate in the right lower lobe. Contrast-enhanced CT scan of the chest shows right lower lobe bronchiectasis and partial volume loss of this lobe; endobronchial obstruction is suggested. There is no lymphadenopathy. Which of the following is the most likely diagnosis? A Adenocarcinoma B Carcinoid tumor C Small cell carcinoma D Squamous cell carcinoma

B Carcinoid tumors are neuroendocrine tumors with an indolent growth pattern that often present with endobronchial obstruction. A carcinoid tumor is the most likely tumor in a young person who has never smoked and who has evidence of endobronchial obstruction. Bronchial carcinoid is a slow growing tumor that originally was classified as an adenoma but has been reclassified as a malignant neoplasm because of its ability to metastasize. Most bronchial carcinoid tumors are located in proximal airways and cause symptoms by either obstructing an airway or bleeding. Common presenting symptoms include cough or wheeze, hemoptysis, and recurrent pneumonia in the same pulmonary lobe. The carcinoid syndrome is caused by systemic release of vasoactive substances such as serotonin, and the most typical features include cutaneous flushing and diarrhea. Bronchial carcinoids are not commonly associated with the carcinoid syndrome because of their relatively small amount of serotonin production. Although adenocarcinoma is the most common cancer cell type in a never-smoker, it rarely causes endobronchial obstruction. Small cell and squamous cell cancers frequently cause bronchial obstruction but rarely, if ever, occur in a young, never-smoker. Furthermore, the development of bronchiectasis denotes a chronic process of airway obstruction and infection. It occurs most commonly in association with foreign body aspiration, endobronchial obstruction by an indolent tumor such as carcinoid tumor, or secondary to extraluminal compression. Carcinomas, including those causing endobronchial obstruction, tend to grow very quickly, resulting in patient treatment (or death) before they can cause focal bronchiectasis. Treatment of carcinoid tumor is surgical resection when possible, and, for typical carcinoids, 5-year survival is about 90%.

Pneumo 105 A 42-year-old man is evaluated in the surgical intensive care unit after surgery for a bowel obstruction. He presented yesterday to the emergency department for a 2-day history of fever, confusion, and abdominal pain. On examination, the patient's temperature was 38.4 °C (101.1 °F). Cardiac examination was normal; examination of the lungs revealed diffuse bilateral crackles. The abdomen was diffusely and markedly tender, with rebound and guarding. Leukocyte count was 18,400/µL (18.4 × 109/L) with 80% segmented neutrophils and 6% band forms. In the emergency department, his plasma glucose concentration was 205 mg/dL (11.4 mmol/L); the patient has no history of diabetes mellitus. Chest radiograph showed pulmonary edema, and the patient was hospitalized; imipenem/cilastatin was started, and he was taken to surgery. The patient was found to have a bowel obstruction with perforation and contamination of the peritoneal cavity. After surgery, the plasma glucose concentration is 300 mg/dL (16.7 mmol/L). Which of the following is the most appropriate management of the patient's hyperglycemia? A Any insulin regimen that follows a sliding scale B Intravenous insulin drip C Subcutaneous intermediate-acting basal insulin D Subcutaneous long-acting basal insulin

B Continuous intravenous insulin is the most effective method for adequate glycemic control in critically ill patients. Glucose control in critically ill patients is now practiced widely. Hyperglycemia is believed to contribute to various physiologic derangements, such as inflammation and coagulopathy, that should be controlled in the septic patient. The exact range and goal for target glucose levels have been controversial, with previous studies suggesting that intensive glucose control (a target level of 80 to 110 mg/dL [4.4 to 6.1 mmol/L]) reduced mortality. However, in a recent large, international, randomized trial (the NICE-SUGAR study), intensive glucose control increased mortality among adults in the intensive care unit; a conventional blood glucose target of less than or equal to 180 mg/dL (10.0 mmol/L) resulted in lower mortality than did a target of 81 to 108 mg/dL (4.5 to 6.0 mmol/L). Continuous intravenous insulin is the most effective method for adequate glycemic control in these patients. Subcutaneous regular insulin based on a sliding scale and intermediate-acting or long-acting basal insulin do not offer the best acute titration of glucose in intensive care unit patients who may have volatile glucose levels. Furthermore, current guidelines emphasize the need to reconsider the widespread use of regular insulin sliding scales, which often result in labile glucose control, as the sole antihyperglycemic therapy in hospitalized patients. Instead, more active, physiologic insulin regimens are advisable.

Pneumo 92 A 68-year-old man with a history of severe chronic obstructive pulmonary disease and baseline arterial blood gases of pH 7.35, PCO2 60 mm Hg, and PO2 60 mm Hg breathing ambient air was admitted to the intensive care unit and intubated for an exacerbation of his respiratory disease. His condition improved over the next 72 hours on therapy with bronchodilators, systemic corticosteroids, and levofloxacin. He is now afebrile and hemodynamically stable with a respiration rate of 15/min and an oxygen saturation of 95%. He is alert, oriented, and moving all extremities. He has a strong cough and minimal secretions. Breath sounds are clear aside from faint end-expiratory wheezes. Heart sounds are normal. Arterial blood gases are pH 7.42, PCO2 35 mm Hg, and PO2 68 mm Hg on the following ventilator settings: volume control mode, tidal volume 600 mL, set rate 15/min, positive end-expiratory pressure 5 cm H2O, and FiO2 0.35. He is placed on a spontaneous breathing trial but develops marked respiratory distress with a blood pressure of 180/100 mm Hg, pulse rate of 130/min, respiration rate of 28/min, oxygen saturation of 90%, and diaphoresis. He rapidly returns to baseline with resumption of his prior volume control settings. Which of the following is the most likely explanation for the patient's difficulty weaning from the ventilator? A Anxiety B Excessive ventilation on volume control mode C Inadequate oxygenation D Intensive care unit-acquired weakness

B Excessive ventilation can cause difficulty weaning in patients who otherwise require only modest ventilator support. Ventilatory support in patients with chronic ventilatory failure should be targeted to match the baseline PCO2. At baseline, the patient has a chronic respiratory acidosis, but his PCO2 normalized while he was receiving mechanical ventilation. Subsequent reversal of his renal compensation prevented a sustained period of metabolic alkalosis. However, he cannot sustain a PCO2 of 40 mm Hg during trials of spontaneous breathing, resulting in acute respiratory acidosis and difficulty with ventilator weaning. The PCO2 is inversely proportional to alveolar ventilation. Reducing the ventilator rate from 15/min to 10/min will result in a roughly proportional increase in PCO2 to his baseline of 60 mm Hg. The patient otherwise is a good candidate for weaning in that he is requiring only a modest amount of ventilator support and has minimal secretions. The patient's oxygen desaturation during his weaning trial is mild, and the primary problem is respiratory acidosis. Anxiety and agitation can complicate ventilator weaning in good candidates. However, this typically manifests when sedation is reduced. This patient is alert, oriented, and doing well while receiving full ventilator support, which makes it unlikely that an increase in sedation will expedite weaning. The patient's critical illness and exposure to systemic corticosteroids place him at risk of intensive care unit (ICU)-acquired weakness. However, he has a strong cough, is moving all extremities, and has been critically ill for only 3 days. ICU-acquired weakness develops in approximately 25% of ICU patients receiving mechanical ventilation for at least 7 days.

Pneumo 75 An 18-year-old high school soccer player is evaluated for recurrent episodes of dyspnea, chest tightness, and cough that occur during games and that prevent her from continuing playing. The episodes are more common in cold weather, and they resolve spontaneously over 20 to 30 minutes. The patient's father has allergies but no known lung disease. The patient is otherwise healthy and takes no medications. On physical examination, vital signs are normal and the lungs are clear. Spirometry reveals an FEV1 of 90% predicted and an FEV1/FVC ratio of 80%. Chest radiograph is normal. Which of the following is the most appropriate next step in the evaluation of this patient? A Allergy skin testing B Exercise challenge test C Measurement of lung volumes and diffusing capacity for carbon monoxide D Measurement of resting arterial blood gases

B Exercise-induced asthma is confirmed by exercise challenge testing (to greater than 85% of maximal predicted heart rate) with postexercise spirometry showing a 15% or greater fall in FEV1. Exercise-induced asthma (EIA) occurs in nearly 90% of patients with asthma who exercise at sufficient intensity and is more common with exercise in cold, dry air. Exercise challenge testing (to greater than 85% of maximal predicted heart rate) with postexercise spirometry confirms the diagnosis. An exercise challenge test has a high specificity for the diagnosis when the fall in FEV1 after exercise is 15% or greater. Airway obstruction after exercise peaks in 5 to 15 minutes and resolves in 20 to 30 minutes. Prophylaxis with short-acting inhaled β-agonists, such as albuterol, 5 to 10 minutes before exercise, prevents EIA in more than 80% of patients. With appropriate management, patients with EIA need not limit their involvement in sports. Measurement of lung volumes and diffusing capacity for carbon monoxide is helpful in evaluating patients with suspected parenchymal lung diseases but is not likely to help in the evaluation of this patient given the symptoms and normal chest radiograph. Allergy skin test could be done in this patient at some point, and if positive, would support the diagnosis of asthma because most patients with asthma have allergies. But many patients with allergies do not have asthma; therefore, skin testing alone will not provide an explanation for the patient's symptoms. Checking resting arterial blood gases would have a very low yield in this young and otherwise healthy patient with a normal chest radiograph and spirometry.

Pneumo 53 A 60-year-old woman is evaluated in a scheduled follow-up visit in the office. The patient has a long-standing history of asthma, which has been stable for the past 6 months on therapy with a high-dose inhaled corticosteroid and a long-acting β-agonist. She rarely uses her albuterol inhaler except for prophylaxis of exercise-induced bronchospasm. The patient is pleased with the control of the disease, has not been waking up at night with asthma, and has not missed work due to asthma for almost a year. She has no side effects from her current medical regimen. On physical examination, vital signs are normal; spirometry shows an FEV1 of 3.60 L (85% of predicted) and FEV1/FVC of 75%. Which of the following is the most appropriate management for this patient? A Continue the inhaled corticosteroid and long-acting β-agonist at current doses B Continue the long-acting β-agonist at the current dose and reduce the dose of the inhaled corticosteroid C Discontinue the inhaled corticosteroid and the long-acting β-agonist D Discontinue the inhaled corticosteroid and start a leukotriene modifying agent E Discontinue the long-acting β-agonist and reduce the dose of the inhaled corticosteroid

B In a patient taking high-dose inhaled corticosteroids as part of therapy for persistent asthma whose disease is stable, reducing the dose of corticosteroids should be considered to prevent therapy-related side effects. This patient with persistent asthma is doing well and her disease is well controlled. Therefore, her long-acting β-agonist should be continued at the current dose and the inhaled corticosteroid should be reduced from high-dose to moderate-dose to limit the risk of corticosteroid-related side effects (particularly osteoporosis), which increase significantly after the menopause. Although inhaled corticosteroids are generally safe in asthma, their long-term use, particularly at higher doses, can lead to osteoporosis, glaucoma, cataract, easy bruising, and suppression of the hypothalamic-adrenal axis. For many asthma outcomes, the dose-response curve for inhaled corticosteroids is relatively flat after reaching 400 µg/d of beclomethasone (or equivalent drug). Therefore, when a patient's condition is stable on inhaled corticosteroid therapy, lowering the dose should be considered. Asthma control is better with the combination of a moderate-dose inhaled corticosteroid plus a long-acting β-agonist than with a high-dose inhaled corticosteroid without a long-acting β-agonist. Although the benefits of using a long-acting β-agonist in patients with asthma are well established, concern about increased asthma-related deaths led the U.S. Food and Drug Administration to mandate including a black box warning in the package insert for these drugs to warn patients about this possible risk. The National Asthma Education and Prevention Program's recent recommendations affirmed the practice of adding a long-acting β-agonist to the therapy for patients whose disease is not well controlled on moderate-dose inhaled corticosteroids. Using a leukotriene modifying agent in place of an inhaled corticosteroid is an alternative approach in patients with mild to moderate persistent asthma. Therefore, in patients with persistent asthma, an inhaled corticosteroid is the first-line therapy, and a leukotriene receptor antagonist can be tried in patients who have recurrent corticosteroid-related side effects, who are unwilling to use inhaled corticosteroids regularly, or who have a strong preference for a medication in pill form rather than an inhaler. To minimize long-term side effects, patients should always be treated with the fewest medications at the lowest dose that adequately controls their disease. Therefore, follow-up at 3- to 6-month intervals is recommended to evaluate the patient and make step-wise adjustments to therapy as warranted.

Pneumo 56 A 53-year-old woman is evaluated in the emergency department after 1 day of severe right flank pain, fever, and mild nausea. Urinalysis reveals many leukocytes and leukocyte casts. Gram stain of her urine shows gram-positive cocci, and therapy with vancomycin and gentamicin is started. Despite therapy, the patient develops progressive hypoxemia and requires mechanical ventilation. On examination, she appears very anxious but is not trying to remove the endotracheal tube or any other support devices. The temperature is 37.5 °C (99.5 °F), blood pressure is 120/60 mm Hg, and pulse rate is 130/min. Electrocardiogram shows sinus tachycardia. Blood and urine cultures grow Enterococcus faecalis. While the patient is on mechanical ventilation, which of the following is the most appropriate sedation strategy? A Deep continuous sedation B Intermittent bolus dosing of sedatives with daily awakening C No sedation D Sedation with intermittent paralytic agents as tolerated

B Intermittent bolus dosing of sedatives titrated via a validated sedation scale plus a daily spontaneous awakening trial appears to be the superior method of delivering sedation in critically ill patients. A randomized controlled trial by Girard and colleagues in critically ill patients showed that intermittent bolus dosing of sedatives titrated against a validated sedation scale and with a daily spontaneous awakening trial with total cessation of sedatives yielded a 4-day reduction in intensive care unit (ICU) and hospital length of stay and an improvement in 1-year survival. In critically ill patients, this method allows enough sedation for comfort and reduces the likelihood of excess use of potent psychoactive medications, such as lorazepam. A sedative drip with propofol would also be a consideration, as long as sedation was interrupted daily. The newer-generation sedation scales are being widely adopted by ICUs to aid in drug titration and ease of communication. Therefore, sedation would be beneficial in this patient. The use of sedation with intermittent paralytic agents as tolerated has the potential for increasing the risk of ICU-acquired weakness and paralysis without adequate sedation; this method is generally reserved for patients with severe ventilator dyssynchrony. Deep continuous sedation until the patient is extubated leads to unnecessarily prolonged and pronounced sedation. The notion of trying to "eliminate" memories from the ICU appears to be a risk factor for post-traumatic stress disorder, which occurs in about 15% to 20% of ICU survivors. Neuromuscular blockade with paralytic agents should be avoided unless absolutely necessary because of problems such as myopathy and prolonged paralysis.

Pneumo 96 A 26-year-old woman is evaluated for a 4-month history of progressive dyspnea, initially on exertion only but now also at rest. She was recently evaluated in the intensive care unit for an episode of syncope. Clinical findings were compatible with pulmonary hypertension, which was confirmed with transthoracic echocardiography. Pulmonary function tests and ventilation/perfusion scan were normal. She is sent to the catheterization laboratory. Just before the patient went to the catheterization laboratory, her vital signs were: blood pressure, 95/50 mm Hg; pulse rate, 125/min; and respiration rate, 24/min. A right-heart catheterization shows a low cardiac index, elevated pulmonary artery pressures, normal pulmonary capillary wedge pressure, and elevated pulmonary vascular resistance. Which of the following is the most appropriate management for this patient? A Emergent pulmonary thromboendarterectomy B Intravenous epoprostenol C Intravenous tissue plasminogen activator D Intubation and mechanical ventilation E Oral diltiazem

B Intravenous epoprostenol can have an immediate and substantial beneficial effect in severe pulmonary arterial hypertension, but it is best used in a specialized center. This patient has severe pulmonary arterial hypertension. Intravenous epoprostenol can have an immediate and substantial beneficial effect in such patients. However, because of the complexity and risk associated with epoprostenol therapy, it is best used in a center with special expertise in the evaluation and treatment of pulmonary arterial hypertension. There is no evidence of hemodynamically significant acute thromboembolism in this patient, and thrombolytic medications would not affect the mechanisms causing pulmonary arterial hypertension. By increasing intrathoracic pressure and limiting venous return, mechanical ventilation may in fact worsen the patient's hemodynamic status and should be reserved for when the patient is truly in respiratory failure. Only a minority of patients (approximately 6%) with pulmonary arterial hypertension respond well to calcium channel blockade. In this patient, with preexisting hemodynamic instability, calcium channel blockade should be avoided. The ventilation/perfusion scan that disclosed no regional perfusion defects essentially ruled out chronic thromboembolic pulmonary hypertension, and pulmonary thromboendarterectomy is, therefore, not indicated.

Pneumo 82 A 60-year-old woman is evaluated for progressive shortness of breath and a dry cough. She was diagnosed with idiopathic pulmonary fibrosis 3 years ago but has no other medical problems. On physical examination, she has resting hypoxemia and her other vital signs are normal. Clubbing of the fingers is present. There is no jugular venous distention. The cardiac examination reveals no extra sounds or murmurs. There are late inspiratory crackles on pulmonary examination. The rest of the physical examination is normal. Chest radiograph shows decreased lung volumes and bilateral basal reticular opacities. High-resolution CT scan shows bilateral, basal-predominant, extensive subpleural reticular opacities associated with patchy honeycombing and traction bronchiectasis. Pulmonary function testing shows significantly decreased FVC, total lung capacity, and functional residual capacity; DLCO is 35% of predicted. There has been a decrement in the FVC greater than 10% in the last 6 months. Which of the following is the best treatment option for this patient? A Interferon-γ B Lung transplantation C N-acetylcysteine D Prednisone and azathioprine

B Lung transplantation in eligible patients is the only therapy known to improve survival in patients with idiopathic pulmonary fibrosis. Lung transplantation is the only therapy that improves survival in patients with idiopathic pulmonary fibrosis (IPF), and referral to a transplant center should be considered for all eligible patients with the disorder. Patients referred for transplantation should be free of significant other organ dysfunction, be younger than 65 years, have acceptable nutritional status, and have a satisfactory psychosocial profile and support system. IPF is a relentlessly progressive form of interstitial lung disease, with a median survival from diagnosis of 3 to 4 years. No medical therapy has shown survival benefit in randomized, placebo-controlled clinical trials. Current options to manage patients with IPF include (1) referral of eligible patients for transplantation, (2) referral of interested patients for clinical research trials, (3) observation with supportive care and no medical therapy, and (4) a trial of corticosteroid therapy with or without a cytotoxic agent in an educated patient aware of the risks of unproven therapy. Treatment of patients with immunosuppressive therapy including prednisone and cytotoxic agents, such as azathioprine, has not been shown to improve survival in IPF. A recent, large clinical trial investigating interferon-γ in patients with IPF was stopped early because of lack of benefit and a trend toward worsening outcome in patients treated with interferon-γ. N-acetylcysteine has not been shown to improve the mortality rate in patients with IPF. Addition of N-acetylcysteine to prednisone and azathioprine may modestly improve lung function or protect against the cytotoxic effects of azathioprine. The effect of N-acetylcysteine monotherapy in IPF is not known and a large National Institutes of Health-sponsored clinical trial is currently underway to investigate these issues.

Pneumo 113 A 64-year-old woman who resides in a nursing home and has a history of end-stage kidney disease for which she receives hemodialysis presents with fever, copious sputum production, and hypoxemia. Chest radiograph shows a right-sided infiltrate. She is intubated and started on vancomycin, levofloxacin, and piperacillin-tazobactam for treatment of diffuse right-sided pneumonia. On day 3 in the hospital, she has been afebrile for 48 hours, and her secretions are scant and white. On physical examination, the temperature is 36.9 °C (98.5 °F), pulse rate is 88/min, respiration rate is 14/min, and blood pressure is 140/86 mm Hg; oxygen saturation is 96% on FiO2 0.4. There are diffuse right-sided inspiratory crackles. Leukocyte count is 9600/µL (9.6 × 109/L) (20,400/µL [20.4 × 109/L] on admission). Admission sputum culture is now growing methicillin-resistant Staphylococcus aureus; blood cultures are negative. Chest radiograph shows slight improvement in the right-sided infiltrate compared with admission. Which of the following is the most appropriate antibiotic therapy at this time? A Complete an 8-day course of all three antibiotics B Complete an 8-day course of vancomycin and discontinue the other antibiotics C Complete a 14-day course of all three antibiotics D Complete a 14-day course of vancomycin and discontinue the other antibiotics E Discontinue all antibiotics

B Most causes of health care-associated pneumonia, including ventilator-associated pneumonia, require only approximately 8-day antibiotic courses. The patient has had a good clinical response to treatment for methicillin-resistant Staphylococcus aureus (MRSA) pneumonia and, therefore, does not need antibiotic therapy beyond 7 to 9 days. The patient's residence in a nursing home and chronic hemodialysis place her at risk for infection with resistant organisms, including MRSA and Pseudomonas aeruginosa, and she received appropriate empiric coverage for health care-associated pneumonia. Now that MRSA has been isolated from her sputum, empiric coverage for other organisms should be discontinued. In the absence of bacteremia, it appears treatment for MRSA pneumonia can be safely limited to approximately 8 days. Patients with P. aeruginosa pneumonia appear to have a greater risk of recurrent infection when antibiotic therapy is limited to 8 rather than 15 days.

Pneumo 13 A 75-year-old woman with a history of chronic obstructive pulmonary disease is evaluated in the intensive care unit for delirium. She had a median sternotomy and repair of an aortic dissection and was extubated uneventfully on postoperative day 4. Two days later she developed fluctuations in her mental status and inattention. While still in the intensive care unit, she became agitated, pulling at her lines, attempting to climb out of bed, and asking to leave the hospital. Her arterial blood gas values are normal. The patient has no history of alcohol abuse. The use of frequent orientation cues, calm reassurance, and presence of family members has done little to reduce the patient's agitated behavior. Which of the following is the most appropriate therapy for this patient's delirium? A Diphenhydramine B Haloperidol C Lorazepam D Propofol

B No drug is U.S. Food and Drug Administration-approved for the treatment of delirium, but clinical practice guidelines recommend antipsychotic agents, such as haloperidol. When supportive care is insufficient for prevention or treatment of delirium, symptom control with medication is occasionally necessary to prevent harm or to allow evaluation and treatment in the intensive care unit. The appropriate treatment for this patient is haloperidol. The recommended therapy for delirium is antipsychotic agents, although no drugs are U.S. Food and Drug Administration-approved for this indication. Ongoing randomized, placebo-controlled trials are investigating different management strategies for intensive care unit delirium. A recent systematic evidence review found that the existing limited data indicate no superiority for second-generation antipsychotics compared with haloperidol for delirium. Haloperidol does not cause respiratory suppression, which is one reason that it is often used in patients with hypoventilatory respiratory failure who require sedation. All antipsychotic agents, and especially "typical" agents such as haloperidol, pose a risk of torsades de pointes and extrapyramidal side effects as well as the neuroleptic malignant syndrome. Diphenhydramine and other antihistamines are a major risk factor for delirium, especially in older patients. Lorazepam is actually deliriogenic, and its use in a delirious patient should be carefully re-evaluated, other than perhaps in patients experiencing benzodiazepine withdrawal or delirium tremens. There is no evidence that propofol has any role in treating delirium.

Pneumo 36 A 59-year-old man with chronic obstructive pulmonary disease is evaluated before planned air travel from Denver, Colorado, to Beijing, China. He is physically active but has episodic wheezing and a "racing" heartbeat with exertion. His current therapy consists of inhaled bronchodilators and supplemental continuous oxygen at 2 L/min. On examination, the pulse rate is 68/min, respiration rate is 18/min, and oxygen saturation is 90% with the patient breathing oxygen, 2 L/min. Cardiac examination is normal. Breath sounds are distant without wheezes or crackles. Chest radiograph shows hyperexpanded lungs but no infiltrates or bullae. Which of the following is the most appropriate next step in this patient's management? A Calculate oxygen needs using a prediction algorithm B Order a hypoxia inhalation test C Prescribe in-flight supplemental oxygen level at 3 L/min D Recommend that the patient not fly

B Patients who require continuous oxygen therapy owing to chronic respiratory or cardiovascular disorders may not tolerate commercial air travel and should be assessed for the need for possible additional in-flight supplemental oxygen. Air travel is generally safe for patients with stable chronic respiratory disorders but can pose significant hazards for those who require continuous oxygen therapy for hypoxemia. Cabin air pressure for most commercial airplanes is maintained at about 4500 to 8000 ft above sea level, but the pressure can change significantly from flight to flight and from one airplane model to another. At a cabin pressure equivalent to 8000 ft above sea level, PO2 is expected to fall, potentially leading to hypoxemia and cardiac events in persons who require supplemental oxygen for hypoxemia. Therefore, using the same level of oxygen supplementation used at sea level may be inappropriate for many patients. Additional supplemental oxygen is recommended for patients whose predicted PO2 during flight is less than 50 to 55 mm Hg or oxygen saturation less than 85%. In-flight PO2 can be predicted using published algorithms or determined by performing a hypoxia inhalation test. Prediction algorithms may fail to identify patients who will develop significant hypoxemia during air travel and, unlike the hypoxia inhalation test, cannot predict the development of cardiac events related to low oxygen levels. Simply increasing the level of oxygen supplementation runs the risk of underestimating the required oxygen demands during flight. The hypoxia inhalation test involves placing the patient in either a hypobaric chamber or having the patient breath a 15% oxygen mixture at sea level for at least 15 minutes with continuous pulse oximetry and electrocardiographic monitoring

Pneumo 115 A 50-year-old woman with myasthenia gravis is hospitalized because of worsening weakness and transferred to the intensive care unit for monitoring. Her only medication is pyridostigmine bromide. On physical examination, the patient is alert; the temperature is 36.9 °C (98.4 °F), the blood pressure is 150/90 mm Hg, the pulse rate is 100/min, and the respiration rate is 24/min. Oxygen saturation is 100% with the patient breathing oxygen, 4 L/min by nasal cannula. There is no evidence of bulbar dysfunction. Breath sounds are clear bilaterally but diminished at the bases. Arterial blood gases while the patient was breathing ambient air were pH 7.44, PCO2 35 mm Hg, and PO2 90 mm Hg. Which of the following would be the most appropriate method for monitoring for ventilatory failure in this patient? A Arterial blood gas measurements B Bedside measurement of vital capacity C Continuous monitoring of oxygen saturation D Rapid shallow breathing index

B Patients with neuromuscular weakness and tenuous respiratory status should be monitored with serial measurement of vital capacity. In patients with neuromuscular weakness and tenuous respiratory status, serial measurement of vital capacity is an effective method to anticipate the need for mechanical ventilatory support before the onset of respiratory arrest. Patients with vital capacity under 15 to 20 mL/kg, who are unable to generate more than 30 cm H2O of negative inspiratory force, or with declining values are at high risk of ventilatory failure requiring invasive mechanical ventilation. Patients with bulbar dysfunction may not be able to accurately perform bedside spirometry and are at increased risk of rapid deterioration due to acute aspiration events. Measuring arterial blood gases is inappropriate because carbon dioxide retention is typically a sign of impending respiratory arrest and is therefore not optimal for identifying patients best managed with elective, rather than emergent, intubation. Continuous monitoring of oxygen saturation is inappropriate because the patient could develop life-threatening respiratory acidosis before oxygen desaturation if she were receiving supplemental oxygen. Desaturation is also a relatively late finding suggestive of impending respiratory arrest. The rapid shallow breathing index is used to assess patients during ventilator weaning but not in this clinical context. Furthermore, the index requires measurement of the patient's tidal volume, which is not routinely obtained in patients who are breathing without invasive or noninvasive ventilatory support.

Pneumo 89 A 28-year-old man is evaluated for an abnormal chest radiograph done for chronic intermittent nonproductive cough of 6 months' duration. The radiograph showed bilateral hilar lymphadenopathy and normal lung parenchyma. The patient has fatigue and intermittent mild central chest discomfort when he coughs. He has not had weight loss, fever, night sweats, or recent respiratory illness. Vital signs and cardiopulmonary examination are normal. There is no cervical or axillary lymphadenopathy and no skin findings. Laboratory studies show normal electrolytes and normal complete blood count with differential; serum calcium is 10.8 mg/dL (2.63 mmol/L). Chest CT scan with contrast shows bilateral hilar and mediastinal and subcarinal lymphadenopathy, along with bilateral small lung nodules with a perihilar distribution. Which of the following is the most appropriate next test in the evaluation of this patient? A Bone marrow biopsy B Bronchoscopy with mediastinal lymph node and lung biopsy C Mediastinoscopy D Serum Histoplasma antibody testing

B Patients with suspected sarcoidosis should have tissue biopsy of accessible lesions to confirm the diagnosis. This patient has radiographic features suggesting sarcoidosis, with mediastinal, hilar, and pulmonary parenchymal involvement and mild hypercalcemia. There is no definitive diagnostic test for sarcoidosis. Diagnosis is established by demonstrating histopathologic evidence of noncaseating granulomas in a compatible clinical setting with supportive imaging and exclusion of diseases with similar histopathologic findings. Patients with suspected sarcoidosis should have tissue biopsy performed on accessible lesions to confirm the diagnosis. Fiberoptic bronchoscopy with transbronchial lung biopsy is the initial procedure when suspecting sarcoidosis and has a reported diagnostic yield of 46% to 90%. The addition of transbronchoscopic needle aspiration of mediastinal lymph nodes and bronchoalveolar lavage to transbronchoscopic lymph node biopsy increases the diagnostic sensitivity of bronchoscopy and helps to exclude competing diagnoses. If available, endobronchial ultrasonography-guided transbronchoscopic needle aspiration of abnormal lymph nodes is a low-risk procedure with a reported sensitivity of 85% for sarcoidosis. Bone marrow biopsy occasionally shows noncaseating granulomatous inflammation in patients with systemic sarcoidosis but should not be considered the next diagnostic test in this patient because the diagnostic yield is low in a patient with no hematologic abnormalities. Although fungal disease is a diagnostic consideration, fungal serologies, such as Histoplasma antibody testing, are not specific. Mediastinoscopy remains the gold standard for undiagnosed mediastinal lymphadenopathy, but it has higher morbidity and cost than bronchoscopy with parenchymal and mediastinal biopsies. If bronchoscopy is not diagnostic, mediastinoscopy may be the next appropriate test. Observation and follow-up chest radiograph in 6 months of suspected cases of sarcoidosis is occasionally appropriate. For instance, patients presenting with typical findings of Löfgren syndrome (erythema nodosum with hilar lymphadenopathy and migratory arthralgia) is highly specific for sarcoidosis, has a good prognosis for spontaneous improvement, and may be carefully observed without treatment. However, this patient does not have findings suggesting Löfgren syndrome.

Pneumo 33 A 37-year-old man with asthma is evaluated for frequent episodes of wheezing and dyspnea unrelieved by short-acting β-agonist therapy. He uses his controller medications regularly, including an inhaled long-acting β-agonist and inhaled high-dose corticosteroids. He has symptoms daily and frequent nocturnal symptoms. On physical examination, the patient is in mild respiratory distress. The temperature is 37.0 °C (98.6 °F), blood pressure is 140/85 mm Hg, pulse rate is 90/min, and respiration rate is 18/min. He has bilateral wheezing. Spirometry shows an FEV1 of 65% of predicted. After the supervised use of a bronchodilator in the office, there was some relief of symptoms, and repeat spirometry 10 minutes after the administration of the bronchodilator showed that the FEV1 increased to 85% of predicted. Which of the following is the appropriate next step in this patient's management? A Add a leukotriene modifying drug B Have the patient demonstrate his inhaler technique C Have the patient keep a symptom and treatment log D Start oral prednisone therapy

B Poor inhaler technique is a major reason why patients with asthma do not respond well to specific asthma therapy. The best initial management approach for this patient is to have him demonstrate his inhaler technique. Patient education is a key component in asthma care. Studies have shown that patient education by the physician decreases the number of visits to the emergency department and improves asthma control. Improper technique in the use of inhalers is a major reason that patients do not respond well to medications. A clue suggesting poor inhaler technique includes the patient's rapid improvement in FEV1 after the supervised use of a bronchodilator. Although there used to be one type of inhalation device (the metered-dose inhaler) with one technique that could be taught to the patient, there are now several new and different devices with significant differences in the technique needed for their use. Physicians should learn the proper technique for use of these inhalers before prescribing them to patients in order to ensure proper technique to optimize drug delivery and effectiveness and to reduce side effects. Adding a leukotriene modifying agent would be appropriate if the patient is effectively using the current medications. Oral prednisone would be appropriate for an exacerbation of poorly controlled severe persistent asthma. It would improve asthma control, but without proper education in the use of the inhaler, symptoms would most likely return when the corticosteroid dosage is tapered. Furthermore, oral corticosteroids have increased adverse effects. Simply having the patient return with a symptom and treatment log would not be expected to identify poor inhaler technique, although it would be helpful to assess compliance and symptom pattern.

Pneumo 68 A 71-year-old man is evaluated for a 3-month history of daily nonproductive cough. He walks regularly and his weight has been stable. He has not had hemoptysis, postnasal drainage, or gastroesophageal reflux. The patient smoked a pack of cigarettes a day for 40 years but quit smoking 6 years ago. He has a history of mild airway obstruction, and his only medication is albuterol as needed. On physical examination, vital signs are normal; the BMI is 23.5. Breath sounds are mildly reduced without wheezes or crackles. There is no lymphadenopathy. Spirometry shows an FEV1 of 80% of predicted, an FVC of 81% of predicted, and an FEV1/FVC ratio of 75%. The DLCO is 80% of predicted. Complete blood count, liver enzymes, and serum calcium are normal. Chest radiograph shows a 3-cm right hilar mass. CT scan shows a 3.4- × 3.2-cm spiculated mass in the right upper lobe and right hilar lymphadenopathy but no mediastinal lymphadenopathy. Bronchoscopy shows erythema and nodular change in the right upper lobe bronchus, and biopsy specimen shows squamous cell carcinoma. Which of the following is the most appropriate next step in the management of this patient? A Dynamic CT contrast enhancement study B Positron emission tomography-CT (PET-CT) C Quantitative lung perfusion scan D Surgical resection

B Positron emission tomography (PET) scanning (or PET-CT if available) is helpful in the preoperative evaluation of patients with known or suspected non-small cell lung cancer. Positron emission tomographic (PET) scanning has been shown to be cost effective in the preoperative management of patients with non-small cell lung cancer. Randomized studies evaluating the addition of PET to the standard workup found that the procedure identifies advanced disease and precludes unnecessary thoracotomy in approximately 1 in 5 patients. PET-CT has been shown to have higher sensitivity and specificity for assessing lung cancer stage than CT and PET done separately. PET-CT may not be widely available but will likely completely replace PET alone. Current guidelines identify low-risk patients for surgery based upon preoperative spirometry. An FEV1 greater than 2.0 L (or greater than 80% of predicted) identifies low-risk patients for pneumonectomy, whereas an FEV1 greater than 1.5 L is low risk for lobectomy. Measurement of DLCO is generally reserved for patients who have an acceptable FEV1 but also have exertional dyspnea or coexistent interstitial lung disease. Patients with preoperative results for FEV1 and DLCO that are both greater than 80% of predicted do not need further physiologic testing. The patient has no limitation to walking and adequate pulmonary function, and, therefore, quantitative perfusion lung scan would not be needed. Surgical consultation is probably premature before staging the tumor and determining its resectability. A dynamic CT enhancement study is helpful in the evaluation of indeterminate nodules but would not be expected to add helpful information in the patient with an established diagnosis.

Pneumo 76 A 50-year-old man is evaluated for a 1-year history of cough productive of mucoid sputum. He has not had fever or lost weight during that time. The patient has smoked one pack of cigarettes a day for 20 years. He has no history of allergic disease or gastroesophageal reflux disease. On physical examination, the temperature is 37.0 °C (98.6 °F), the blood pressure is 124/76 mm Hg, the pulse rate is 78/min and regular, and the respiration rate is 15/min; BMI is 25. The lungs are clear, and the chest radiograph is normal. Spirometry shows an FEV1 of 85% of predicted and an FEV1/FVC ratio of 75%; there is no change in results after administration of a bronchodilator. Which of the following is the most likely diagnosis? A Asthma B At risk for chronic obstructive pulmonary disease (COPD) C Mild (GOLD stage I) COPD D Moderate (GOLD stage II) COPD

B Smokers with chronic cough, sputum production, and normal lung function are classified as "at risk" for chronic obstructive pulmonary disease. The patient's lung function as measured by spirometry does not fall into any of the Global Initiative on Obstructive Lung Disease (GOLD) stage classifications for chronic obstructive pulmonary disease (COPD). He is, however, at risk for COPD because of his smoking history and chronic cough and sputum production. The only definitive diagnosis of COPD is by spirometry. At-risk patients used to be classified as having GOLD stage 0 disease. The current staging system of GOLD is as follows: GOLD Stage At risk Normal spirometry Chronic symptoms (cough, sputum production) I. Mild FEV1/FVC ≤70, FEV1 ≥80% of predicted II. Moderate FEV1/FVC ≤70% FEV1 50% to 80% of predicted III. Severe FEV1/FVC ≤70%, FEV1 30% to 50% of predicted IV. Very Severe FEV1/FVC ≤70% FEV1 ≤30% of predicted or ≤50% of predicted plus chronic respiratory failure The outcome for treating at-risk patients is not yet clear. This population of patients is largely underdiagnosed. Because they are younger and mostly active smokers, detection at this stage may provide an opportunity for early intervention and initiating a smoking cessation program. About 15% of long-time smokers develop clinically significant COPD. Patients who quit smoking have a reduced decline in pulmonary function. This patient does not have asthma. There is no evidence of bronchospasm, atopy, or airflow hyperreactivity or reversibility.

Pneumo 73 A 68-year-old man with chronic obstructive pulmonary disease, hypertension, and hyperlipidemia is being weaned from mechanical ventilation after an exacerbation. The patient's current medications are ipratropium bromide and albuterol (both by metered-dose inhaler through the ventilator), prednisone, lisinopril, and atorvastatin. He is started on a spontaneous breathing trial, which he initially tolerates well but later shows evidence of oxygen desaturation and agitation. He is given increasing doses of lorazepam to cause sedation, and assist-control ventilation is resumed. The following day he is calm but is not focused and fails to follow commands consistently. Which of the following is the best test to assess the patient's mental status? A Beck Depression Inventory B Confusion Assessment Method for the Intensive Care Unit (CAM-ICU) C CT scan of the head D Metabolic profile E Mini-Mental State Examination

B The Confusion Assessment Method for the Intensive Care Unit (CAM-ICU), a clinical instrument for use in evaluating a patient in the intensive care unit for delirium, takes less than 1 minute and is recommended for all mechanically ventilated patients. The Confusion Assessment Method for the Intensive Care Unit (CAM-ICU) is an instrument for nurses and physicians to use in evaluating a patient for delirium; the assessment takes less than 1 minute and is recommended for routine monitoring of all mechanically ventilated patients. The CAM-ICU, a well-validated and highly reliable method now translated into more than 10 languages, is widely used for monitoring delirium in ICU patients. The prevalence of delirium in most studies of mechanically ventilated patients is between 50% and 80%. ICU delirium has been shown to be an independent predictor of ICU and hospital length of stay, cost of care, cognitive status at hospital discharge, and 6-month mortality. The agitated, hyperactive subtype of delirium is much less common than the "quiet," hypoactive subtype, which is generally associated with a lower likelihood of survival. The Beck Depression Inventory II consists of 21 items to assess the intensity of depression in clinical and normal patients. The Mini-Mental State Examination (MMSE) is a 30-point questionnaire that is used to screen for cognitive impairment. It is commonly used to screen for dementia. It is also used to estimate the severity of cognitive impairment at a given time and to follow the course of cognitive changes in a patient over time. The diagnosis of delirium is a clinical one, and there are no laboratory tests, imaging studies, or other tests that can provide greater accuracy than the CAM-ICU algorithm. Specifically, a head CT scan and metabolic profile will not establish the diagnosis of delirium as effectively as CAM-ICU.

Pneumo 110 A 28-year-old man is admitted to the intensive care unit after having sustained a large aneurysmal subarachnoid bleed and is intubated for airway protection and ventilatory support. Because of increased intracranial pressure and patient-ventilator dyssynchrony, propofol infusion is started at 100 µg/kg/min. The patient's medical course is complicated by gram-positive sepsis and adrenal insufficiency on day 3 in the intensive care unit, for which he is treated with broad-spectrum antibiotic therapy, volume resuscitation, norepinephrine, and hydrocortisone with the goal of maintaining a mean arterial pressure greater than 65 mm Hg. On day 4, the patient remains sedated on mechanical ventilation, and he appears to be improving. On day 5, the patient develops acute kidney injury, hyperkalemia, metabolic acidosis, rhabdomyolysis, and jugular venous distention with cardiac failure. Electrocardiogram shows no signs of an acute myocardial infarction or ischemia and no peaked T waves or conduction disturbances. Which of the following is the most urgent next step in the management of this patient? A CT scan of the head without contrast B Discontinuation of propofol C Intravenous heparin D Plasmapheresis

B The propofol infusion syndrome in adults occurs primarily in patients with acute neurologic or acute inflammatory diseases complicated by severe infection or sepsis and who are receiving catecholamines or corticosteroids in addition to propofol. This patient has the propofol infusion syndrome, and the drug should be discontinued and replaced by fentanyl and midazolam. The propofol infusion syndrome is a rare and often fatal syndrome originally described in critically ill children undergoing long-term propofol infusion at high doses. The syndrome has recently been reported in adults, mostly in patients with acute neurologic illnesses or acute inflammatory diseases complicated by severe infections or even sepsis and who are receiving catecholamines and/or corticosteroids in addition to propofol. The main features of the syndrome consist of cardiac failure, rhabdomyolysis, severe metabolic acidosis, and renal failure associated with hyperkalemia. Central nervous system activation with production of catecholamines and corticosteroids, and systemic inflammation with cytokine production are priming factors for cardiac and peripheral muscle dysfunction. High-dose propofol, but also supportive treatments with catecholamines and corticosteroids act as triggering factors. At the subcellular level, propofol impairs free fatty acid utilization and mitochondrial activity. The syndrome can be lethal if not identified early, and caution should be exercised when using prolonged (more than 48 h) propofol sedation at doses greater than 75 µg/kg/min, particularly in patients with acute neurologic or inflammatory illnesses. In these cases, alternative sedative agents should be considered immediately, and monitoring of the plasma levels of troponin I, creatine kinase, and myoglobin should be undertaken. There is no need to obtain a CT scan of the head, which would pose added risk of transport for the patient. Beyond immediate discontinuation of the drug, the treatment of propofol infusion syndrome is supportive. Supportive treatment may ultimately include bicarbonate infusion, hemodialysis, treatment of heart failure, and cardiac pacing for profound bradycardia. There is no indication for intravenous heparin at this time. Plasmapheresis has no role in managing the propofol infusion syndrome.

Pneumo 47 A 70-year-old woman is hospitalized for pneumococcal pneumonia. She was ambulatory until the day of admission but is now unable to leave her bed. She has a permanent venous catheter through her left subclavian vein. The patient has a history of stage IIIB non-small cell lung cancer. On physical examination, the temperature is 37.8 °C (100.0 °F), the blood pressure is 110/70 mm Hg, the pulse rate is 110/min, and the respiration rate is 22/min. On admission, the patient's serum creatinine level was 3.0 mg/dL (265.2 µmol/L); it was 0.7 mg/dL (61.9 µmol/L) 1 month ago. Which of the following is the best management to prevent venous thromboembolism in this patient? A Aspirin B Low-dose unfractionated heparin, subcutaneously C Low-molecular-weight heparin, subcutaneously D Periodic lower extremity ultrasonography E Warfarin to maintain the INR at 2.0 to 3.0

B Unfractionated heparin does not depend on normal renal function for clearance and is appropriate for prophylaxis of venous thromboembolism for patients with renal insufficiency. This patient is at substantial risk for venous thromboembolism because of her malignancy and her acute medical illness. Unfractionated heparin and low-molecular-weight heparins have been shown to reduce the risk of venous thromboembolism and are highly recommended for patients at moderate or high risk. Unfractionated heparin does not rely on normal renal function for clearance and would be an appropriate option in this patient. Aspirin is not commonly recommended for the prevention of venous thromboembolism and has not been demonstrated by clinical trials to reduce the incidence of thromboembolism in most populations at risk. Low-molecular-weight heparins are renally cleared and would accumulate unpredictably in this patient with acutely worsening renal function. The same limitation is applicable to fondaparinux. Ultrasonographic surveillance for deep venous thrombosis and subsequent treatment has not been shown to improve outcomes. Warfarin, especially at therapeutic doses, is not recommended as a first choice for prophylactic use in medical patients. Its long duration of action may be especially detrimental in medical inpatients who may need invasive procedures. In addition, it would take several days before the INR reached the therapeutic target.

Pneumo 35 A 42-year-old man is evaluated in the hospital for dyspnea and pleuritic chest pain. The patient had a fracture of the right femur 3 weeks ago. He has hypertension, and his only medication is hydrochlorothiazide. On physical examination, the temperature is 38.1 °C (100.6 °F), the pulse rate is 110/min, the respiration rate is 22/min, the blood pressure is 130/78 mm Hg, and the BMI is 24. Routine laboratory studies are normal; serum troponins are undetectable. Electrocardiography shows increased height of R waves in leads V4-V6; the QRS complex has a leftward axis. Contrast-enhanced CT scan shows pulmonary emboli in the arteries perfusing the lingula and the posterior basal segment of the left lower lobe. Which of the following is the most appropriate treatment for this patient? A Inferior vena cava filter B Intravenous unfractionated heparin C Intravenous tissue plasminogen activator D Mechanical clot dissolution E Surgical embolectomy

B Unfractionated heparin, low-molecular-weight heparin, or fondaparinux is generally sufficient initial therapy for acute pulmonary embolism. The patient has an acute pulmonary embolism. In the absence of contraindications, the patient should be treated initially with intravenous or subcutaneous unfractionated heparin, low-molecular-weight heparin, or fondaparinux. Electrocardiographic abnormalities are present in 70% of patients with pulmonary embolism. Most common abnormalities are ST segment and T wave changes (49%). Cor pulmonale, right axis deviation, right bundle branch block, and right ventricular hypertrophy occur less frequently. T wave inversions in precordial leads may indicate more severe right ventricular dysfunction. This patient's electrocardiographic findings support the diagnosis of left ventricular hypertrophy/strain, most likely because of his essential hypertension. Patients with hemodynamically unstable pulmonary embolism have a high mortality rate. The role of thrombolytic agents in pulmonary embolism is unclear. There are no clinical trials comparing thrombolytic agents with other forms of therapy for massive pulmonary embolism, and management decisions must therefore be made by inference from studies in stable patients. The Urokinase in Pulmonary Embolism Trial reported a short-term improvement in cardiac output and pulmonary pressure with thrombolytic therapy but no improvement in morbidity or mortality and increased bleeding. Acute pulmonary embolectomy is rarely warranted because medical therapy is successful, patient selection difficult, and the results of acute embolectomy unimpressive. However, if experienced surgical intervention is possible, embolectomy may be considered for a confirmed, massive embolism that fails to respond promptly to medical therapy. Mechanical clot dissolution has been performed in a small number of patients in cardiogenic shock secondary to massive pulmonary embolism. The mortality rate is high, and this intervention is not readily available in most institutions. Placement of an inferior vena cava filter might be considered in a patient with a contraindication to anticoagulation, the onset of clinically important bleeding during anticoagulation, recurrent pulmonary embolism despite adequate anticoagulation, or in hemodynamically unstable patients.

Pneumo 43 A 45-year-old woman is evaluated in the emergency department for an exacerbation of asthma that started 2 days after the onset of an upper respiratory tract infection. Her symptoms include wheezing, a productive cough, and chest tightness that have not responded to frequent use of an albuterol inhaler. Her medical history is unremarkable except for long-standing asthma. She has been hospitalized for asthma five or six times, one of which required treatment in the intensive care unit. Her current medications include a high-dose inhaled corticosteroid, inhaled long-acting β-agonist, and albuterol as needed. On physical examination, she is alert but anxious; the temperature is 37.0 °C (98.6 °F), the blood pressure is 140/85 mm Hg, the pulse rate is 115/min, and the respiration rate is 32/min. Oxygen saturation is 95% on low-flow supplemental oxygen. Chest examination shows reduced breath sounds over both lung fields with prolonged expiration and minimal wheezing. Spirometry reveals an FEV1 of 1.08 L (35% of predicted) and an FVC of 2.9 L (70% of predicted). After nebulized albuterol/ipratropium bromide, the patient's condition improves minimally. Repeat physical examination shows a temperature of 37.1 °C (99.0 °F), pulse rate of 130/min, and respiration rate of 24/min. Examination of the lungs shows very decreased air entry and minimal wheezing; spirometry shows an FEV1 of 0.96 L (31% of predicted) and FVC of 2.8 L (67% of predicted). Which of the following is the most important next step in the evaluation of this patient? A Complete blood count B Electrocardiography C Measurement of arterial blood gases D Measurement of peak expiratory flow

C In patients in status asthmaticus, pulse oximetry is a good monitoring tool but is not a substitute for determining actual oxygenation by measuring arterial blood gases. This patient is in status asthmaticus. She has not responded well to bronchodilator therapy and is at risk for respiratory failure. Pulse oximetry is a good screening and monitoring tool but is not a substitute for measuring blood gases. Oximetry would be in a normal range even when PO2 has declined significantly because of the sigmoid shape of the oxygen dissociation curve, where the hemoglobin saturation reaches a flat part of the curve at PO2 levels higher than 60 mm Hg. Measuring blood gases is essential to evaluate her ventilation and to direct management because decreased alveolar ventilation may not be reflected on pulse oximetry, particularly in patients receiving supplemental oxygen. In mild acute asthma exacerbations, the PCO2 is decreased. With increasing severity of the attack, PCO2 increases and reaches normal levels in moderate to severe attacks. Elevated PCO2 is an ominous sign, indicating severe obstruction and risk of respiratory arrest. A chest radiograph would be expected to show hyperinflation in acute severe asthma and can be very helpful to exclude other diagnoses or evaluate complications (for example, pneumothorax, atelectasis). After blood gases have been measured, a chest radiograph is the next most important step in evaluating this patient. Complete blood count with differential is not expected to change the management in most patients with acute asthma. Electrocardiography is likely to show sinus tachycardia and would not be expected to help in the management of this patient. Peak expiratory flow rate is unlikely to add useful information beyond what is gained from spirometry.

Pneumo 59 A 54-year-old woman is evaluated for a 1-year history of progressive exertional dyspnea. She does not smoke cigarettes or use illicit drugs. The patient has a history of obesity and has used various appetite suppressants but takes no other medications. She has no significant family or personal or medical history. On physical examination, she is afebrile; the blood pressure is 100/60 mm Hg, the pulse rate is 98/min, the respiration rate is 20/min, and the BMI is 38. The lungs are clear on auscultation and percussion. There is a loud pulmonic component to S2, which is also split during inhalation and exhalation. Spirometry and plethysmography are normal. Arterial blood gases are normal. Ventilation/perfusion scan shows diffusely nonhomogeneous perfusion but no segmental or subsegmental regions of perfusion defect. Chest radiograph shows enlarged pulmonary arteries and enlarged right-sided heart chambers but no parenchymal abnormalities. Transthoracic echocardiography shows decreased cardiac output, right ventricular hypertrophy and dilation, and right atrial enlargement. The left ventricle is somewhat compressed by the intraventricular septum. Which of the following is the most appropriate next step in the management of this patient? A High-resolution CT scan of the chest B Myocardial perfusion imaging C Right-heart catheterization D Therapeutic trial of enalapril E Transbronchial lung biopsy

C In patients with pulmonary hypertension, right-heart catheterization will confirm the presence of pulmonary arterial hypertension, quantify pulmonary vascular resistance, determine the magnitude of right ventricular dysfunction, and guide therapy. This patient has signs and symptoms of pulmonary hypertension and a history of use of appetite suppressants, which have been associated with pulmonary hypertension. Echocardiography has ruled out the presence of cardiac diseases associated with pulmonary hypertension. Pulmonary function testing has ruled out parenchymal obstructive and restrictive lung diseases, and ventilation/perfusion scanning has ruled out chronic thromboembolic pulmonary hypertension. Right-heart catheterization will confirm the presence of pulmonary arterial hypertension, measure pulmonary vascular resistance, determine the magnitude of right ventricular dysfunction, and guide therapy. High-resolution CT scan is useful for the evaluation of pulmonary parenchymal disease, but the absence of parenchymal abnormalities on chest radiograph and the normal pulmonary function tests make interstitial disease unlikely. This patient's physiologic testing discloses no evidence of lung parenchymal disease. Myocardial perfusion imaging can help detect compromised coronary flow. However, the echocardiographic findings can be explained by right ventricular overload and do not suggest acute coronary disease. Lung biopsy would add little to the diagnosis of pulmonary hypertension and entails a risk of bleeding in patients with pulmonary hypertension. Therapy with an angiotensin-converting enzyme (ACE) inhibitor may improve left ventricular function in patients with left ventricular systolic dysfunction. However, this patient's left ventricular dysfunction is attributable to compression from right ventricular hypertrophy and dilation. ACE inhibitors would not improve the pulmonary arterial resistance and might even cause dangerous decreases in this patient's blood pressure.

Pneumo 90 A 65-year-old man is evaluated for a 9-month history of progressive dyspnea; for the past month the dyspnea has been severe and accompanied by dizziness. The patient had an acute pulmonary embolism 3 years ago, treated acutely with heparin and then warfarin for 6 months. He has no other significant medical history and takes no medication. On physical examination, he is afebrile; the blood pressure is 105/60 mm Hg, the pulse rate is 105/min at rest and 115/min with minimal exertion, the respiration rate is 22/min, and the BMI is 32. The S2 is split, there is a right ventricular heave, and there is a holosystolic murmur that increases with inspiration. Echocardiography reveals severe right ventricular dilation and dysfunction and right atrial enlargement. Ventilation/perfusion scan shows decreased perfusion to most of the right lower lobe, right middle lobe, and left lower lobe, with normal ventilation. Warfarin therapy is started. Which of the following is the most appropriate management for this patient? A Intravenous tissue plasminogen activator B Oral bosentan C Pulmonary thromboendarterectomy evaluation D Vasodilator therapy

C The definitive therapy for chronic thromboembolic pulmonary hypertension is pulmonary thromboendarterectomy. This patient has echocardiographic evidence of pulmonary hypertension and evidence on ventilation/perfusion scanning of regional perfusion defects suggestive of chronic thromboembolic pulmonary hypertension (CTEPH). Pulmonary angiography is typically performed next and can confirm the diagnosis and determine whether the large-vessel obstruction is amenable to surgical therapy. The definitive therapy is pulmonary thromboendarterectomy, which is performed only in centers with special expertise in the preoperative evaluation, surgery, and postoperative therapy for these patients. CTEPH is a progressive disease that causes significant mortality if not evaluated and treated in a timely manner. Although CTEPH is a complication of acute pulmonary embolism, the disease is due to a combination of organized intra-arterial scarring and a secondary small-vessel reaction. Acute anticoagulation with heparin followed by chronic warfarin therapy or thrombolysis would do little to reverse the disease. Bosentan may have a role in the treatment of patients with CTEPH in whom surgery is not possible. However, bosentan therapy is controversial and would not relieve the primary cause of the pulmonary artery obstruction. It would not be appropriate to use this controversial treatment instead of referral for definitive treatment by pulmonary thromboendarterectomy.

Pneumo 88 A 55-year-old woman is evaluated in follow-up for chronic obstructive pulmonary disease. The disorder was diagnosed 1 year ago, and she has had two exacerbations requiring hospitalization. She has a chronic cough with sputum production and mild dyspnea on exertion. She smokes a pack and a half of cigarettes a day. She wants to stop smoking and once tried a nicotine patch but was unable to stop. She also has a history of hypertension, seizure disorder, and major depression with suicidal ideation that is in remission. Her medications are albuterol as needed, an inhaled corticosteroid, salmeterol, enalapril, phenytoin, and sertraline. On physical examination, vital signs and heart rate and rhythm are normal, and there are no murmurs or crackles. Breath sounds are diminished bilaterally. Spirometry shows an FEV1 of 50% of predicted and an FEV1/FVC ratio of less than 70%. Oxygen saturation with exertion is 93% on ambient air. Chest radiograph shows hyperinflation with no infiltrates. Which of the following is the best option for smoking cessation in this patient? A Bupropion B Bupropion and a nicotine patch C High-dose nicotine patch with as-needed nicotine replacement with gum or spray D Varenicline

C A high-dose nicotine patch in combination with another antismoking therapy, such as nicotine replacement therapy (gum and/or spray), is the most effective therapy for smoking cessation. This patient has already used a nicotine patch unsuccessfully. However, using the high-dose patch again but in combination with another therapy, such as nicotine replacement with gum or spray, gives the best long-term cessation rate. Bupropion has been shown to be effective in smoking cessation and is generally well tolerated. However, seizures occur in approximately 0.1% of patients who take the drug, and the risk appears to be higher in patients with a preexisting seizure disorder, anorexia nervosa, or bulimia. This patient has a history of seizure disorder, and, therefore, bupropion is contraindicated. Varenicline can be used for smoking cessation, and the patient should try to stop smoking 1 week after starting varenicline. The common side effects are nausea and abnormal dreams. The U.S. Food and Drug Administration (FDA) has issued information regarding post-marketing reports of suicidal thoughts and erratic/aggressive behavior in patients who have taken varenicline. Many cases suggest new-onset depression, suicidal ideation, and emotional/behavioral changes within days to weeks after treatment initiation. It is not clear whether these effects are related to the drug itself or to the effects of smoking cessation. In this patient with a history of major depression and suicidal ideation, varenicline would not be indicated. In 2008, the FDA reported that it is increasingly likely that varenicline is associated with serious neuropsychiatric symptoms. There are also reports of patients experiencing drowsiness that affects their ability to drive or operate heavy machinery. The investigation by the FDA is ongoing.

Pneumo 16 A 30-year-old medical resident is evaluated for cough, right-sided chest pain, and fever of 21 days' duration. He has no significant medical history or family history, and he takes no medications. Hemoglobin is 14 g/dL (140 g/L), and the leukocyte count is 8000/µL (8 × 109/L). Chest radiograph shows a right pleural effusion occupying approximately 50% of the hemithorax without other abnormalities. Thoracentesis yields turbid, yellow fluid, and analysis shows: Erythrocyte count 500/µL Nucleated cell count 3500/µL (3.5 × 109/L) with 20% neutrophils, 60% lymphocytes, 10% macrophages, 4% mesothelial cells, and 6% eosinophils Total protein 4.2 g/dL (42 g/L) Lactate dehydrogenase 240 U/L pH 7.35 Glucose 68 mg/dL (3.8 mmol/L) Serum total protein is 7.0 g/dL (70 g/L) and serum lactate dehydrogenase is 100 U/L. Gram stain shows no organisms and culture is pending. Which of the following is the most appropriate next step in management? A Chest CT scan B Flexible bronchoscopy C Pleural biopsy D Repeat chest radiograph after a 5-day course of azithromycin

C A patient with tuberculous pleural effusion typically presents with a lymphocyte-predominant exudative effusion; however, within the first 1 to 2 weeks, neutrophils can predominate as the cellular response evolves from neutrophils to lymphocytes. The patient likely has a tuberculous pleural effusion based on the subacute (3-week) duration of symptoms and the characteristics of the pleural effusion. Because of the patient's age and the presentation with an isolated pleural effusion, primary tuberculosis is most likely. A tuberculous effusion is typically exudative by both protein (pleural fluid to serum protein ratio greater than 0.5) and lactate dehydrogenase (LDH) criteria (pleural fluid to serum LDH ratio greater than 0.6 and pleural fluid to serum upper limits of normal LDH ratio greater than 0.67). The cellular response in the pleural fluid is classically lymphocytic (greater than 80% mature lymphocytes). However, it can be neutrophilic within the first 2 weeks, after which it typically evolves into the classic lymphocyte-predominant exudate. Whereas pleural fluid cultures for Mycobacterium are positive in less than one third of cases, the combination of pleural biopsy for histologic evaluation and culture is typically positive in more than two thirds of cases. The 3-week history of symptoms is too long for a typical bacterial pneumonia, no definite infiltrate was present on the chest radiograph, and the cellular response in the pleural fluid was primarily lymphocytic rather than neutrophilic. Therefore, a bacterial pneumonia with a parapneumonic effusion is unlikely, and an empiric course of azithromycin would not be appropriate. Chest CT scan might be helpful to assess whether there is an underlying parenchymal infiltrate that was not visible on plain chest radiograph, but it would not help in determining the underlying cause of the pleural effusion. Flexible bronchoscopy, with collection of samples for histology and culture, is useful for diagnosing pulmonary tuberculosis in the setting of pulmonary parenchymal disease. However, the yield from culture of bronchopulmonary secretions (obtained either as sputum or bronchoscopic samples) is low, especially in the absence of pulmonary parenchymal abnormalities on chest radiograph.

Pneumo 79 A 60-year-old man is evaluated for the insidious onset of dyspnea on exertion over the past several weeks. He has also had a productive cough and lost 4.5 kg (10 lb). The patient has a 40-pack-year history of cigarette smoking; his medical history includes hypertension, and his only medication is enalapril. On physical examination, he appears healthy and relaxed. The temperature is 37.0 °C (98.6 °F), the blood pressure is 138/80 mm Hg, the pulse rate is 85/min and regular, and the respiration rate is 18/min; oxygen saturation is 93% on ambient air. There is no cervical, supraclavicular, or axillary lymphadenopathy. There is dullness to percussion at the left lung base with minimally decreased breath sounds compared with the right. Chest radiograph shows evidence of emphysema and a small left-sided pleural effusion. Thoracentesis is done, and analysis of the pleural fluid shows: Erythrocyte count 1000/µL Nucleated cell count 1700/µL (1.7 × 109/L) with 7% neutrophils, 63% lymphocytes, 18% macrophages, 8% mesothelial cells, and 4% eosinophils Total protein 4.0 g/dL (40 g/L) Lactate dehydrogenase 200 U/L pH 7.25 Glucose 65 mg/dL (3.6 mmol/L) Amylase 160 U/L Pleural fluid to serum amylase ratio is 2.1. Isoenzyme analysis reveals that 85% of the amylase is salivary. Cytology report is pending Which of the following is the most likely diagnosis? A Acute pancreatitis B Esophageal rupture C Lung cancer D Pancreatic pseudocyst

C An elevated pleural fluid amylase level suggests that the cause of the pleural effusion is pancreatic disease, esophageal perforation, or malignancy. A pleural fluid amylase concentration greater than the upper limits of normal for serum amylase or a pleural fluid to serum amylase ratio greater than 1.0 suggests the following diagnoses: pancreatic disease (either acute pancreatitis or pancreaticopleural fistula), malignancy (most commonly adenocarcinoma of the lung), and esophageal perforation. Finding salivary isoamylase in pleural fluid virtually excludes acute pancreatitis and pancreaticopleural fistula. As the patient has not had an endoscopic procedure, particularly esophageal dilation, and has had no chest pain, fever, chills, severe vomiting, and retching, it would be highly unlikely that he had barogenic esophageal perforation. The pleural fluid amylase level was relatively low compared with values frequently over 100,000 U/L in the setting of a direct communication between a pancreatic pseudocyst and the pleural space with a pancreaticopleural fistula. Therefore, finding a high salivary isoamylase concentration in pleural fluid in the absence of esophageal perforation increases the likelihood of malignancy, most commonly adenocarcinoma of the lung because adenocarcinoma cells produce a salivary-like isoamylase.

Pneumo 114 A 30-year-old woman with persistent asthma is evaluated for frequent nocturnal awakenings. She has wheezing several nights a week despite treatment with a moderate-dose inhaled corticosteroid and a long-acting β-agonist. She does not have daytime symptoms. She developed oral thrush when she was taking higher doses of the inhaled corticosteroid. The patient is obese and occasionally snores but has had no witnessed apneic episodes. She has no other symptoms and takes no other medications. She has excellent inhaler technique as documented by direct observation. On physical examination, vital signs are normal; BMI is 30. The cardiovascular examination is normal, and the lungs are clear. Which of the following is the most appropriate management for this patient? A Increase the evening long-acting β-agonist dose B Increase the dose of inhaled corticosteroid C Prescribe a 2-month trial of omeprazole D Recommend an allergen-impermeable bedcover

C In patients with asthma who have increased nocturnal symptoms despite adequate daytime control, a trial of gastric acid suppression therapy is warranted. Gastroesophageal reflux disease (GERD) is common in patients with asthma. Obesity is also associated with an increased prevalence of GERD, which may lead to acute airway hyperresponsiveness and an exacerbation of asthma. Because of the patient's increased nocturnal symptoms, a trial of gastric acid suppression with a proton pump inhibitor is warranted. Additional measures to reduce gastric acid and GERD include elevating the head of the bed and refraining from eating and drinking 2 hours before bedtime. Certain foods (tomatoes, chocolate, caffeinated beverages) promote acid reflux and should be avoided. Increasing the inhaled corticosteroid dose is not appropriate because the patient has good daytime control, suggesting that a factor specifically exacerbating her bronchospasm at night is present. Furthermore, increasing the corticosteroid dose is likely to lead to a repeated episode of oral thrush. Increasing the β-agonist dose is also inappropriate because the approved dose is fixed and such an increase could cause difficulty sleeping. The use of an allergen-impermeable bedcover as a main intervention to reduce exposure to house dust mites is not very effective in patients with asthma. It has been proposed that its use as part of a vigorous approach to reduce environmental allergen exposure might be of some benefit in allergic patients with difficult-to-control asthma. b

Pneumo 66 A 75-year-old woman with long-standing asthma is evaluated for a 1-month history of nocturnal asthma symptoms at least weekly and the need to use an albuterol inhaler daily. Her asthma therapy is a moderate-dose inhaled corticosteroid. The patient is otherwise healthy. On physical examination, she has occasional wheezing, but the rest of the examination is unremarkable. On office spirometry, the FEV1 is 2.2 L (70% of predicted) and FVC is 3.9 L (85% of predicted). Which of the following is the most appropriate management for this patient? A Adding a leukotriene modifier B Adding a long-acting anticholinergic agent C Adding a long-acting β-agonist D Adding theophylline E Doubling the corticosteroid dose

C In patients with persistent asthma not adequately controlled with daily low- or moderate-dose inhaled corticosteroids, adding a long-acting β-agonist improves asthma control and quality of life. This patient has persistent asthma, which is defined as having asthma symptoms 2 or more days per week or 2 or more nights per month. Patients with persistent asthma should be treated with daily corticosteroid therapy. When asthma is not adequately controlled on low- or moderate-dose inhaled corticosteroid therapy, adding a long-acting β-agonist (salmeterol or formoterol) has been shown to be superior to doubling the dose of the corticosteroid for improving asthma control and quality of life. The concerns about increased asthma-related deaths in patients using a long-acting β-agonist led the U.S. Food and Drug Administration to include a black box warning in the package insert for these drugs. The National Asthma Education and Prevention Program (NAEPP) expert panel guidelines in 2007 affirmed the recommendation of adding a long-acting β-agonist in patients whose disease is not controlled with an inhaled corticosteroid but advised against using a long-acting β-agonist as a single controller therapy. Theophylline and leukotriene-modifying drugs are third-line agents that should be considered in patients who remain symptomatic despite the addition on a long-acting β-agonist to the corticosteroid therapy. Long-acting anticholinergic drugs are beneficial in patients with chronic obstructive pulmonary disease; however, their role in management of asthma is not defined.

Pneumo 70 An 82-year-old homeless man is brought to the emergency department after being found unconscious in December. On physical examination, the patient appears malnourished; the rectal temperature is 29.0 °C (84.2 °F), the blood pressure is 110/60 mm Hg, and the pulse rate is 55/min. He is unresponsive to voice but responds to deep painful stimuli. His pupils are sluggishly reactive to light. Which of the following is the most appropriate next step in this patient's management? A Active external rewarming of the extremities B Active external rewarming of the truncal areas C Active internal rewarming techniques D Passive rewarming techniques

C In patients with severe hypothermia (temperature less than 30.0 °C [86.0 °F]), active internal rewarming measures using warmed intravenous fluids, warm and humid oxygen, peritoneal lavage, and extracorporeal rewarming should be considered. Hypothermia is a significant and common problem among elderly people because of such factors as impaired shivering. Normal body temperature is 35.6 °C (96.1 °F) to 36 .8 °C (98.3 °F) in the elderly. Medications may also predispose the elderly to hypothermia. There are three types of rewarming techniques: passive external rewarming, active external rewarming, and active internal core rewarming. Passive rewarming (use of blankets and insulation) and active external rewarming techniques (warm blankets, heating pads, radiant heat, warm baths, forced warm air) are indicated for mild hypothermia with temperatures from 33.9 °C (93.0 °F) to 36.1 °C (97.0 °F). The same maneuvers can be used for moderate hypothermia except that only the truncal areas should be externally rewarmed. Heat applied to the arms and legs forces cold blood back toward the heart, lungs, and brain, causing the core body temperature to decrease, which can worsen the preexisting hypothermia. In this patient with severe hypothermia (temperature less than 30.0 °C [86.0 °F]), active internal rewarming measures should be considered, such as warmed intravenous fluids, warm and humid oxygen, peritoneal lavage, and extracorporeal rewarming. Complications of rewarming include rhabdomyolysis, compartment syndromes, disseminated intravascular coagulation, pulmonary edema, and acute tubular necrosis.

Pneumo 1 A 28-year-old man is evaluated for a 6-month history of episodic dyspnea, cough, and wheezing. As a child, he had asthma and allergies, but he has been asymptomatic since his early teenage years. His recent symptoms started after an upper respiratory tract infection, and they are often triggered by exercise or exposure to cold air. He is also awakened with asthma symptoms 5 or 6 nights a month. He is otherwise healthy and takes no medications. On physical examination, vital signs are normal. There is scattered wheezing in both lung fields. Chest radiograph is normal. Spirometry shows an FEV1 of 70% of predicted with a 15% improvement after inhaled albuterol. Which of the following is the most appropriate therapy for this patient? A Azithromycin B Inhaled albuterol as needed C Inhaled low-dose corticosteroids plus inhaled albuterol as needed D Long-acting β-agonist E Long-acting β-agonist plus inhaled albuterol as needed

C Inhaled corticosteroids are the cornerstone of therapy for persistent asthma. Asthma symptoms on 2 or more days a week (or 2 or more nights a month) is the defining characteristic of persistent asthma. Inhaled corticosteroids are the cornerstone of therapy for persistent asthma. Regular use of inhaled corticosteroids is associated with improved pulmonary function, reduced airway hyperresponsiveness, decreased asthma exacerbations, and reduced mortality. Side effects of inhaled corticosteroids include oral candidiasis and dysphonia related to laryngeal muscle myopathy. Systemic effects may occur with use of inhaled corticosteroids and are generally related to the dose and duration of use. In adults, these effects include osteopenia, skin thinning, and increased risk for cataracts or glaucoma. Therefore, the lowest dose consistent with disease control should be used. The treatment should be reevaluated every 3 to 6 months in stable patients, and adjustments made to step-up or step-down therapy based on disease control and occurrence of exacerbations. In between visits, patients should use a written asthma management plan, devised by their physician, to guide potential changes to their treatment. Albuterol should be used as needed in all patients with asthma, but by itself is not adequate therapy because it does not affect the underlying airway inflammation. Long-acting β-agonists (salmeterol and formoterol) provide bronchodilation for up to 12 hours and are effective in preventing exercise-induced asthma. These drugs do not have a clinically significant anti-inflammatory effect; therefore, their use without concomitant administration of corticosteroids may mask worsening of asthma control and lead to increased asthma-related complications, including the possibility of increased asthma-related deaths. Therefore, long-acting β-agonists are not appropriate as monotherapy or in place of inhaled corticosteroids. The use of antibiotics for atypical infections (Mycoplasma, Chlamydia) in asthma is under investigation. However, antibiotic therapy is not recommended unless there is evidence of an acute bacterial infection.

Pneumo 111 A 21-year-old woman is evaluated for a positive methacholine challenge test that she had as part of a research study. The patient has no history of wheezing or chest tightness. She has seasonal allergies manifesting as hay fever in fall and spring but is now asymptomatic. She is otherwise healthy and takes no medications. On physical examination, vital signs are normal. Examination of the conjunctivae, nose, posterior pharynx, heart, and lungs is normal. Routine laboratory studies are all normal. Which of the following is the most appropriate management for this patient? A Albuterol inhaler B Inhaled corticosteroids and a long-acting β-agonist C Seasonal use of nasal corticosteroids D Repeat methacholine challenge testing

C No specific therapy is needed for asymptomatic bronchial hyperresponsiveness. Airway responsiveness to methacholine is increased in patients with hay fever, smoking, bronchitis, chronic obstructive pulmonary disease, and cystic fibrosis, as well as after viral infections. The severity of airway hyperresponsiveness is most pronounced in patients with asthma. No specific asthma therapy is needed for asymptomatic bronchial hyperresponsiveness. This patient may benefit from therapy for hay fever, with nasal corticosteroids with or without antihistamines. There is no evidence that the patient has bronchial asthma, and, therefore, treatment with albuterol, inhaled corticosteroids, and long-acting β-agonists is not indicated. There is also no need for a repeat methacholine challenge test because no therapy is needed for the mildly abnormal response in asymptomatic patients.

Pneumo 57 An 84-year-old woman is transferred to the intensive care unit for severe dehydration and malnutrition. She had an episode of acute gastroenteritis 2 weeks ago, and since then she has had little to eat or drink because of persistent nausea. On physical examination, she appears cachectic and malnourished; the blood pressure is 80/40 mm Hg, pulse rate is 130/min, and respiration rate is 15/min. She has diffuse abdominal tenderness but no rebound tenderness. Her serum prealbumin level on admission is 10 mg/dL (100 mg/L) (less than 30 mg/dL [300 mg/L] is indicative of malnutrition). The patient weighs 50 kg (110 lb) and is 157 cm (62 in) tall. BMI is 20.3. Which of the following is the most appropriate daily nutritional support for this patient? A 500 kcal and 75 g protein B 1250 kcal and 100 g protein C 1500 kcal and 75 g protein D 3000 kcal and 120 g protein

C Patients in the intensive care unit require 25 to 30 nonprotein kcal/kg/d and 1.0 to 1.5 protein kcal/kg/d to meet the energy expenditures associated with critical illness. Patients in the intensive care unit generally require 25 to 30 nonprotein kcal/kg/d and 1.0 to 1.5 protein kcal/kg/d to meet the energy expenditures associated with critical illness. This severely malnourished patient should receive approximately 1500 kcal (30 nonprotein kcal/kg/d) and 75 g protein (1.5 protein kcal/kg/d). The oral or enteral route is usually preferred. Albumin and prealbumin levels are indicators of visceral protein status. Albumin has a half-life of approximately 20 days. When albumin values are below normal levels, a sizeable amount of the serum pool has been lost. Generally, albumin is considered a late indicator of malnutrition. Albumin concentrations rise slowly during nutritional therapy (refeeding) and in patients recovering from stress. Reliable changes in albumin require at least 2 to 3 weeks of nutritional supplementation. Since albumin is formed in the liver, diseases of the liver cause the hepatocytes to lose the ability to synthesize albumin. Prealbumin is another protein status indicator. Prealbumin's short half-life of 2 days and small serum pool allow small changes in nutritional status to be identified in a short time frame. Low prealbumin levels result from either inadequate nutrition or inflammatory stress. Prealbumin levels less than 5 mg/dL (50 mg/L) indicate severe protein and calorie malnutrition. Prealbumin should be used as an indicator of nutritional improvement and as a measure of how well nutritional interventions are working. Prealbumin can be measured once or twice per week and used as a sensitive monitor of nutritional progress.

Pneumo 109 A 32-year-old woman developed worsening hypoxemia and respiratory distress 12 days after she underwent myeloablative allogeneic stem cell transplantation for recurrent non-Hodgkin lymphoma. She had been intermittently febrile for the previous 5 days despite broad-spectrum antibiotic therapy. A chest radiograph showed bilateral alveolar infiltrates, which were not present on a radiograph taken 2 days before. Arterial blood gases were pH 7.48, PCO2 30 mm Hg, and PO2 56 mm Hg with the patient breathing 80% oxygen by face mask; oxygen saturation was 86%. Noninvasive positive-pressure ventilation (NPPV) was started. Now, 2 hours after she began NPPV, the patient is alert, speaks in three-word sentences, and is using accessory respiratory muscles. The temperature is 38.8 °C (101.8 °F), the blood pressure is 120/60 mm Hg, the pulse rate is 116/min, and the respiration rate is 34/min. There are coarse bilateral inspiratory crackles. Arterial blood gases with the patient breathing 100% oxygen by NPPV are pH 7.40, PCO2 40 mm Hg, and PO2 62 mm Hg; oxygen saturation is 91%. Which of the following is the most appropriate next step in the management of this patient? A Continue NPPV at current settings B Discontinue NPPV; start 100% oxygen by non-rebreather mask C Intubate and begin mechanical ventilation D Prescribe low-dose sedation

C Patients with acute respiratory failure who do not respond after a 1- to 2-hour trial of noninvasive positive-pressure ventilation should undergo elective intubation. Noninvasive positive-pressure ventilation (NPPV) reduces intubation rates and improves mortality rates in immunosuppressed patients with acute hypoxemic respiratory failure. Patients with acute respiratory failure whose condition does not stabilize within the first 1 to 2 hours of a NPPV trial should undergo elective intubation rather than incur the risk of subsequent respiratory arrest requiring emergent intubation. The patient's respiratory status remains tenuous 2 hours into a trial of NPPV as evidenced by severe hypoxemia, accessory respiratory muscle use, and an elevated respiration rate. This patient is already receiving 100% supplemental oxygen on NPPV; the elimination of positive end-expiratory pressure with removal of NPPV would place the patient at risk of further deterioration in oxygenation. This patient should remain on NPPV right up to the time of intubation because some limited data suggest that NPPV provides better oxygenation than bag-mask ventilation in hypoxemic patients being prepared for intubation. Sedation could precipitate respiratory arrest in this patient. Reduced level of consciousness is a relative contraindication to NPPV, and sedation should be used cautiously. In addition, there is little to suggest that excess ventilatory drive or anxiety is independently contributing to her persistent respiratory distress.

Pneumo 5 A 30-year-old man is evaluated for difficulty weaning from the ventilator. The patient was intubated 7 days ago for a severe exacerbation of asthma. Despite receiving a high-dose inhaled β-agonist; methylprednisolone, 60 mg/d; and aggressive sedation, he had persistent severe auto-positive end-expiratory pressure with elevated ventilator pressures. Therefore, a continuous infusion of vecuronium, a paralytic agent, was started and continued for 24 hours until his respiratory mechanics improved. Today, he underwent a ventilator weaning trial but became tachycardic and diaphoretic with a rapid shallow breathing index of 120. On physical examination, the patient is alert and responsive; vital signs are normal. There is minimal expiratory wheezing and otherwise normal vesicular breath sounds. He has flaccid weakness involving all extremities, including decreased bilateral hand grip strength. There is no rash. Routine laboratory studies reveal normal liver enzyme tests and renal function. Which of the following is the most likely cause for the patient's difficulty weaning from the ventilator? A Acute inflammatory demyelinating polyneuropathy (Guillain-Barré syndrome) B Churg-Strauss syndrome C Intensive care unit-acquired weakness D Prolonged neuromuscular blockade

C Patients with intensive care unit-acquired weakness have diffuse, flaccid weakness and often present with difficulty with ventilator weaning. Patients with intensive care unit (ICU)-acquired weakness have diffuse weakness and decreased muscle tone. The disorder may be first recognized in patients with unexplained difficulty weaning from the ventilator. ICU-acquired weakness is a term used to encompass critical-illness poly-neuropathy and critical-illness myopathy. Treatment with paralytic agents and systemic corticosteroids, as well as sepsis and immobilization, increase the risk of developing ICU-acquired weakness. Treatment is supportive, including discontinuation or reduction of corticosteroids, aggressive management of existing disorders, and physical rehabilitation. Acute inflammatory demyelinating polyneuropathy (Guillain-Barré syndrome) can also cause diffuse weakness, but in contrast to this patient, weakness is the presenting symptom. Furthermore, weakness associated with acute inflammatory demyelinating polyneuropathy typically develops gradually over a longer time course (1 to 2 weeks) and typically is preceded by an infection. Prolonged neuromuscular blockade is a condition in which the effect of paralytic agents, such as vecuronium, can persist for days after discontinuing the medication. However, this is rarely encountered and is caused by altered drug metabolism due to liver and/or renal dysfunction, neither of which this patient has. Churg-Strauss syndrome is associated with asthma and, in the vasculitic stage, can cause weakness. However, the syndrome most often manifests as mononeuritis multiplex rather than generalized weakness. Furthermore, this patient does not have other manifestations of vasculitis such as rash or renal dysfunction.

Pneumo 108 A 57-year-old man with sarcoidosis is evaluated for a 4-week history of progressive dyspnea. He also has fatigue, ankle swelling, lower extremity aching, and lack of appetite. The patient had sarcoidosis diagnosed 7 years ago. The disease has slowly progressed despite therapy. He has no other medical disorders, and his medications are methotrexate, 15 mg weekly; prednisone, 20 mg/d; folic acid; and trimethoprim-sulfamethoxazole. On physical examination, he is afebrile; blood pressure is 104/96 mm Hg, the pulse rate is 110/min, and the respiration rate is 28/min. Oxygen saturation is 90% with the patient breathing oxygen, 3 L/min by nasal cannula. Cardiac examination reveals a loud P2 and fixed splitting of S2; jugular venous distention is present. The chest is clear to auscultation. There is pitting edema at both ankles to the mid-shin. Chest radiograph, which is unchanged from 3 months ago, shows small lung volumes with fibrosis extending into the mid- and upper lungs bilaterally and bilateral hilar lymph node enlargement. Spirometry, which is also unchanged from 3 months ago, shows a moderate restrictive pattern. Which of the following is the most appropriate next diagnostic test for this patient? A Bronchoscopy with bronchoalveolar lavage B Pulmonary angiography C Transthoracic echocardiography D Ventilation/perfusion scan

C Pulmonary hypertension is a frequent cause of worsening dyspnea in patients with interstitial lung disease. There are many causes for worsening dyspnea in a patient with end-stage interstitial lung disease, but this patient's presentation of subacute worsening dyspnea with lower extremity swelling, fatigue, and anorexia suggests that he has developed pulmonary hypertension. Pulmonary hypertension is a frequent cause of worsening dyspnea in patients with interstitial lung disease, and a report of patients with sarcoidosis referred for lung transplantation demonstrated that 73.8% had pulmonary hypertension by right heart catheterization. Transthoracic echocardiography is a useful diagnostic test for suspected pulmonary hypertension. The test can assess for valvular disease, evaluate right atrial and ventricular enlargement and function, and estimate right ventricular systolic pressure. The diagnosis of pulmonary hypertension is usually confirmed by right-heart and pulmonary artery catheterization. However, this invasive test is not typically the first diagnostic test to evaluate pulmonary hypertension. This patient has no clinical or radiographic features to suggest community-acquired respiratory infection. Although he is immunosuppressed, he is treated with trimethoprim-sulfamethoxazole as prophylaxis against Pneumocystis jirovecii infection. Therefore, bronchoscopy with bronchoalveolar lavage should not be the next diagnostic test. Venous thromboembolic disease should always be considered in patients with dyspnea. However, this patient's dyspnea has been progressing over the past 4 weeks, a course that is not typical for pulmonary embolism. Finally, this patient has an existing condition that can account for the presence of pulmonary hypertension; therefore, ventilation/perfusion scan would be premature at this time.

Pneumo 45 An 18-year-old woman with a 4-month history of type 1 diabetes mellitus is evaluated in the emergency department for mild diabetic ketoacidosis. She developed flu-like symptoms 48 hours ago, and she did not administer her usual insulin therapy. She takes no other medications, has no known allergies, and has never been hospitalized before. The patient is examined, and therapy with insulin and fluids is started; an intravenous line and an indwelling urinary catheter are inserted. Shortly thereafter, the patient develops shortness of breath and chest tightness. On repeat examination, she now has facial flushing and bilateral wheezing. Her blood pressure has decreased from 120/90 mm Hg to 90/60 mm Hg, and her pulse rate has increased from 96/min to 120/min with a respiration rate of 30/min. There is no evidence of perioral or oral swelling or upper airway compromise. Which of the following is the most appropriate next step in the management of the patient? A Emergent cricothyroidotomy B Endotracheal intubation C Intramuscular epinephrine D Intravenous glucagon

C The most common causes of anaphylaxis are foods, insect venoms, latex, and medications. Systemic allergic reactions are usually of acute onset and can involve multiple organ systems. Death may occur from refractory bronchospasm, respiratory failure with upper airway obstruction, and cardiovascular collapse. The pathogenesis consists of an IgE-mediated hypersensitivity reaction. The most common causes are foods, insect venoms, latex, and medications. Common skin manifestations include flushing, pruritus, urticaria, and angioedema. However, 10% to 20% of patients do not have cutaneous signs, and up to 80% of patients with food-induced anaphylactic reactions do not have any skin changes. An "ABCD" approach is recommended including: * Airway: Evaluate for airway compromise and consider endotracheal intubation or emergent cricothyroidotomy; oxygen * Bronchospasm: Epinephrine intramuscular or subcutaneous (0.3-0.5 mL of 1:1000 solution); inhaled β-agonists * Circulatory collapse: Aggressive fluid resuscitation; continuous epinephrine infusion; vasopressor infusion * Drugs: Antihistamines; H1-antagonist; H2-antagonist; corticosteroids This patient likely had an anaphylactic reaction to latex, which is contained in gloves, drains, and catheters. Repeated exposure leads to a higher risk of anaphylaxis. Reactions to latex can vary from mild with irritant contact dermatitis to an acute hypersensitivity reaction. Patients with a known latex allergy should wear a MedicAlert® bracelet and have an epinephrine auto-injector (EpiPen®) quickly accessible. All patients with moderate to severe symptoms regardless of the inciting agent should be closely monitored for at least 12 hours in the hospital, preferably in the intensive care unit for a possible late recurrence. Most patients who receive epinephrine therapy will not require intubation or an emergent cricothyroidotomy. Glucagon is not indicated in the treatment of anaphylaxis unless the patient is taking a β-blocker and is not responding to first-line treatment.

Pneumo 80 A 73-year-old woman is admitted to the hospital in August for mental status changes. No family members are available and the patient lives alone. A friend states that the patient takes medications for Parkinson disease but that she ran out of medication 2 days ago. The emergency medical transport team found an empty bottle of levodopa/carbidopa at the house but no evidence of other medications. On physical examination, the patient is diaphoretic; the temperature is 39.0 °C (102.0 °F), the blood pressure is 170/120 mm Hg, and the pulse rate is 120/min. She is disoriented to time and place but follows simple commands, although there is neck stiffness as well as generalized muscular rigidity. Laboratory tests reveal an anion gap metabolic acidosis. Serum potassium is 5.8 meq/L (5.8 mmol/L), creatine kinase is 1250 U/L, and lactate dehydrogenase is 310 U/L. Which of the following is the most likely diagnosis? A Heat stroke B Malignant hyperthermia C Neuroleptic malignant syndrome D Serotonin syndrome

C The neuroleptic malignant syndrome can result from administration of neuroleptic agents and abrupt withdrawal of antiparkinsonian medications. The neuroleptic malignant syndrome can result from administration of neuroleptic agents, such as haloperidol. Acute and abrupt withdrawal of antiparkinsonian medications may also precipitate the syndrome and may be the cause for this patient's symptoms. The pathogenesis of this syndrome results from dopamine D2 receptor antagonism. Muscular rigidity and altered mental status occur early, followed by hyperthermia, hypertension, tremors, fever, dysphagia, diaphoresis, myoclonus, and autonomic dysfunction. Criteria for the diagnosis of neuroleptic malignant syndrome are based on clinical features, although elevations in serum potassium, creatine kinase, and lactate dehydrogenase levels and a metabolic acidosis are common. The mortality rate is 10% to 20%, although the rate is generally higher in patients who develop severe muscle necrosis and rhabdomyolysis. The first sign of serious heat stroke is the absence of sweating and warm and dry skin. This patient's diaphoresis makes heat stroke unlikely. Malignant hyperthermia is an inherited skeletal muscle disorder characterized by a hypermetabolic state precipitated by exposure to volatile inhalational anesthetics (halothane, isoflurane, enflurane, desflurane, sevoflurane) and the depolarizing muscle relaxants succinylcholine and decamethonium. Findings include sustained muscle contractions with skeletal muscle rigidity and masseter muscle spasm, tachycardia, hypercarbia, hypertension, hyperthermia, tachypnea, and cardiac arrhythmias. This patient's history does not include exposure to drugs responsible for malignant hyperthermia, making this diagnosis unlikely. The serotonin syndrome is caused by use of selective serotonin reuptake inhibitors. Distinguishing features include shivering, hyperreflexia, myoclonus, and ataxia. The diagnosis of serotonin syndrome is less likely in the absence of evidence that the patient was taking a selective serotonin reuptake inhibitor drug.

Pneumo 112 A 60-year-old woman is evaluated for a 2-year history of fatigue and daytime sleepiness and an 8-month history of dyspnea that occurs only during moderate exercise. She does not smoke cigarettes and works inside the home. She has hypertension, and her only medication is hydrochloro-thiazide. On physical examination, vital signs are normal. The patient is obese (BMI 33); the neck is short and thick (circumference 50.8 cm [20 in]), and the posterior airway is crowded. Jugular venous distention cannot be adequately assessed. There is increased intensity of P2 with fixed splitting of S2 and a grade 1-2/6 holosystolic murmur that increases with inspiration heard best along the left lower sternal border. Chest radiograph is normal. Spirometry, plethysmography, and arterial blood gases with the patient breathing ambient air are normal. Transthoracic echocardiography shows evidence of right ventricular hypertrophy and mild to moderate pulmonary hypertension. A ventilation/perfusion scan is normal. Which of the following is the most appropriate management for this patient? A Lung transplantation B Nocturnal continuous positive airway pressure therapy C Polysomnography D Right-heart catheterization

C The presence of pulmonary arterial hypertension should prompt an evaluation for associated conditions, such as obstructive sleep apnea and connective tissue diseases. This patient has evidence of mild to moderate pulmonary hypertension. Her body habitus and long-standing history of fatigue and daytime sleepiness raise the possibility of obstructive sleep apnea, a contributing factor or cause of the pulmonary hypertension. Therefore, a full polysomnography would be appropriate in this patient. Identification and treatment of coexisting obstructive sleep apnea may improve the outcome of pulmonary arterial hypertension. In addition to weight loss, continuous positive airway pressure therapy at nighttime improves quality of life, cognitive function, and symptoms of daytime sleepiness. Initiating therapy without a diagnostic evaluation is not indicated. Right-heart catheterization may be appropriate at some point in the evaluation of this patient, but noninvasive testing to rule out secondary causes of pulmonary hypertension should be done first. Although pulmonary arterial hypertension was at one time a relentless disease, many effective treatment options exist for this patient, and evaluation for lung transplantation would be inappropriate.

Pneumo 106 A 35-year-old man is evaluated for episodic wheezing, dyspnea, and cough. Two months ago he had an acute episode of cough, dyspnea, wheezing, and chest tightness within minutes of inhaling high concentrations of ammonia solution after an accidental spill at work. In the emergency department, the chest radiograph was normal, and he received inhaled bronchodilators and a brief course of oral corticosteroids. The patient does not have a history of asthma or allergies and is otherwise healthy and takes no medications. The physical examination is normal. Spirometry shows an FEV1 of 90% of predicted; FEV1/FVC ratio is 82%. Which of the following is the most appropriate next test in this patient's evaluation? A Bronchoscopy with biopsy B Inhalation challenge with chlorine C Methacholine challenge test D Spirometry before and after work

C The reactive airways dysfunction syndrome follows a single, accidental inhalation of high levels of a nonspecific respiratory irritant in patients who typically do not have a history of asthma; the diagnosis is confirmed by a positive methacholine challenge test. Many chemicals are irritants to the respiratory tract, affecting both the upper and lower airways. Asthma occurring after an acute, high-level exposure to an irritant is called the reactive airways dysfunction syndrome (RADS). Respiratory tract irritants reported to cause RADS include chlorine and its derivatives (hydrochloric acid, chlorine dioxide, phosgene), acids, ammonia, bromine, bleaches, isocyanates, formaldehyde, pyrolysis products contained in smoke, and aromatic hydrocarbons in sealants. RADS follows a single, accidental inhalation of high levels of a nonspecific respiratory irritant in patients who typically do not have a history of asthma. Within minutes of exposure, the patient develops cough, wheezing, dyspnea, and chest tightness. Symptoms persist even after the exposure has stopped and may last for years. The diagnosis is based on history and confirmed by a positive methacholine challenge test. Methacholine challenge testing can be done safely in patients with asthma provided that appropriate guidelines are followed and that the FEV1 is greater than 70% of predicted. Pharmacologic management is similar to that of other forms of asthma. Avoidance of nonspecific asthma triggers is often helpful. Exposure challenge with chlorine poses an unnecessary risk of severe attacks and would not be indicated in this patient. Histologic features of RADS include epithelial desquamation, submucosal inflammation, and thickening of the basement membrane. However, bronchial biopsy is not necessary for the diagnosis or management of the patient's symptoms. Spirometry before and after work is not helpful in cases of accidental exposure to irritants because these exposures are not likely to happen on a daily basis; therefore, these measurements, which are used in other forms of occupational asthma, are not useful in the diagnosis of RADS.

Pneumo 97 A 52-year-old man is evaluated for a 4-week history of nonproductive cough and fatigue without fever. He has lost 2.2 kg (5 lb) over the past several months; he has not traveled recently. The patient underwent resection of a rectal carcinoma 18 months ago followed by chemotherapy and radiation therapy. Chest radiograph at that time was normal. The patient has been otherwise healthy and takes no medications. His father died of colon cancer. On physical examination, the temperature is 36.6 °C (97.9 °F), the pulse rate is 62/min, the respiration rate is 18/min, the blood pressure is 126/70 mm Hg, and the BMI is 24. The lungs are clear, the abdomen is soft with a well-healed surgical scar, and there is no lymphadenopathy. Laboratory studies, including serum carcinoembryonic antigen (CEA) level, are normal; CEA level was normal at the time of resection. Chest radiograph shows multiple pulmonary nodules. Contrast-enhanced CT scan of the chest shows five nodules ranging from 1.0 to 1.8 cm in diameter; four are in the right lung (one in the upper lobe, three in the lower lobe) and one is in the left lower lobe. CT scan of the abdomen shows postoperative changes but is otherwise normal. Which of the following is the most appropriate next step in the management of this patient? A Bronchoscopy and biopsy of a nodule B Positron emission tomography-CT (PET-CT) C Transthoracic needle aspiration of a nodule D Video-assisted thoracoscopic resection of the nodules

C Transthoracic needle aspiration has high yield for evaluation of pulmonary nodules 1 to 2 cm in diameter. This patient has multiple pulmonary nodules, and metastatic disease would lead the differential diagnosis. A CT-guided needle biopsy would likely establish the diagnosis and would have significantly higher yield for nodules of this size than would bronchoscopy. There are reported yields of about 90% for nodules of 1 to 2 cm with transthoracic needle aspiration and less that 50% for bronchoscopy. Video-assisted thoracoscopic nodule resection would also establish the diagnosis but by a more invasive means than needle aspiration. Because of the number and bilateral nature of the nodules, primary resection would not be pursued for treatment. Positron emission tomography-CT would likely show these nodules to be 18F-fluorodeoxyglucose (FDG)-avid, but this finding would not obviate the need for tissue diagnosis or alter the stage.

Pneumo 62 A 75-year-old woman with long-standing asthma is evaluated for palpitations, nausea, and vomiting of 3 days' duration. Her asthma treatment regimen includes low-dose inhaled corticosteroids, albuterol as needed, and theophylline; her most recent serum theophylline level was 9 µg/mL (49.9 µmol/L). A week ago, she was evaluated at a walk-in clinic for productive cough and sore throat, and therapy with ciprofloxacin was started. The cough improved, but soon afterward her current symptoms developed. Which of the following is the most appropriate initial step in the management of this patient? A Stop the inhaled corticosteroid B Stop albuterol C Stop theophylline D Measure serum amylase and lipase

C Various medications influence theophylline metabolism in the liver, leading to increased serum levels and potential toxicity (for example, ciprofloxacin, cimetidine, erythromycin, allopurinol, and zileuton) or decreased levels (for example, rifampin, phenobarbital, and ethanol). This patient requires a thorough physical examination and diagnostic studies, including electrocardiography, a comprehensive metabolic profile, and measurement of the serum theophylline level. She may require urgent hospitalization. The initial step should be to stop theophylline. Various medications influence theophylline metabolism in the liver, leading to increased serum levels (for example, ciprofloxacin, cimetidine, erythromycin, allopurinol, and zileuton) or decreased levels (for example, rifampin, phenobarbital, and ethanol). Ciprofloxacin can increase the serum theophylline level several-fold, potentially resulting in theophylline toxicity. Theophylline clearance is also decreased in the elderly and in those with heart failure or liver disease. The target serum theophylline level should be between 8 and 12 µg/mL (44.4 and 66.6 µmol/L) to limit its side effects. Many patients have side effects from theophylline (tremor, palpitations) at high therapeutic levels (15 to 20 µg/mL [83.2 to 111 µmol/L]). Higher levels lead to nausea, vomiting, arrhythmia, and seizures and can result in death. Despite theophylline's ease of use, low cost, and mild anti-inflammatory activity, its regular use in the management of asthma is not recommended because of its limited bronchodilatory activity and narrow therapeutic range. Local side effects of inhaled corticosteroids include oral candidiasis and dysphonia. Systemic effects include suppression of the hypothalamic-adrenal axis, reduced growth velocity in children, osteopenia, skin thinning, and increased risk for cataracts or glaucoma. Common adverse effects of inhaled β-agonists include tachycardia, palpitations, tremors, and hypokalemia. Concomitant use of a fluoroquinolone antibiotic does not increase the incidence of adverse effects from inhaled corticosteroids or β-agonists, and stopping these medications could significantly worsen the patient's asthma. Drug-induced pancreatitis is relatively uncommon and difficult to substantiate but seems not to be related to the use of commonly prescribed asthma control medications or fluoroquinolone antibiotics. Finally, pancreatitis is typically associated with abdominal pain in addition to nausea and vomiting.

Pneumo 101 A 25-year-old woman is evaluated for recurrent episodes of acute dyspnea associated with rapid breathing, tightness in her chest and throat, voice changes, and inspiratory difficulty. When she has these episodes she feels that she "can't get enough air." The symptoms typically start and resolve abruptly, and treatment with albuterol provides only minimal relief. On physical examination, her vital signs are normal. Lungs are clear. Spirometry shows an FEV1 of 2.8 L (88% of predicted) and FEV1/FVC ratio of 80%. Which of the following is the most appropriate next diagnostic step? A Chest radiography B CT scan of the neck C Flow volume loops D Thyroid function tests

C Vocal cord dysfunction is diagnosed by flow volume loops showing an inspiratory cut-off in flow with relatively preserved expiratory flow indicative of an extrathoracic variable obstruction. Vocal cord dysfunction (VCD) can mimic asthma. However, in contrast to asthma, the symptoms often begin and end abruptly and do not usually respond to inhaled β-agonists. Symptoms often include throat or neck discomfort, inspiratory wheezing, and anxiety. Many affected patients have asthma as well as VCD, thus complicating the diagnosis. VCD can occur during exercise, in which case it must be distinguished from exercise-induced asthma. An important difference between asthma and VCD is that airflow limitation in asthma is mainly during expiration, whereas in VCD the limitation occurs mainly during inspiration. Flow volume loops document the inspiratory and expiratory flow by recording the flow while the patient inhales as deeply as possible and then exhales as much as possible. An inspiratory cut-off in flow (flattening of the inspiratory portion of the flow loop) with relatively preserved expiratory flow indicates an extrathoracic variable obstruction that is typical in patients with VCD. However, flow volume loops may be normal when the patient is asymptomatic between episodes. Another confirmatory test for VCD is laryngoscopy at the time of symptoms that reveals abnormal adduction of the vocal cords on inspiration. Management should target predisposing factors, mainly postnasal drip and gastroesophageal reflux disease. Even after controlling these problems, many patients continue to have episodes and require additional approaches, including speech therapy, relaxation techniques, and antianxiety medications. Chest radiograph, CT scan of the neck, and thyroid function tests would not add any additional diagnostic information in this patient with VCD.

Pneumo 37 A 36-year-old man is evaluated for a 1-year history of progressive shortness of breath; his wife has noticed that he has a dry cough and wheezing when he returns from work. The patient has worked as an automobile painter for the past 4 years. He has a 5-pack-year history of cigarette smoking, but quit smoking 8 years ago. He has no history of allergic disease. On physical examination, vital signs and review of systems are normal. Chest radiograph is normal. Pulmonary function testing shows: FEV1 52% of predicted FVC 83% of predicted FEV1/FVC ratio 47% DLCO 85% of predicted Spirometry after a bronchodilator shows: FEV1 83% of predicted FVC 100% of predicted FEV1/FVC ratio 62% Which of the following would be an appropriate next step in the evaluation of this patient? A Chest CT scan B Methacholine challenge test C Obtaining a detailed description of his current job tasks D Skin testing to common aeroallergens

C Reviewing a patient's occupational exposures, the job process and tasks, and substances involved in the patient's activities is necessary for an evaluation for possible occupational asthma. This patient likely has isocyanate-induced asthma caused by exposure to isocyanate-containing automobile paints. The most helpful piece of information to support the diagnosis of occupational asthma in this patient is an occupational history that substantiates direct or bystander exposure to isocyanate-containing paints with associated symptoms development. Although this man's job title may suggest such exposure, it is important to review the patient's daily job tasks to ascertain whether he is actually at risk for direct or bystander exposure to an isocyanate-containing product. The significant reversible airflow limitation on spirometry in this patient is already compatible with a diagnosis of asthma, and a confirmatory methacholine challenge test is not necessary and potentially dangerous by provoking bronchospasm. CT scan of the chest would be indicated in a patient with possible parenchymal disease; this patient has obstructive airways disease. Skin prick testing to common aeroallergens would determine whether the patient is atopic; however, atopy is not a risk factor for the development of asthma from exposure to low-molecular-weight substances such as isocyanates. Review of Material Safety Data Sheets from the patient's workplace can be important in establishing a diagnosis of work-related illness, but only when reviewed with a clear understanding of what the patient does and to which products he is exposed.

Pneumo 84 A 64-year-old man with a history of chronic obstructive pulmonary disease is evaluated in the emergency department for increased dyspnea over the past 48 hours. There is no change in his baseline production of white sputum, but he has increased nasal congestion and sore throat. His medications include inhaled tiotropium, combination fluticasone and salmeterol, and albuterol. Therapy with methylprednisolone, 60 mg intravenously, and inhaled albuterol and ipratropium bromide is started. The patient is alert but in mild respiratory distress. The temperature is 38.6 °C (101.5 °F), the blood pressure is 150/90 mm Hg, the pulse rate is 108/min, and the respiration rate is 30/min. Oxygen saturation with the patient breathing ambient air is 90%. Breath sounds are diffusely decreased with bilateral expiratory wheezes; he is using accessory muscles to breathe. With the patient breathing oxygen, 2 L/min by nasal cannula, arterial blood gases are pH 7.27, PCO2 60 mm Hg, and PO2 62 mm Hg; oxygen saturation is 91%. Which of the following is the most appropriate next step in the management of this patient? A Increase supplemental oxygen to 5 L/min B Intubate and begin mechanical ventilation C Start aminophylline infusion D Start noninvasive positive-pressure ventilation

D Noninvasive positive-pressure ventilation should be initiated early in the course of moderate or severe exacerbations of chronic obstructive pulmonary disease unless there is a specific contraindication to use of non-invasive ventilation. The patient is having a moderate to severe exacerbation of chronic obstructive pulmonary disease (COPD) and should be placed on noninvasive positive-pressure ventilation (NPPV). A landmark study found that NPPV reduced the need for intubation, the length of hospital stay, and the mortality rate in such patients. Suitable candidates for NPPV include patients with moderate to severe dyspnea, use of accessory respiratory muscles, respiration rate greater than 25/min, and pH less than 7.35 with PCO2 greater than 45 mm Hg. Contraindications to NPPV include impending respiratory arrest, cardiovascular instability, altered mental status, high aspiration risk, production of copious secretions, and extreme obesity, as well as surgery, trauma, or deformity of the face or upper airway. Intubation is inappropriate because the patient is not in respiratory arrest and is a suitable candidate for NPPV. However, if the patient's condition deteriorates or does not improve after 1 to 2 hours of NPPV, intubation should be considered. Most patients with exacerbations of COPD are usually easily oxygenated on low levels of inspired oxygen. Excessive oxygen supplementation can worsen carbon dioxide retention during a COPD exacerbation. Therefore, oxygen should be titrated to maintain a saturation of approximately 90%; increasing the nasal oxygen to 5 L/min is not indicated at this time. Methylxanthines are generally not recommended for the treatment of acute exacerbations of COPD because they are not more effective than inhaled bronchodilators and corticosteroid therapy but can cause nausea and vomiting.

Pneumo 10 A 72-year-old woman is evaluated for fatigue and decreased exercise capacity. The patient has severe chronic obstructive pulmonary disease, which was first diagnosed 10 years ago, and was hospitalized for her second exacerbation 1 month ago. She is a former smoker, having stopped smoking 5 years ago. She has no other significant medical problems, and her medications are albuterol as needed, an inhaled corticosteroid, a long-acting bronchodilator, and oxygen, 2 L/min by nasal cannula. On physical examination, vital signs are normal. Breath sounds are decreased, and there is 1+ bilateral pitting edema. Spirometry done 1 month ago showed an FEV1 of 28% of predicted, and blood gases measured at that time (on supplemental oxygen) showed pH 7.41, PCO2 43 mm Hg, and PO2 64 mm Hg; DLCo is 30% of predicted. There is no nocturnal oxygen desaturation. Chest radiograph at this time shows hyperinflation. CT scan of the chest shows homogeneous distribution of emphysema. Which of the following would be the most appropriate management for this patient? A Lung transplantation B Lung volume reduction surgery C Nocturnal assisted ventilation D Pulmonary rehabilitation

D Pulmonary rehabilitation in patients with advanced lung disease can increase exercise capacity, decrease dyspnea, improve quality of life, and decrease health care utilization. This patient who is on maximum medical treatment for chronic obstructive pulmonary disease (COPD) and is still symptomatic would benefit from pulmonary rehabilitation. Comprehensive pulmonary rehabilitation includes patient education, exercise training, psychosocial support, and nutritional intervention as well as the evaluation for oxygen supplementation. Referral should be considered for any patient with chronic respiratory disease who remains symptomatic or has decreased functional status despite otherwise optimal medical therapy. Pulmonary rehabilitation increases exercise capacity, reduces dyspnea, improves quality of life, and decreases health care utilization. Reimbursement for pulmonary rehabilitation treatment remains an impediment to its widespread use. The effect of lung volume reduction surgery is larger in patients with predominantly nonhomogeneous upper-lobe disease and limited exercise performance after rehabilitation. The ideal candidate should have an FEV1 between 20% and 35% of predicted, the DLCO no lower than 20% of predicted, hyperinflation, and limited comorbidities. There is no indication for nocturnal assisted ventilation because she does not have daytime hypercapnia and worsening oxygen desaturation during sleep. Lung transplantation should be considered in patients hospitalized with COPD exacerbation complicated by hypercapnia (PCO2 greater than 50 mm Hg) and patients with FEV1 not exceeding 20% of predicted and either homogeneous disease on high-resolution CT scan or DLCO less than 20% of predicted who are at high risk of death after lung volume reduction surgery. Lung transplantation is, therefore, not an option for this patient.

Pneumo 21 A 35-year-old woman is evaluated in the hospital for chest pain and dyspnea 1 day after vaginal delivery of her second child. She had an uncomplicated pregnancy but a prolonged labor. On physical examination, the temperature is 37.0 °C (98.6 °F), the blood pressure is 100/60 mm Hg, the pulse rate is 115/min, and the respiration rate is 22/min. The lungs are clear, heart sounds are normal, and there is no evidence of bleeding on pelvic examination. Complete blood count on admission revealed a hematocrit of 34% and a platelet count of 150,000/µL (150 × 109/L). Chest radiograph is normal. Ventilation/perfusion scan shows mismatched perfusion defects in 20% of her lung volume. Which of the following would be an acceptable therapy for this patient? A Inferior vena cava filter B Intravenous argatroban C Intravenous low-molecular-weight heparin D Subcutaneous unfractionated heparin

D Acute pulmonary embolism can be treated initially with subcutaneous unfractionated heparin, low-molecular-weight heparins, or fondaparinux without the need for dosage adjustment. This patient has had an acute pulmonary embolism 1 day post partum. The patient has no evidence of active bleeding, and there is no increased risk for bleeding from anticoagulation. Subcutaneous administration of unfractionated heparin, low-molecular-weight heparins, and fondaparinux are all safe and effective for the treatment of acute pulmonary embolism. A recent clinical trial showed that high-dose subcutaneous unfractionated heparin, administered without dose adjustment guided by the activated partial thromboplastin time, was as safe and effective as low-molecular-weight heparin administered in the same fashion. Intravenous argatroban, a direct thrombin inhibitor, might be useful in the setting of heparin-induced thrombocytopenia. However, the patient's platelet count is normal. Monitoring of the platelet count would be appropriate after initiating either unfractionated or low-molecular-weight heparin, but the patient's current platelet count is not a contraindication for either drug; therefore, there is no indication to begin treatment with argatroban. Neither low-molecular-weight heparins nor fondaparinux has been evaluated in large clinical trials for intravenous use. Therefore, although it may be theoretically possible to use these agents intravenously, appropriate dosing and monitoring guidelines have not been validated. There are four generally accepted indications for placement of an inferior vena cava filter: (1) absolute contraindication to anticoagulation (for example, active bleeding); (2) recurrent pulmonary embolism despite adequate anticoagulation therapy; (3) bleeding complication of anticoagulation therapy; and (4) hemodynamic or respiratory compromise severe enough that a subsequent pulmonary embolism might be lethal. This patient has no indication for an inferior vena cava filter.

Pneumo 48 A 65-year-old man is evaluated for a 6-month history of fatigue and decreased exercise capacity. The patient has severe chronic obstructive pulmonary disease with frequent exacerbations and has been hospitalized three times within the past 2 years for exacerbations of his disease. He recently stopped his participation in a pulmonary rehabilitation program. He also has hypertension, and his medications are albuterol as needed; salmeterol; tiotropium; theophylline; an inhaled corticosteroid; oxygen, 2 L/min by nasal cannula; and hydrochlorothiazide. On physical examination, he is afebrile; the blood pressure is 130/70 mm Hg, the pulse rate is 84/min, the respiration rate is 16/min, and the BMI is 24. Heart and breath sounds are diminished; there is no jugular venous distention, hepatomegaly, or edema. Spirometry shows an FEV1 of 35% of predicted and an FEV1/FVC ratio of 45%. Oxygen saturation is 92% (PO2 is 65 mm Hg) on supplemental oxygen. Laboratory studies: Hematocrit 40% Albumin 3.6 g/dL (36 g/L) Creatinine 1.0 mg/dL (88.4 µmol/L) Thyroid-stimulating hormone 3.0 µU/mL (3.0 mU/L) Chest radiograph shows hyperinflation but no infiltrates. Which of the following would be the most appropriate management for this patient? A Increase nasal oxygen B Refer for lung transplantation C Refer for lung volume reduction surgery D Screen for depression

D Anxiety and depression are common in patients with severe chronic obstructive pulmonary disease. Anxiety and depression are common in patients with chronic obstructive pulmonary disease (COPD). Dyspnea is a contributing factor to anxiety. Depression may be precipitated by the loss and grief associated with the disability of COPD. Depression also affects adherence to therapy. It is not easy to diagnose depression in patients with COPD because of overlapping symptoms. Dyspnea, fatigue, and altered sleep can occur both in depression and COPD. However, patients should be asked whether they have experienced depressed mood and anhedonia during the last month using the two-question model: "Over the past 2 weeks have you felt down, depressed, hopeless?" and "Over the past 2 weeks have you felt little interest or pleasure in doing things?" A positive response to either question should be followed-up with more detailed questioning to establish the diagnosis of a mood disorder. This patient with several recent exacerbations and hospitalizations could benefit from pulmonary rehabilitation and has discontinued his participation. Therapy with antidepressants and exercise training are often effective in treating anxiety and depression and improving quality of life. The patient's oxygenation is adequate (oxygen saturation ≥88%) and, therefore, increasing the nasal oxygen is not indicated. In highly selected patients, lung volume reduction surgery and lung transplantation may improve lung function and quality of life but should not be considered in this patient before screening for depression.

Pneumo 74 A 46-year-old man is evaluated after being found unresponsive in his hospital bed on day 3 after a right total knee replacement. The patient has a history of asthma, hypertension, and untreated obstructive sleep apnea. His medications are inhaled combination fluticasone and salmeterol, inhaled albuterol, and lisinopril. In the hospital, he was given fondaparinux for thromboembolism prophylaxis and morphine for pain. The patient is difficult to arouse. He is morbidly obese. The temperature is 35.5 °C (96.0 °F), the pulse rate is 100/min, the respiration rate is 8/min, and the blood pressure is 130/80 mm Hg. Oxygen saturation with the patient breathing ambient air is less than 80%. He has decreased bibasilar breath sounds; heart sounds are normal. Neurologic examination reveals no focal abnormalities. Arterial blood gases are pH 7.08, PCO2 80 mm Hg, and PO2 40 mm Hg; 100% oxygen by face mask is started. Which of the following is the most appropriate next step in the management of this patient? A Albuterol by nebulizer B Continuous positive airway pressure at 10 cm H2O C Intravenous heparin and stat chest CT angiography D Intravenous naloxone

D Continuous naloxone infusion is used to treat opioid intoxication. The patient's acute ventilatory failure is most consistent with hypoventilation resulting from opioid intoxication. In pure hypoventilation, the alveolar-arterial difference is normal. The alveolar-arterial difference equals [(PiO2 - PCO2/0.8) - PO2] where PiO2 is the partial pressure of oxygen in inspired air. PiO2 equals the atmospheric pressure (760 mm Hg at sea level) minus the partial pressure of water (47 mm Hg at 37.0 °C) times the fraction of inspired oxygen (0.21). This patient's alveolar-arterial difference [(150 - 80/0.8) - 40] is 10 mm Hg, which is normal for the patient. The normal alveolar-arterial gradient increases with age and can be estimated in patients breathing ambient air using the following equation: [2.5 + (0.21 × age in years)]. Improved alertness after administration of naloxone confirms opioid intoxication. Naloxone has a short half-life and is typically given as a continuous intravenous infusion. If the response to naloxone is inadequate, endotracheal intubation would be appropriate. Hypoxemia in asthma exacerbations is caused by ventilation/perfusion mismatch, which is associated with elevation of the alveolar-arterial difference. Patients with an asthma exacerbation typically present with a respiratory alkalosis; respiratory arrest without first coming to medical attention would be unusual in a hospitalized patient. Continuous positive airway pressure would not likely offer sufficient ventilation in this hypoventilating patient. The bilevel pressures used in noninvasive positive pressure ventilation might provide adequate ventilation but would be high risk in this patient with a depressed level of consciousness. Patients with a pulmonary embolism typically present with respiratory alkalosis and elevated alveolar-arterial difference, which is not consistent with this patient's findings. Therefore, intravenous heparin and CT angiography are not indicated.

Pneumo 103 A 44-year-old man is evaluated for a 4-month history of progressive dyspnea and cough. The patient is otherwise healthy and takes no medications. He has smoked 2 packs of cigarettes a day for 20 years. On physical examination, vital signs are normal. Cardiopulmonary examination is normal. Pulmonary function tests show a mixed obstructive-restrictive defect. High-resolution CT of the chest shows bibasilar reticular or reticulonodular abnormalities in a centrilobular distribution with diffuse areas of ground-glass opacification. Which of the following is the most likely diagnosis? A Cryptogenic organizing pneumonia B Idiopathic pulmonary fibrosis C Nonspecific interstitial pneumonia D Respiratory bronchiolitis interstitial lung disease

D Diagnosis of respiratory bronchiolitis interstitial lung disease requires the clinical context of a patient with a significant past or current smoking history. Although the current American Thoracic Society/European Respiratory Society classification of interstitial lung disease (ILD) lists respiratory bronchiolitis interstitial lung disease (RB-ILD) as an idiopathic ILD, it is causally linked to smoking. Diagnosis of RB-ILD requires the clinical context of a patient with a significant smoking history. Respiratory bronchiolitis, which is also called smoker's bronchiolitis, is common among smokers and is thought to be responsible for the reduction in air flow in young, otherwise healthy smokers. The histopathologic diagnosis of respiratory bronchiolitis is made when pigmented macrophages accumulate in the bronchioles and peribronchiolar alveolar spaces. The difference between respiratory bronchiolitis and RB-ILD is poorly defined in the literature, but the diagnosis of RB-ILD is appropriate when patients have clinical and radiographic evidence of ILD in addition to airway disease. Pulmonary function testing typically shows a mixed obstructive-restrictive pattern with a slightly reduced diffusing capacity for carbon monoxide but in some cases, testing may be normal or associated with only an increase in the residual volume. Histopathologically, RB-ILD is usually associated with a greater degree of peribronchiolar fibrosis if lung biopsy is obtained. The most important treatment intervention is smoking cessation. Although previous reports suggest clinical improvement can be expected in patients who quit smoking, a recent study by Portnoy and colleagues reported clinical improvement in less than one third of RB-ILD patients who quit smoking. Although smoking is associated with an increased relative risk for other ILDs, including idiopathic pulmonary fibrosis, RB-ILD is causally linked to inhalation of tobacco smoke, whereas cryptogenic organizing pneumonia, idiopathic pulmonary fibrosis, and nonspecific interstitial pneumonia are not. Finally, these disorders are typically associated with restrictive physiology without evidence of airflow obstruction, which is an important point in distinguishing these diseases from RB-ILD.

Pneumo 39 A 57-year-old man is evaluated after a chest radiograph taken in a preoperative assessment for a knee replacement showed a 1.0-cm nodule in the right lower lobe of the lung. The patient lives in Montana and has not traveled recently. He does not recall ever having been exposed to tuberculosis or having been tested for the disease. His most recent chest radiograph was 10 years ago; the result was normal, and the radiograph is no longer available. About 6 months ago he had abdominal pain that was evaluated with an abdominal CT scan, and the pain has since resolved. The patient has a 20-pack-year history of cigarette smoking but quit 10 years ago. He is otherwise healthy. On physical examination, vital signs are normal, lungs are clear, and there is no lymphadenopathy. Which of the following is the most appropriate next step in the management of this patient? A 18F-fluorodeoxyglucose and positron emission tomography scan (FDG-PET) B MRI of the chest C Repeat CT scan in 3 months D Review of lung images from CT scan of the abdomen E Thin-section CT scan of the chest

D Evaluation of a pulmonary nodule always begins by reviewing prior pertinent imaging studies. Evaluation of a pulmonary nodule should always begin by review of any previous images. The CT scan of the abdomen may show that the nodule was present but missed: evidence of growth would indicate a high likelihood of malignancy; evidence of shrinkage would indicate a high likelihood of a benign lesion. If the nodule was not present or not included on the previous CT images, CT chest evaluation is appropriate, and if a dynamic CT enhancement study shows no significant enhancement, then the nodule is likely benign. 18F-fluorodeoxyglucose positron emission tomography (FDG-PET) is generally done after CT of the chest. Lung cancers are metabolically active and take up FDG avidly, whereas benign pulmonary nodules do not. FDG-PET is most helpful in patients with a nodule 1 cm or larger and an intermediate probability of malignancy. MRI has a limited role in the evaluation of a pulmonary nodule; it is more expensive than CT, and CT can diagnose and stage lung cancer in a single examination. This patient is at significant risk for lung cancer because of his smoking history and the size of the nodule; therefore, more information to suggest it is benign (such as its presence unchanged on the old film, no enhancement on dynamic CT, or negative FDG-PET) would be desired before choosing to follow-up with CT in 3 months. If the nodule shows significant uptake on FDG-PET, CT-guided biopsy or removal should be done.

Pneumo 32 A 20-year-old male college student is evaluated for a 3-year history of persistent daytime sleepiness. He snores loudly but has had no witnessed apneas or cataplexy. He has occasional episodes of sleep paralysis in which he cannot move for about a minute after awakening from sleep. He typically goes to bed at 11:30 PM on weekdays and at 1:00 AM on weekends. He falls asleep easily, sleeps uneventfully, and awakens at about 6:00 AM on weekdays and 11:00 AM on weekends. His medical history includes depression diagnosed 1 year ago for which he takes a selective serotonin reuptake inhibitor. He drinks three or four caffeinated drinks a day. Which of the following is the most appropriate management for this patient? A Order thyroid function tests B Perform a multiple sleep latency test C Prescribe modafinil D Recommend longer nighttime sleep E Refer for polysomnography

D Excessive daytime sleepiness is defined by a persistent or recurrent inability to both achieve and sustain alertness required to accomplish the tasks of daily living and is most commonly secondary to insufficient sleep. Evaluation of excessive daytime sleepiness consists of a careful history with inquiries into sleep duration and quality, daytime consequences of sleepiness, medical disorders, and medication or substance use. Inadequate sleep duration is the most common cause of excessive daytime sleepiness and it can cause sleep paralysis. Sleep paralysis is a complete inability to move for 1 or 2 minutes immediately after awakening. Although sleep paralysis is one of the clinical characteristics of narcolepsy, it can occur in other conditions, including sleep deprivation. Although the adequate duration of sleep varies, most persons require about 8 hours of sleep each night. Patients with sleep deprivation generally describe a habitual sleep duration that is shorter than normal for most age-matched persons, and there may be a significant difference in nighttime sleep duration during weekends, with a longer "rescue" sleep. A trial of longer nighttime sleep duration is often all that is necessary to improve excessive daytime sleepiness. Thyroid disease can cause central sleep apnea, which is more common in older persons and is very unlikely in this young man. Therefore, thyroid function tests would be unlikely to help in the management of his daytime sleepiness. Stimulant agents, such as modafinil, should not be used as a substitute for getting sufficient sleep. Sleep latency is the duration from getting into bed to the onset of sleep and is an objective measure of sleepiness. Polysomnography involves the measurement of several physiologic variables during sleep and is useful in the evaluation of excessive sleepiness. Neither polysomnography nor a multiple sleep latency test is required for the diagnosis of insufficient sleep syndrome.

Pneumo 14 A 60-year-old woman is evaluated 3 weeks after starting continuous positive airway pressure (CPAP) therapy for obstructive sleep apnea. The patient was initially evaluated for excessive sleepiness, and obstructive sleep apnea was diagnosed based on results of polysomnography. It was determined that CPAP at a pressure of 14 cm H2O normalized respiration and oxygen saturation during sleep. She was prescribed CPAP at this pressure along with heated humidification administered via a nasal mask. She has been using CPAP, but she is still often sleepy during the day. She has a history of hypertension and osteoarthritis, and her medications are hydrochlorothiazide and ibuprofen. She does not smoke or drink alcohol. On physical examination, she is afebrile; the blood pressure is 145/85 mm Hg, and the BMI is 36.5. She has a slightly receding (zurückgetretenes Kinnd) jaw; otherwise, physical features are unremarkable. Which of the following is the most appropriate next step in the management of this patient? A Order a multiple sleep latency test B Prescribe hormone replacement therapy C Prescribe modafinil D Review CPAP compliance

D Excessive sleepiness that persists despite positive airway pressure therapy may be due to poor adherence with treatment. Obstructive sleep apnea is associated with significant, even life-threatening complications, and CPAP is effective therapy. CPAP therapy reduces the number of apneas and hypopneas and improves sleep architecture, hypertension, and quality of life. Excessive sleepiness that persists despite CPAP therapy may be due to poor adherence with treatment, insufficient sleep duration, presence of coexisting sleep disorders, surreptitious use of sedating medications, or mood disorders. Adherence to CPAP therapy is often suboptimal, and many patients remain symptomatic and report either not being able to tolerate the device or using it intermittently. Verifying proper use and adherence is important before CPAP therapy is discontinued or additional studies are ordered or therapy prescribed. Many CPAP devices have data cards or meters that monitor use, and they can be evaluated to determine whether CPAP use is suboptimal. A multiple sleep latency test can be performed if daytime sleepiness persists in a patient who is consistently able to use CPAP set at an optimal pressure; in this patient it will objectively confirm her complaints of sleepiness but will not aid in identifying its cause. Stimulant and wake-promoting agents, such as caffeine or modafinil, may be used as adjunct therapy to improve residual daytime sleepiness in patients receiving optimal CPAP therapy, but should not be used as a substitute for proper CPAP use. Similarly, hormone replacement therapy is not indicated as sole therapy for postmenopausal women with obstructive sleep apnea.

Pneumo 78 A 78-year-old woman is evaluated for a 6-month history of fatigue and an unintentional 9.0-kg (20-lb) weight loss. She has occasional cough but no significant shortness of breath. She has never smoked and takes no medications. On physical examination, the temperature is 36.8 °C (98.0 °F), the blood pressure is 116/68 mm Hg, the pulse rate is 72/min, the respiration rate is 14/min, and the BMI is 31. There is a palpable 1.5-cm supraclavicular lymph node on the left. The skin is normal, the lungs are clear, and the abdomen is soft and without organomegaly. Complete blood count, liver enzymes, and serum calcium are normal. Chest radiograph shows a 5-cm left lung mass with left hilar lymphadenopathy. CT scan shows a 5-cm left upper lobe mass with left hilar lymphadenopathy and normal-sized mediastinal lymph nodes. Which of the following is the most appropriate management for this patient? A Bronchoscopy with biopsy of the mass B CT-guided transthoracic needle biopsy of the mass C Positron emission tomography-CT (PET-CT) D Supraclavicular lymph node biopsy E Surgical referral for mediastinoscopy

D In the evaluation of possible lung cancer, obtaining a tissue diagnosis and staging for lung cancer should be done simultaneously. In the evaluation of a patient with suspected lung cancer, obtaining a tissue diagnosis is critical for treatment planning and determining prognosis. Staging the cancer and determining whether the patient is a candidate for resection are also important parts of the evaluation. In this patient, determining whether the supraclavicular node is involved with non-small cell cancer should be done, and the next step in the evaluation would be to sample the lymph node; this would likely establish both a diagnosis and that the patient has advanced unresectable disease. As the clinical stage is already suggesting an advanced stage of disease, positron emission tomography (PET)-CT would not be needed for staging if the supraclavicular node is positive. A positive PET-CT in the supraclavicular lymph nodes would not obviate the need for node sampling. Biopsy of the mass or hilar lymph nodes by either CT guidance or bronchoscopy would establish the diagnosis but not the stage and would not determine resectability.

Pneumo 54 A 69-year-old man is evaluated in the hospital for fever, progressive cough productive of purulent sputum, and increasing dyspnea. The patient was diagnosed with severe (GOLD stage III) chronic obstructive pulmonary disease 3 years ago. He smokes a pack of cigarettes a day. His medical history also includes hypertension and hypercholesterolemia, and his medications are albuterol as needed, tiotropium, simvastatin, hydrochlorothiazide, and aspirin. On physical examination, the patient is in mild respiratory distress and is sitting up in bed. The temperature is 38.2 °C (101.0 °F), the blood pressure is 110/70 mm Hg, the pulse rate is 104/min, and the respiration rate is 20/min; BMI is 25. The oxygen saturation is 88% with the patient breathing ambient air and 93% on oxygen, 2 L/min by nasal cannula. There are bilateral expiratory wheezes but no digital clubbing or peripheral edema. Chest radiograph shows hyperinflation without infiltrates. Arterial blood gases while the patient was breathing ambient air are pH 7.40, PCO2 41 mm Hg, and PO2 53 mm Hg. Spirometry done 2 months ago showed an FEV1 of 45% of predicted and an FEV1/FVC ratio of 52%. Therapy with nebulized albuterol and ipratropium is begun. Which of the following additional therapy would be most appropriate for this patient? A Inhaled corticosteroids B Inhaled salmeterol C Intravenous theophylline and antibiotics D Intravenous corticosteroids and antibiotics E Noninvasive positive-pressure ventilation

D Inhaled short-acting bronchodilators (β2-agonists and anticholinergic agents), systemic corticosteroids, and antibiotics are the mainstay of treatment of exacerbations of chronic obstructive pulmonary disease in hospitalized patients. In moderate to severe exacerbations of chronic obstructive pulmonary disease (COPD), inhaled short-acting bronchodilators (β2-agonists and anticholinergic agents) and systemic corticosteroids, antibiotics, and supplemental oxygen are the mainstays of treatment. In one trial of 100 patients with COPD, 16 responded to albuterol only; 17 responded to ipratropium only; and 47 responded to both. Corticosteroids improve lung function, reduce hospital stay, and lower rates of relapse and treatment failure. Antibiotic therapy has been shown to significantly reduce mortality, treatment failure, and sputum purulence in patients with exacerbations of COPD. Noninvasive positive-pressure ventilation should be considered if patients have persistent hypoxemia and/or hypercapnia with a pH less than 7.35 and a PCO2 greater than 45 mm Hg and a respiration rate greater than 25/min despite maximal medical therapy. Exclusion criteria include respiratory arrest, cardiovascular instability, somnolence, impaired mental status, lack of cooperation, high risk of aspiration, recent facial or gastrointestinal surgery, craniofacial trauma, and extreme obesity. Patients who are hypoxemic need supplemental oxygen to achieve an oxygen saturation of 88% or greater. The role of theophylline in COPD exacerbations is controversial. The conclusion from four trials with 169 patients was that theophylline therapy did not affect FEV1 at 2 hours but slightly improved FEV1 at 3 days. There were also increased rates of relapse and occurrence of tremor, palpitations, and arrhythmias. Long-acting β2-agonists, such as salmeterol, have no role in the therapy of acute exacerbations of COPD.

Pneumo 26 A 59-year-old man is evaluated for tachycardia and hypertension 6 hours after undergoing an uncomplicated open cholecystectomy under general anesthesia. The patient had intraoperative high blood pressure and was treated postoperatively with metoprolol, 5 mg every 4 hours by intravenous bolus. The patient underwent repair of a laceration of the liver 5 years ago and had an uncomplicated intraoperative and postoperative course. He has a history of essential hypertension, and his medications are hydrochlorothiazide and metoprolol. On physical examination, the temperature is 39.2 °C (102.5 °F), the blood pressure is 190/110 mm Hg, and the pulse rate is 115/min. There is significant rigidity of all his extremities. Which of the following is the most appropriate therapy for this patient? A Alcohol sponge baths B Ampicillin-sulbactam C Corticosteroids D Dantrolene E Sodium nitroprusside

D Malignant hyperthermia is a life-threatening skeletal muscle disorder characterized by a hypermetabolic state precipitated by exposure to volatile inhalational anesthetics or depolarizing muscle relaxants. This patient likely has malignant hyperthermia, an inherited skeletal muscle disorder characterized by a hypermetabolic state precipitated by exposure to volatile inhalational anesthetics (halothane, isoflurane, enflurane, desflurane, sevoflurane) and the depolarizing muscle relaxants succinylcholine and decamethonium. Although malignant hyperthermia usually occurs at the time of exposure intraoperatively, it can occur several hours after the initial exposure and can develop in patients who were previously exposed to the drug without any effect. Increased intracellular calcium leads to sustained muscle contractions with skeletal muscle rigidity and masseter spasm, tachycardia, hypercarbia, hypertension, hyperthermia, tachypnea, and cardiac arrhythmias. Malignant hyperthermia is life-threatening unless treated promptly and aggressively. Supportive measures include hydration and decreasing the fever. Dantrolene, a skeletal muscle relaxant, is given as a bolus of 1 mg/kg intravenously and then 2 mg/kg every 5 to 10 minutes until the symptoms resolve. Response to dantrolene is not diagnostic of the disorder but is supportive if signs and symptoms resolve quickly. For those patients with a known history, pretreatment with dantrolene before the anesthetic agent is administered prevents the development of symptoms. Alcohol sponge baths are generally not recommended as an augmentation of evaporative cooling in any hyperthermic patient, including malignant hyperthermia, owing to the possibility of substantial alcohol absorption through the skin. Furthermore, augmented cooling (typically accomplished with water misting and forced air circulation by fans) may result in shivering which can increase body temperature unless it is suppressed with benzodiazepine administration. Ampicillin-sulbactam might be a consideration if acute ascending cholangitis were suspected; however, this is unlikely only hours after an elective cholecystectomy. Furthermore, an infection cannot account for the patient's muscular rigidity. Corticosteroids would be effective treatment for an allergic reaction, but there are no symptoms suggesting an allergic reaction such as rash, urticaria, angioedema, or wheezing. Sodium nitroprusside is indicated in patients with hypertensive emergencies. However, this patient's blood pressure is elevated secondary to malignant hyperthermia, and treatment of the underlying disorder is the preferred therapy.

Pneumo 27 A 56-year-old woman is evaluated for a 2-year history of episodic cough and chest tightness. Her symptoms began after a severe respiratory tract infection. Since then, she has had cough and chest discomfort after similar infections, typically lasting several weeks before resolving. She feels well between episodes. She is otherwise healthy and takes no medications. Physical examination reveals no abnormalities, and spirometry is normal. Which of the following is the most appropriate next step in the evaluation of this patient? A Bronchoscopy B CT scan of the sinuses C Exercise echocardiography D Methacholine challenge testing

D Methacholine challenge testing is most useful in evaluating patients with suspected asthma who have episodic symptoms and normal baseline spirometry. This patient's history is consistent with, but not typical of, asthma. This presentation is sometimes referred to as cough-variant asthma. Asthma is often an episodic disease, with normal examination findings and spirometry between episodes. In such cases, a bronchial challenge test, such as with methacholine, can induce bronchoconstriction even when the patient is asymptomatic and spirometry is normal. Methacholine challenge testing is done by giving the patient increasing concentrations of methacholine by nebulization and performing spirometry after each dose until there is a greater than 20% decrease in FEV1 from baseline. The methacholine dose that leads to a 20% decrease in the FEV1 is known as the provocative concentration 20 (PC20) and is calculated from a dose-response curve. In general, a PC20 of less than 4 mg/mL is consistent with asthma. A PC20 between 4 and 16 mg/mL suggests some bronchial hyperreactivity and is less specific for asthma. A PC20 above 16 mg/mL is considered normal. The sensitivity of a positive methacholine challenge test in asthma is in the range of 85% to 95%. False-positive results can occur in patients with allergic rhinitis, chronic obstructive pulmonary disease, heart failure, cystic fibrosis, or bronchitis. Bronchoscopy to evaluate the trachea could be helpful if an anatomic lesion is suspected. However, the symptoms in patients with such lesions are persistent or progressive rather than intermittent. Since this patient has intermittent symptoms, bronchoscopy is not indicated. Exercise echocardiography could help determine the presence of cardiac ischemia or myocardial dysfunction, the typical symptoms of which are dyspnea on exertion, chest tightness, or pain. Cough and wheezing can occur in coronary artery disease, particularly when associated with acute decompensation of the left ventricle, but this patient's intermittent episodes of cough and wheezing are provoked by an upper respiratory tract infection, making the diagnosis of coronary artery disease unlikely. Patients with rhinosinusitis have symptoms consisting of nasal congestion, purulent nasal secretions, sinus tenderness, and facial pain. Radiography, including sinus CT scan, is not indicated in the initial evaluation of acute sinusitis.

Pneumo 30 A previously healthy 62-year-old man was intubated 3 days ago for the acute respiratory distress syndrome complicating community-acquired pneumonia. His condition improved with antibiotic therapy and supportive care. He was extubated 2 hours ago after tolerating a trial of spontaneous breathing, but he has subsequently developed respiratory distress. The patient is alert, cooperative, and speaking in short sentences. The temperature is 36.9 °C (98.4 °F), blood pressure is 150/90 mm Hg, pulse rate is 110/min, and respiration rate is 34/min. Oxygen saturation is 86% on FiO2 0.7 by face mask. There are inspiratory crackles bilaterally, and the patient is using neck and abdominal muscles to breathe. Cardiac examination reveals regular tachycardia without extra sounds or murmurs; the jugular venous pressure could not be assessed because of the patient's respiratory effort. Arterial blood gases at the end of the breathing trial on FiO2 0.4 included pH 7.38, PCO2 35 mm Hg, and PO2 68 mm Hg. His current blood gases following extubation on FiO2 0.7 are pH 7.30, PCO2 45 mm Hg, and PO2 56 mm Hg. Chest radiograph after extubation shows reduced lung volumes but otherwise no change in bilateral alveolar infiltrates from his previous radiograph. Which of the following is the most appropriate management for this patient? A Administer intravenous furosemide B Administer intravenous naloxone C Increase FiO2 to 1.0 D Reintubate and resume mechanical ventilation E Start noninvasive positive-pressure ventilation

D Noninvasive positive-pressure ventilation is potentially harmful in patients with hypoxemic respiratory failure who are failing a trial of extubation. This patient has acute hypoxemic ventilatory failure and should be electively reintubated. The respiratory examination, large supplemental oxygen needs, and rising PCO2 suggest that he is at high risk for respiratory arrest. The most likely explanation for his failing a trial of extubation is insufficient recovery from his initial lung injury. Residual elevated dead space and poor lung compliance, coupled with loss of positive end-expiratory pressure after extubation, account for his deterioration in gas exchange. Furosemide would not be indicated because there is little to suggest new-onset heart failure. Increasing supplemental oxygen is unlikely to reduce the patient's work of breathing or reverse the upward trend in Pco2. Shunt is the primary cause of hypoxemia in patients with the acute respiratory distress syndrome (ARDS), and placing the patient on 100% oxygen will likely only modestly improve hypoxemia. Noninvasive positive-pressure ventilation (NPPV) may prevent postextubation respiratory failure if initiated immediately after extubation. Recent randomized studies have found no benefit, or even increased mortality, with the use of NPPV for patients failing a trial of extubation. Also, routine use of NPPV in patients with ARDS is not recommended. The patient is alert and making vigorous respiratory efforts. Therefore, respiratory failure is unlikely to be the result of excessive sedation, and a trial of naloxone is unlikely to be efficacious.

Pneumo 7 An 18-year-old man is evaluated in the emergency department after his mother found him unconscious in his bed at home. She reported that her son had gone to a party two nights ago, but she was not sure when he returned home. When she checked on him, he was unarousable. He has no significant medical history and takes no medications. In the emergency department, he is afebrile, blood pressure is 110/70 mm Hg, the pulse rate is 50/min, and respiration rate is 6/min; he is intubated for airway protection. Laboratory studies: Hemoglobin 12.2 g/dL (122 g/L) Leukocyte count 3400/µL (3.4 × 109/L) Platelet count 110,000/µL (110 × 109/L) Creatinine 3.2 mg/dL (282.9 µmol/L) Aspartate aminotransferase 80 U/L Alanine aminotransferase 46 U/L Creatine kinase 18,400 U/L INR 1.2 Alkaline phosphatase, bilirubin, and albumin are normal. Urine dipstick is 4+ positive for occult blood. Blood alcohol level is 0.8 g/dL (174 mmol/L). Toxicology testing is positive for opiates and cocaine. Bladder catheterization reveals only 30 mL of brown urine. Which of the following is the most likely cause of the patient's renal failure? A Hemolytic anemia B Hemolytic-uremic syndrome C Hepatorenal syndrome D Rhabdomyolysis E Sepsis

D Nontraumatic causes of rhabdomyolysis include drug use, metabolic disorders, and infections. This patient most likely has rhabdomyolysis, which is caused by skeletal muscle damage that leads to release of intracellular components into the circulation, such as creatine kinase and lactate dehydrogenase, the heme pigment myoglobin, purines, and potassium and phosphate. The syndrome was first identified in patients with traumatic crush injuries, but there are nontraumatic causes, such as alcohol (due to hypophosphatemia), drug use, metabolic disorders, and infections. The classic triad of findings includes muscle pain, weakness, and dark urine. The diagnosis is based on clinical findings and a history of predisposing factors (such as prolonged immobilization or drug toxicity) and confirmed by the presence of myoglobinuria, an increased serum creatine kinase level, and, in some cases, hyperkalemia. The disorder usually resolves within days to weeks. Treatment consists of aggressive fluid resuscitation; fluids should be adjusted to maintain the hourly urine output at least 300 mL until the urine is negative for myoglobin. Acute kidney injury resulting from acute tubular necrosis occurs in approximately one third of patients. Dialysis is sometimes necessary. Although fulminant hepatic failure may result in coma, dark urine, and renal failure, other tests of synthetic liver function in this patient are normal. There are no clinical features to suggest sepsis. The patient has mild anemia, but the proportionate reduction in the leukocyte and platelet counts suggests alcohol-induced bone marrow suppression. Hemolytic anemia would not explain the patient's elevated creatine kinase level and usually does not cause renal failure. Hemolytic uremic syndrome is not consistent with the clinical findings of polysubstance overdose or the laboratory finding of the elevated serum creatine kinase level.

Pneumo 95 A 49-year-old woman is evaluated for fatigue, fever, and a 5.5-kg (12-lb) weight loss since undergoing video-assisted thoracoscopic surgery for a left lower lobe abnormality 3 months ago. Four months ago, the patient developed fever and left-sided chest pain; CT scan (shown) exhibited a left lower lobe abnormality. The abnormality was removed, and the pathologic diagnosis was organizing pneumonia with noncaseating granulomatous features. Fungal stains were negative. She recovered from surgery but remained fatigued and had intermittent fevers. Subsequent evaluation shows a new right upper lobe lesion on CT-positron emission tomographic scan demonstrating marked uptake in the focal lesion and no other abnormalities. The patient has a history of essential thrombocythemia, which has been treated for the past 18 months with pegylated interferon alfa after the disease failed to respond to hydroxyurea. She takes no other medications and does not smoke. Bronchoscopy with transbronchoscopic lung biopsy of the lesion in the right upper lobe again demonstrates organizing pneumonia with noncaseating granulomatous inflammation. Cultures are negative. Which of the following is the most appropriate management for this patient? A Follow-up CT in 3 months B Oral voriconazole therapy C Surgical resection of the right upper lobe lesion D Stop pegylated interferon alfa and start prednisone

D Organizing pneumonia is a nonspecific reparative reaction that can occur in infections, connective tissue diseases, drug reactions, and radiation-induced lung injury. This patient has organizing pneumonia, which is likely caused by treatment with pegylated interferon alfa. Organizing pneumonia is a nonspecific reparative reaction that can occur in infections, connective tissue diseases, drug reactions, and radiation-induced lung injury. The presentation and radiographic findings depend on the cause of the organizing pneumonia. Organizing pneumonia presenting without an identifiable cause is termed cryptogenic organizing pneumonia (COP, formerly called idiopathic bronchiolitis obliterans organizing pneumonia). Patients with COP often present with such systemic symptoms as fever and weight loss. The typical radiographic presentation of COP is bilateral migratory patchy alveolar infiltrates. Organizing pneumonia can present radiographically as a distinct nodule or nodules indistinguishable from malignancy. Focal organizing pneumonia is typically positron emission tomographic (PET) scan-positive, and, therefore, PET does not differentiate focal organizing pneumonia from malignancy. Pegylated interferon alfa has been reported to cause drug-induced lung injury with several histopathologic patterns, including organizing and granulomatous lung disease. When a drug is suspected of causing pulmonary toxicity, it should be stopped whenever reasonable. Treatment with corticosteroids may benefit this patient with systemic symptoms. Bronchoscopic biopsy provides diagnostic tissue, and resection is not needed. Although fungal infection could cause a similar clinicoradiographic presentation, this patient has had multiple biopsies and cultures that failed to show fungal infection; therefore, antifungal therapy is not needed. Observation and follow-up CT in 3 months would not be appropriate. Failure to recognize and treat drug-induced lung disease leads to continuous or progressive symptoms while the offending agent is continued.

Pneumo 77 A 41-year-old woman is evaluated for a 1-year history of progressive dyspnea on exertion and leg swelling. She can walk on level ground but becomes dyspneic when walking up stairs. The patient had a pulmonary embolism at age 35 years while she was taking oral contraceptive pills; she was treated acutely with heparin and then with warfarin for 6 months. She has no other significant medical history and takes no medications. On physical examination, she is afebrile; the blood pressure is 120/78 mm Hg, the pulse rate is 97/min, the respiration rate is 16/min, and the BMI is 27. The lungs are clear and diaphragmatic excursion is normal; the S2 has a fixed split. Chest radiograph shows no parenchymal abnormalities. Spirometry and plethysmography are normal. Transthoracic echocardiography reveals a dilated right atrium and a hypertrophied, enlarged right ventricle. There is moderate tricuspid regurgitation but no other valvular abnormalities. Which of the following is the most appropriate next diagnostic test in this patient? A Coronary angiography B Endocardial biopsy C Pulmonary angiography D Ventilation/perfusion scan

D Patients with chronic thromboembolic pulmonary hypertension often present with dyspnea on exertion without (or out of proportion to) pulmonary parenchymal disease. The patient has chronic symptoms and echocardiographic evidence of pulmonary hypertension. Her history of acute pulmonary embolism raises the possibility of chronic thromboembolic pulmonary hypertension (CTEPH). Patients with CTEPH often present with unexplained progressively worsening dyspnea, especially dyspnea on exertion, in the absence of (or out of proportion to) pulmonary parenchymal disease. The predisposing acute pulmonary embolism may have been asymptomatic or undiagnosed, and, therefore, only about 50% of patients have a history of clinically detected pulmonary embolism. Ventilation/perfusion scanning can help determine whether the patient's pulmonary hypertension is due to obstruction of large pulmonary arteries (as is characteristic of CTEPH) or to widespread obstruction of the small pulmonary vasculature. Pulmonary angiography can confirm the diagnosis of CTEPH and determine whether the large-vessel obstruction is amenable to surgical endarterectomy. Pulmonary arteriographic findings in CTEPH are very different from those in acute pulmonary embolism. In acute pulmonary embolism, intraluminal filling defects and abrupt vessel "cutoffs" are diagnostic. In CTEPH, however, the embolic material has been organized into the arterial wall itself, and angiography shows gradual tapering of the lumen, whereas the presence of webs and luminal irregularities give evidence of the body's ineffectual attempts at recanalization. However, screening for CTEPH with a ventilation/perfusion scan should take precedence over the more invasive pulmonary angiography as the next diagnostic test. Coronary artery disease would be unlikely to result in isolated right ventricular hypertrophy and right atrial enlargement. Therefore, coronary angiography is not indicated. Endocardial biopsy might elucidate the cause of a diffuse cardiomyopathy. However, this patient's findings are specific to the right ventricle and suggest chronic strain and hypertrophy.

Pneumo 65 An 87-year-old man is evaluated in the hospital for idiopathic pulmonary fibrosis. He was diagnosed 3 years ago, and his condition has gradually worsened despite therapy with prednisone, azathioprine, and N-acetylcysteine. All therapy has been discontinued over the past 6 months because of failure to respond and side effects. He has been home-bound on high-flow oxygen and has been hospitalized three times in the past year. The patient also has type 2 diabetes mellitus and hypertension, and his current medications are metformin, hydrochlorothiazide, and levofloxacin. He has indicated that he does not want additional aggressive therapy. On physical examination, the patient is in moderate respiratory distress; temperature is 37.2 °C (99 °F), the blood pressure is 132/93 mm Hg, the pulse rate is 122/min, and the respiration rate is 32/min. Oxygen saturation is 89% with the patient breathing 100% oxygen by face mask. The patient is using accessory muscles to breathe, and there are bilateral dry crackles. Leukocyte count is 7200/µL (7.2 × 109/L) with a normal differential count. Chest radiograph shows bilateral lower- and mid-lung fibrotic infiltrates with enlarged pulmonary arteries. Which of the following is the most appropriate recommendation? A Intravenous methylprednisolone B Mechanical ventilation C Noninvasive positive-pressure ventilation D Palliative care

D Patients with idiopathic pulmonary fibrosis who require intensive care unit admission for respiratory failure have a very poor prognosis. This patient has had gradual worsening of idiopathic pulmonary fibrosis and has not responded to immunosuppressive therapies. He now has severe respiratory distress and either needs ventilatory support or initiation of palliative care. Patients with pulmonary fibrosis requiring admission to the intensive care unit for respiratory failure have a very poor prognosis, with in-hospital mortality rates of 61% and post-discharge rates of 92% by 2 months. This patient has no evidence of respiratory infection and has failed to respond to previous appropriate treatment, and he should be informed of his poor prognosis and offered the option to initiate palliative care. End-of-life care should be discussed with all patients with idiopathic pulmonary fibrosis, ideally in the outpatient setting with family present and when there is no urgency to intervene. Mechanical ventilation would be reasonable provided that the patient understands his poor prognosis and wishes to continue with aggressive treatment. Initiation of noninvasive positive-pressure ventilation is not an effective means to avoid mechanical ventilation in patients with pulmonary fibrosis. Noninvasive ventilation may delay the need for intubation and ventilation, but it would not improve this patient's poor prognosis. Methylprednisolone is commonly given to patients with idiopathic pulmonary fibrosis and worsening respiratory distress, but there is no evidence that treatment of such patients with respiratory failure is beneficial, and methylprednisolone is unlikely to reverse this patient's need for more urgent management of respiratory failure.

Pneumo 52 A 65-year-old man is evaluated for daytime sleepiness. His wife says that he snores nightly and in all sleep positions and that she has witnessed the patient having an apnea episode. The patient has repeated episodes of nighttime awakenings when he feels short of breath and has headaches and dry mouth on awakening in the morning. He typically sleeps for about 8 hours each night but awakens unrefreshed. He often takes a 1-hour nap in mid-afternoon. He does not smoke or drink alcohol, coffee, tea, or cola beverages. On physical examination, vital signs are normal; BMI is 37. His neck circumference is 45.72 cm (18 in). Oxygen saturation is 90% with the patient breathing ambient air. The patient is edentulous, and examination of his oropharynx reveals a low-lying palate, narrow pharyngeal wall, and a posteriorly displaced tongue. The rest of the physical examination is normal. Which of the following is the most appropriate management at this time? A Continuous positive airway pressure (CPAP) therapy B Instructions on avoiding a supine sleep position C Oxygen therapy D Polysomnography E Referral for a mandibular repositioner

D Polysomnography is indicated to diagnose obstructive sleep apnea and assess its severity. Obstructive sleep apnea is more common in men than in women, and the prevalence increases with age and body weight. Common clinical features include excessive sleepiness, habitual snoring, witnessed apneas, awakenings with gasping or choking, insomnia, nighttime diaphoresis, morning headaches, nocturia, daytime fatigue, alterations in mood, and neurocognitive decline. Physical examination may reveal hypertension, excessive body weight, large neck circumference (greater than 43.2 cm [17 in] for men and greater than 40.6 cm [16 in] for women), nasal obstruction, tonsillar enlargement, low-lying palate, narrow oropharynx, macroglossia, or retro- or micrognathia. Polysomnography is indicated for the diagnosis because clinical features and physical examination findings are neither sufficiently sensitive nor specific for the disorder. For some patients, respiratory events occur predominantly or exclusively during a supine sleep position. Therapy for obstructive sleep apnea, which should be undertaken only after polysomnography, includes continuous positive airway pressure (CPAP), oral devices, and upper airway surgery. Positive airway pressure therapy is the most effective. Avoidance of alcohol, sedatives, and muscle relaxants is important, as is weight management. For some patients, the upper airway is most vulnerable to collapse during supine sleep; measures to prevent sleep in a supine position may be appropriate if the patient's apnea-hypopnea index normalizes during sleep in a lateral or prone sleep position. Supplemental oxygen is not a primary therapy for obstructive sleep apnea. Mandibular repositioners, the most commonly used oral devices, are contraindicated in persons with compromised or inadequate dentition (such as this patient) and should not be prescribed without a confirmed diagnosis of obstructive sleep apnea.

Pneumo 49 A 36-year-old woman with an 18-month history of diffuse cutaneous systemic sclerosis is evaluated for a 6-month history of dry cough and decrease in exercise capacity. She is otherwise healthy and takes no medications. On physical examination, the blood pressure is 110/68 mm Hg, the pulse rate is 82/min, and the respiration rate is 18/min. Cardiac examination is normal. There are bilateral dry crackles but no other pulmonary abnormalities. The skin over the digits on both hands and parts of the arms and forearms is thickened and tight. Laboratory studies show normal serum electrolytes, creatinine, and complete blood count with leukocyte differential. Anti-Scl-70 antibodies are positive. Pulmonary function tests show an FEV1 of 80% of predicted, an FVC of 76% of predicted, and a DLCO of 83% of predicted. High-resolution chest CT scan shows scant bilateral reticular changes with a basilar and peripheral distribution. There is no evidence of ground-glass opacification. Visual impression of the extent of lung involvement is 10%. Which of the following is the most appropriate management for this patient's pulmonary disease? A Cyclophosphamide and low-dose prednisone B High-dose prednisone C Open lung biopsy D Periodic high-resolution CT scan and lung function testing

D Pulmonary disease occurs in 70% of patients with systemic sclerosis and is the primary cause of mortality. Approximately 50% of patients with systemic sclerosis who have pulmonary manifestations develop severe pulmonary involvement within the first 3 years of disease onset. In this patient, the high-resolution CT (HRCT) findings show minimal lung involvement (less than or equal to 20%) and are not diagnostic of active alveolitis; therefore, therapy is not indicated at this time. Follow-up and HRCT imaging and pulmonary function testing can determine whether this patient will benefit from systemic cytotoxic therapy. Pulmonary disease occurs in 70% of patients with systemic sclerosis and is the primary cause of death: 15% of affected patients develop severe restrictive pulmonary disease, with an associated 10-year mortality rate of 50%. Pulmonary fibrosis is more common in diffuse cutaneous systemic sclerosis than in limited cutaneous disease. A recent study of interstitial lung disease in patients with systemic sclerosis showed that patients with extensive, but not moderate, alveolitis benefit from treatment with cyclophosphamide. FVC less than 70% of predicted was predictive of response to therapy. Goh and colleagues showed that clinicians' visual assessment of extent of lung disease on HRCT of less than 20% was highly predictive of survival at 120 months. Open lung biopsy is not necessary to diagnose systemic sclerosis-associated interstitial lung disease unless HRCT features suggest an alternative diagnosis. Bronchoscopy with bronchoalveolar lavage is less invasive and may be appropriate in select cases to exclude infection or active alveolitis. There is no evidence that high-dose prednisone therapy is effective, and it is associated with an increased risk for scleroderma renal crisis.

Pneumo 102 A 70-year-old woman is evaluated for a 6-month history of fatigue, an unintentional weight loss of 4.4 kg (10 lb), an increase in chronic cough with sputum production, and a decrease in exercise capacity. The patient has a 40-pack-year history of cigarette smoking but stopped smoking 10 years ago when chronic obstructive pulmonary disease was diagnosed. She has no other symptoms and specifically denies abdominal pain, nausea, vomiting, diarrhea, or change in her bowel habits. Her medications are albuterol as needed, an inhaled corticosteroid, and salmeterol. She has been on stable dosages of these drugs for 18 months. Age- and sex-appropriate cancer screening tests done 6 months ago were normal. On physical examination, the temperature is 37.5 °C (99.5 °F), the blood pressure is 128/76 mm Hg, the pulse rate is 94/min and regular, the respiration rate is 16/min, and the BMI is 20. Heart sounds are distant, and breath sounds are diminished bilaterally. There are no abdominal masses or organomegaly and no peripheral edema. Laboratory studies: Hemoglobin 15 g/dL (150 g/L) Albumin 3.0 g/dL (30 g/L) Creatinine 0.8 mg/dL (70.7 µmol/L) Thyroid-stimulating hormone 2.0 µU/mL (2.0 mU/L) Spirometry shows an FEV1 of 40% of predicted and an FEV1/FVC ratio of 45%. Chest radiograph shows hyperinflation. Which of the following is the most likely reason for this patient's weight loss? A Breast cancer B Cervical cancer C Colon cancer D Chronic obstructive pulmonary disease

D Severe chronic obstructive pulmonary disease can cause systemic effects including unintentional weight loss; skeletal muscle dysfunction; and increased risk of cardiovascular disease, osteoporosis, and depression. Severe chronic obstructive pulmonary disease (COPD) can cause systemic effects including unexplained weight loss, skeletal muscle dysfunction, increased cardiovascular morbidity and mortality, increased risk for type 2 diabetes mellitus, osteoporosis, fractures, and depression. Unexplained weight loss occurs in about half of the patients with severe COPD, mostly due to the loss of skeletal muscle mass. Unexplained weight loss carries a poor prognosis in COPD independent of other indicators, such as FEV1 or Pco2. Although the weight loss of malignancy is a possibility in this patient, the absence of gastrointestinal symptoms or other localizing symptoms, the normal cancer screening tests within the last year, and the patient's history of severe COPD make the cachexia of COPD the most likely cause in this patient. The patient's spirometry indicates severe COPD and aggressive management of COPD is necessary. Evaluation for depression is also indicated. The mechanisms underlying these systemic effects are unclear; they are probably interrelated and multifactorial, including inactivity, systemic inflammation, tissue hypoxia, and oxidative stress. Increases in concentrations of inflammatory mediators indicating peripheral blood cell activation also have been found throughout the body and may mediate some of these systemic effects.

Pneumo 87 A 55-year-old woman with a long-standing history of moderate to severe asthma is evaluated for a worsening productive cough, dyspnea, and wheezing. She has not had fever, chills, or chest pain. Her current medications are low-dose inhaled corticosteroids, a long-acting β-agonist, and albuterol. On physical examination, the temperature is 36.5 °C (97.8 °F), the blood pressure is 110/70 mm Hg, the pulse rate is 84/min, and the respiration rate is 18/min. The cardiac examination is normal; pulmonary examination reveals bilateral wheezing and scattered rhonchi over the upper lung fields. Oxygen saturation is 93% with the patient breathing ambient air. The leukocyte count is 8500/µL (8.5 × 109/L) with 45% neutrophils, 35% lymphocytes, 10% eosinophils, and 10% mononuclear cells. Chest radiograph shows patchy infiltrates in both upper lobes and prominent bronchial markings consistent with bronchiectasis. High-resolution chest CT scan confirms the presence of central bronchiectasis and some mucous plugging. Which of the following is the most appropriate next step in the evaluation of this patient? A Bronchoscopy with lung biopsy B Methacholine challenge test C Positron-emission tomography D Skin test for Aspergillus fumigatus E Sweat chloride test

D The characteristic features of allergic bronchopulmonary aspergillosis include moderate to severe persistent asthma, bronchiectasis and chest radiographic abnormalities, elevated serum IgE level, eosinophilia, and a positive skin test to Aspergillus fumigatus. Allergic bronchopulmonary aspergillosis (ABPA) is a rare (occurring in 1% to 2% of patients with asthma), but important, complication in patients with asthma. These patients develop humoral and cell-mediated immune responses to Aspergillus fumigatus in their airways, leading to persistent inflammation, airway damage with development of central bronchiectasis, and eventually pulmonary fibrosis. Features of the disorder include moderate to severe persistent asthma, expectoration of brown sputum that contains Aspergillus organisms, bronchiectasis and radiologic abnormalities (infiltrate, mucous plugging), elevated levels of serum total IgE (typically greater than 0.1 mg/dL [1.0 mg/L]), a positive skin test to A. fumigatus, and eosinophilia. Treatment often requires the use of systemic corticosteroids. A skin test to determine the presence of allergic response to Aspergillus is an important first step in evaluating patients with ABPA because essentially all patients with ABPA have a positive skin test. However, many patients with a positive skin test do not have ABPA, and therefore, the positive predictive value of the skin test is low. A negative skin prick test followed by negative intradermal skin test virtually excludes the diagnosis of ABPA, provided that the tests were done using optimal extracts and technique. If the skin test is positive, the next diagnostic step is measurement of serum total IgE. ABPA is excluded if the serum total IgE concentration is less than 0.1 mg/dL (1.0 mg/L). If skin testing for Aspergillus is not available, measuring serum total IgE may be an efficient way to pursue the diagnosis. Bronchoscopy is an invasive procedure that is not indicated unless an alternative diagnosis (for example, a superimposed opportunistic infection) is suspected. Positron emission tomography may reveal increased uptake in the lungs due to ongoing pulmonary inflammation, but the information is unlikely to be specific or helpful in making a diagnosis in this patient and adds significant cost. A methacholine challenge test would be positive in patients with asthma regardless of the presence of ABPA. The test is not recommended in patients with symptomatic airway obstruction or FEV1 less than 70% because of the risk of severe airway obstruction. The sweat chloride test is a screening test for cystic fibrosis, which leads to central bronchiectasis and chronic pulmonary infection with frequent exacerbations. However, the symptoms of cystic fibrosis typically start in childhood and rarely present for the first time at age 55 years and do not include eosinophilia, unless complicated by ABPA. Patients with cystic fibrosis are, however, at significant risk for ABPA, with most studies showing a prevalence of approximately 10%.

Pneumo 25 A 56-year-old woman is evaluated for a 2-month history of a drooping eyelid, difficulty chewing food and swallowing, and slurred speech. The symptoms are worse when she is tired. The patient has a 15-pack-year history of cigarette smoking but quit smoking 10 years ago. She is otherwise healthy and takes no medications. On physical examination, the temperature is 37.0 °C (98.6 °F), the blood pressure is 118/60 mm Hg, the pulse rate is 72/min, the respiration rate is 16/min, and the BMI is 24.5. There is right-sided ptosis; pupils are equal in size and reactive, and eye movements are normal. Brief neurologic examination is normal, and there is no lymphadenopathy. Routine laboratory studies are normal; acetylcholine receptor binding antibody level is 46.8 nmol/L (normal range, less than 0.2 nmol/L). Chest radiograph shows a 3-cm anterior-superior mediastinal mass. CT scan of the chest is shown . Which of the following is the most likely diagnosis? A Esophageal leiomyoma B Neurolemmoma C Sarcoidosis D Small cell lung cancer E Thymoma

E Thymomas may be associated with a paraneoplastic syndrome such as myasthenia gravis. Up to 75% of patients with myasthenia gravis have thymic abnormalities, such as hyperplasia or tumor. Therefore, evaluation of patients with suspected myasthenia gravis should include CT scan of the chest. The CT image shows an anterior mediastinal mass, and the diagnosis of myasthenia gravis is supported by the findings of ptosis and the elevated acetylcholine receptor binding antibody (AChR-Ab). Myasthenia gravis is characterized by weakness with a predilection for various muscles, particularly the ocular, bulbar, proximal extremities, neck, and respiratory muscles. Symptoms of this condition are traditionally worsened by fatigue, exertion, a rise in body temperature, stress, and intercurrent infections. Approximately 30% to 50% of patients with thymoma have myasthenia gravis, and the tumors are most likely to be located in the anterior mediastinum. An esophageal tumor or a neurolemmoma is typically a posterior mediastinal mass. Although sarcoidosis may be a cause of ptosis, it usually presents with symmetric hilar and middle mediastinal lymphadenopathy. Small cell lung cancer may cause a mediastinal mass and various neurologic syndromes, including Lambert-Eaton syndrome, but is not associated with an elevated AChR-Ab but rather with a serum anti-voltage-gated calcium channel antibody. Lambert-Eaton syndrome usually manifests with proximal upper- and lower-extremity weakness and symptoms of autonomic dysfunction (dry eyes and mouth, orthostatic hypotension, and bowel and bladder dysfunction).

Pneumo 81 A 40-year-old woman is evaluated for worsening asthma symptoms after resolution of an acute respiratory tract infection that was treated with supportive measures. The patient has a 15-year history of asthma that has been well-controlled on moderate-dose inhaled corticosteroids plus as-needed inhaled albuterol. Since her respiratory tract infection 10 days ago, her asthma symptoms have worsened; she has had frequent nighttime episodes of wheezing and has used her albuterol inhaler six to eight times a day. On physical examination, the patient is afebrile and has no chest pain or significant sputum production. Her peak flow is more than 40% below her baseline value. Which of the following is the most appropriate management for this patient? A 7-day course of a fluoroquinolone B Leukotriene modifying agent C Long-acting β-agonist D Nebulized albuterol at home E Short course of an oral corticosteroid

E A short course of oral corticosteroids may help restore asthma control in previously well-controlled patients who have developed unstable disease as a result of a respiratory tract infection. This patient with previously well-controlled asthma has had "loss of control" after having had a respiratory tract infection. A short course of an oral corticosteroid (for example, prednisone 0.5 mg/kg daily for 5 to 7 days) can resolve the asthma symptoms and enable the patient to regain control of her disease. It is unclear whether doubling (or even quadrupling) the dose of inhaled corticosteroids is an effective strategy in place of oral corticosteroids. Antibiotics are generally not recommended for acute respiratory infections in asthma because most of these infections are viral. There are some data to suggest that atypical bacteria (for example, Mycoplasma or Chlamydia) may contribute to acute asthma exacerbations or persistence of symptoms after a respiratory tract infection. However, at this time the routine use of antibiotics in patients with an asthma exacerbation is not recommended. Nebulized therapy at home should be reserved for patients who cannot use a metered-dose inhaler appropriately. Even though nebulized bronchodilator therapy can be more effective in reversing bronchoconstriction than metered-dose inhaled bronchodilators, nebulized therapy should not be used as a substitute for oral corticosteroid therapy in patients with asthma exacerbations. Adding a leukotriene-modifying agent can be considered in patients who cannot or will not take oral corticosteroids; however, leukotriene receptor antagonists are less potent anti-inflammatory agents than corticosteroids and are not effective in patients with significant exacerbations. Adding a long-acting β-agonist would be reasonable in this patient if her symptoms persist after the oral corticosteroid therapy, but the persistence and severity of the patient's current symptoms suggest that there is ongoing airway inflammation and that a systemic corticosteroid is warranted.

Pneumo 6 A 54-year-old man is evaluated in the emergency department for a 1-hour history of chest pain with mild dyspnea. The patient had been hospitalized 1 week ago for a colectomy for colon cancer. His medical history also includes hypertension and nephrotic syndrome secondary to membranous glomerulonephritis, and his medications are furosemide, ramipril, and pravastatin. On physical examination the temperature is 37.5 °C (100 °F), the pulse rate is 120/min, the respiration rate is 24/min, the blood pressure is 110/60 mm Hg, and the BMI is 30. Oxygen saturation is 89% with the patient breathing ambient air and 97% on oxygen, 4 L/min. Cardiac examination shows tachycardia and an S4. Breath sounds are normal. Chest radiograph is negative for infiltrates, widened mediastinum, and pneumothorax. Serum creatinine concentration is 2.1 mg/dL (185.6 µmol/L). Empiric unfractionated heparin therapy is begun. Which of the following is the best test to confirm the diagnosis in this patient? A Assay for plasma D-dimer B CT angiography C Lower extremity ultrasonography D Measurement of antithrombin III E Ventilation/perfusion scan

E Either ventilation/perfusion scanning or contrast-enhanced CT scanning (if not contraindicated) performed with a specific protocol to detect pulmonary embolism is an appropriate noninvasive test to diagnose acute pulmonary embolism. This patient is at high risk for pulmonary embolism because of his recent hospitalization, cancer, and nephrotic syndrome. A positive ventilation/perfusion scan would confirm the diagnosis of pulmonary embolism in this patient with a high pretest probability for the condition, especially in the absence of parenchymal lung defects on chest radiograph. The probability of pulmonary embolism was very high based on this presentation that included chest pain, dyspnea, recent hospitalization and surgery, active cancer, and a protein-losing nephropathy. A negative D-dimer test would not be sufficient evidence to rule out a pulmonary embolism under these circumstances, and a high D-dimer level would add little to the diagnostic work-up. Decreased antithrombin III levels may result from nephrotic syndrome, and levels are lowered during acute thrombosis, especially during treatment with heparin. Therefore, measuring antithrombin III would add little to the accuracy of the diagnosis of pulmonary embolism or have any implication for immediate management decisions. Lower extremity ultrasonography can disclose asymptomatic deep venous thrombosis in a small percentage of patients presenting with symptoms of pulmonary embolism. However, the yield is relatively low and ventilation/perfusion scanning would have a much higher degree of accuracy. CT angiography is an acceptable modality to diagnose acute pulmonary embolism but requires a significant amount of contrast infusion (as much as a pulmonary angiogram) which would be contraindicated in a patient with an elevated serum creatinine level.

Pneumo 85 A 67-year-old man is evaluated in the emergency department for the acute onset of dyspnea associated with severe frontal headaches and diaphoresis. He was diagnosed 5 days ago with a left adrenal pheochromocytoma, and surgery is scheduled in 3 days. He has no other significant medical history. His current medications include phenoxybenzamine and propranolol, but the patient is confused about when to take his medications. On physical examination, he appears anxious and dyspneic; the blood pressure is 220/120 mm Hg, the pulse rate is 120/min, the respiration rate is 28/min, and the oxygen saturation is 92% with the patient breathing ambient air. Auscultation of the lungs reveals bibasilar inspiratory crackles. He has 2+ pedal edema. Chest radiograph shows cardiomegaly and pulmonary congestion. Electrocardiogram shows no evidence of ischemia or infarction. Which of the following is the appropriate initial treatment for this patient's hypertension? A Captopril, orally B Metyrosine, orally C Metoprolol, intravenously D Nicardipine, orally E Nitroprusside infusion

E Hypertensive crisis from pheochromocytoma is typically treated with sodium nitroprusside, phentolamine, or nicardipine. Acute pheochromocytoma hypertensive crises may occur before or during an operation, and patients require intravenous treatment with rapidly acting drugs such as sodium nitroprusside, phentolamine, or nicardipine. Sodium nitroprusside is typically preferred because of its rapid onset of action and short half-life. Oral captopril and oral nifedipine are too slow acting and their blood pressure effect too unpredictable to be useful in the treatment of this patient. Metyrosine inhibits catecholamine synthesis, but this drug is generally used preoperatively in patients who cannot tolerate α- and β-adrenergic blockade. It must be administered 3 to 4 days before surgery and therefore is not indicated for treatment of a hypertensive crisis. The use of a β-blocker, either orally or intravenously, is contraindicated in a patient with a hypertensive crisis from pheochromocytoma because unopposed α-adrenergic activity may actually worsen the blood pressure.

Pneumo 4 A 52-year-old woman is evaluated after a screening CT colonography detected a 3-mm nodule in the right lower lobe of the lung. A tortuous colon prevented complete screening colonoscopy. CT scan of the chest showed no additional nodules and was otherwise normal. The patient has never smoked; she works in the home and has not been exposed to potential carcinogens. She has not had a chest radiograph or other imaging procedure, except mammography. Her medical history includes only hyperlipidemia, and her only medication is simvastatin. Her family history is unremarkable. On physical examination, vital signs are normal. Examination of the skin is normal; there is no lymphadenopathy, and the lungs are clear. Which of the following is the most appropriate next step in the management of this patient? A Chest radiograph in 3 months B CT scan of the chest in 3 months C CT scan of the chest in 6 months D CT scan of the chest in 12 months E No follow-up

E In a patient at low risk for malignancy no follow-up is required for an incidentally noted pulmonary nodule 4 mm or smaller. Studies of chest CT screening have shown that 25% to 50% of patients have one or more pulmonary nodules detected on the initial CT scan. Even in patients at relatively high risk for lung cancer, the likelihood that a small nodule is malignant is low (<1%). For example, the risk of malignancy is about 0.2% for nodules smaller than 3 mm and 0.9% for nodules 4 to 7 mm. The Fleischner Society recommendations include no follow-up for low-risk patients with nodules 4 mm or smaller and follow-up CT at 12 months for patients with such nodules who are at risk for lung cancer. More frequent follow-up is not recommended for nodules of this size. This small nodule is not likely to be visible on chest radiograph, and, therefore, such imaging would not be helpful.

Pneumo 107 A 51-year-old man is evaluated during a routine follow-up examination in November. He was diagnosed with GOLD stage I chronic obstructive pulmonary disease 2 years ago. He has a 20-pack-year history of cigarette smoking but stopped smoking 18 months ago. He also has hypertension, and his medications are inhaled albuterol as needed and hydrochlorothiazide. The patient had influenza and pneumococcal vaccinations 1 year ago. Which of the following is the most appropriate immunization strategy for this patient at this time? A Pneumococcal vaccination only B Trivalent intranasal live influenza vaccination only C Trivalent intranasal live influenza and pneumococcal vaccinations D Trivalent killed influenza and pneumococcal vaccinations E Trivalent killed influenza vaccination only

E Influenza vaccination is recommended annually for all patients with chronic obstructive pulmonary disease regardless of age. Annual influenza vaccination is recommended in patients with chronic obstructive pulmonary disease (COPD) regardless of their age. In the general population, annual influenza vaccination is recommended in persons 50 years of age or older. If this patient received influenza and pneumococcal vaccination 1 year ago, he needs only influenza vaccination. Pneumococcal vaccination is recommended for all adults older than 65 years and for younger patients who are active smokers or who have various disorders, including COPD and asthma, that increase their risk for invasive pneumococcal disease. Influenza vaccination is also recommended for pregnant women whose last two trimesters coincide with the influenza season (late December through mid-March). The main vaccine used in the United States is a trivalent inactivated virus, but an intranasally administered vaccine from a trivalent live attenuated virus is also available for patients aged 5 to 49 years who are not pregnant, immunosuppressed, or living with an immunosuppressed person. Because of the patient's age, the most appropriate vaccination is the trivalent killed influenza vaccination. A single revaccination with pneumococcal vaccine is recommended in adults older than 65 years if they were vaccinated more than 5 years previously at a time when they were less then 65 years of age and in immunosuppressed patients 5 years or more after the first dose. Patients with COPD who received their first pneumococcal vaccination after age 65 years do not need revaccination.

Pneumo 9 A 20-year-old woman is evaluated in the emergency department for an acute episode of wheezing and dyspnea without cough or sputum production. She has had previous frequent evaluations in emergency departments and urgent care centers for similar episodes. In between these episodes, findings on physical examination and pulmonary function testing, including methacholine challenge, have been normal. She is otherwise healthy and takes no medications. On physical examination, the patient has inspiratory and expiratory wheezing and is in moderate discomfort. The temperature is 37.1 °C (98.8 °F), pulse rate is 100/min, and the respiration rate is 24/min; oxygen saturation on ambient air is 96%. After receiving albuterol and intravenous corticosteroids, she continues to wheeze and is in moderate respiratory distress. Oxygen saturation on ambient air remains at 96%. Chest radiograph shows decreased lung volumes. Which of the following is the most appropriate management for this patient? A Chest CT scan B Intravenous aminophylline C Intravenous azithromycin D Intravenous terbutaline E Laryngoscopy

E Laryngoscopy during an exacerbation of vocal cord dysfunction shows adduction of the vocal cords during inspiration. This patient likely has vocal cord dysfunction (VCD). Patients with VCD can have throat or neck discomfort, wheezing, stridor, and anxiety. The disorder can be difficult to differentiate from asthma; however, affected patients do not respond to the usual asthma therapy. Diagnosing VCD is made more difficult by the fact that many of these patients also have asthma. The chest radiograph in this patient showed decreased lung volumes, which is in contrast to hyperinflation that would be expected in acute asthma. Oxygen saturation is typically normal in patients with VCD. Laryngoscopy, especially when done while the patient is symptomatic, can reveal characteristic adduction of the vocal cords during inspiration. Another test that helps make the diagnosis is flow volume loops, in which the inspiratory and expiratory flow rates are recorded while a patient is asked to take a deep breath and then to exhale. In patients with VCD, the inspiratory limb of the flow volume loop is "flattened" owing to narrowing of the extrathoracic airway (at the level of the vocal cords) during inspiration. Recognition of VCD is essential to prevent lengthy courses of corticosteroids and to initiate therapies targeted at VCD, which include speech therapy, relaxation techniques, and treating such underlying causes as anxiety, postnasal drip, and gastroesophageal reflux disease. Intravenous aminophylline is not recommended for treating either acute asthma or VCD. Therapy with intravenous terbutaline or other β-agonists for asthma exacerbations is associated with an unacceptably high rate of side effects. Azithromycin is a reasonable choice for acute bronchitis, but there is little evidence that this patient has acute bronchitis, which would manifest with cough, sputum production, and fever. The chest CT scan can be used to exclude parenchymal lung disease or evaluate the possibility of a pulmonary embolism; however, these disorders are unlikely in this patient with previous normal pulmonary examinations and radiographs and excellent oxygenation, and chest CT scan is unlikely to yield useful information.

Pneumo 23 A 23-year-old man seeks medical advice for an upcoming mountain expedition. A year earlier, a planned 45-day trek to Lhotse in Nepal (elevation 8516 m [27,940 ft]) was cut short when he developed severe dyspnea and cough productive of blood-tinged, frothy sputum shortly after leaving the base camp (elevation 4930 m [16,174 ft]). When his symptoms persisted despite oxygen therapy, he was aided down the mountain. He plans to return to the high Himalayas for another attempt to climb the Lhotse summit. Which of the following would be appropriate prophylaxis for this patient? A Acetazolamide B Dexamethasone C Hydrochlorothiazide D Metoprolol E Nifedipine

E Nifedipine is used both to prevent and to treat high-altitude pulmonary edema. Both the occurrence and severity of respiratory symptoms at high altitude are affected by the degree of elevation, rapidity of ascent, altitude during sleep, comorbid cardiovascular and respiratory disorders, physical exertion at altitude, and individual variations in tolerance to altitude (for example, altitude illness is more common in those with inadequate hypoxic ventilatory drive, prior history of altitude illness, and residence below an altitude of 915 m [3000 ft]). High-altitude pulmonary edema (HAPE) is a form of noncardiogenic pulmonary edema due to leakage of fluid and hemorrhage into the alveolar spaces. The most effective preventive measure for HAPE is an appropriately gradual ascent to altitude (not greater than 300 to 500 m [984 to 1640 ft] daily above an altitude of 2000 m [6562 ft], with scheduled rest days every 3 or 4 days). Nifedipine is used to prevent and to treat HAPE. Acetazolamide is used as prophylaxis for periodic breathing related to high altitude and acute mountain sickness (AMS) but is not indicated for HAPE. Dexamethasone is used for the prevention and treatment of AMS; it is not generally considered as a prophylactic agent for HAPE. Diuretics have been used for HAPE, but their role is not clearly established and there is no compelling evidence for their use. β-Blockers are not indicated for either the prevention or treatment of HAPE. Other agents that might be potentially useful for preventing HAPE include the phosphodiesterase inhibitors tadalafil and sildenafil, as well as salmeterol, but additional studies are required to ascertain their precise role for this indication.

Pneumo 104 A 52-year-old man with a history of amyotrophic lateral sclerosis is admitted to the intensive care unit with hypoxemic respiratory failure as a result of right-sided pneumonia. The patient is intubated, and therapy with ceftriaxone and azithromycin is started. His condition improves after 4 days, but large amounts of thin yellow secretions are being suctioned hourly. On physical examination, the patient is alert and cooperative. The temperature is 37.0 °C (98.6 °F), pulse rate is 90/min, respiration rate is 14/min, and blood pressure is 120/80 mm Hg; oxygen saturation is 96% on the ventilator. There are diffuse rhonchi and inspiratory crackles over the right hemithorax. One hour into a spontaneous breathing trial, the patient's tidal volume is 500 mL, respiration rate 20/min, and oxygen saturation 95% on FiO2 of 0.4. Arterial blood gases are pH 7.35, PCO2 55 mm Hg, and PO2 65 mm Hg. Chest radiograph shows slight improvement in the right middle and lower lobe infiltrates. When would it be appropriate to extubate this patient? A Now B When the chest radiograph clears C When the rapid shallow breathing index improves D When supplemental oxygen needs decrease E When the volume of secretions decreases

E Patients requiring frequent suctioning or with impaired ability to clear secretions are not suitable candidates for extubation even if they do not require mechanical ventilatory support. This patient has clinically improved and tolerated a trial of spontaneous breathing. However, he still is requiring frequent suctioning and would, therefore, be at high risk for failing a trial of extubation. It is particularly important to consider the burden of secretions in patients with impaired cough, such as this patient with neuromuscular weakness. This case emphasizes the importance of addressing whether the patient needs the ventilator and whether the patient needs the endotracheal tube before extubation. Improvement in the radiographic appearance of pneumonia typically lags substantially behind clinical improvement. Therefore, the appearance of chest radiographs does not routinely influence the time course of ventilator weaning. The patient's rapid shallow breathing index (RSBI) during a trial of spontaneous breathing indicates that he is ready to breathe independent of the ventilator. The RSBI (respiration rate/tidal volume) in this patient is 20/min/0.5 L or 40 breaths/min/L; values less than 105 breaths/min/L have been shown to be predictive of extubation success. The patient has adequate oxygenation on 40% supplemental oxygen. Patients with supplemental oxygen needs less than 40% to 50% are considered suitable candidates for weaning.

Pneumo 60 A 66-year-old man is evaluated during a routine examination. The patient has smoked a pack of cigarettes a day for 25 years. He has tried to quit smoking with nicotine gum and varenicline but has been unsuccessful. He has mild airway obstruction but good exercise tolerance and no cough, sputum production, or hemoptysis. The importance of smoking cessation is reviewed with the patient. Which of the following is the recommended screening strategy for lung cancer in this patient? A 18F-fluorodeoxyglucose and positron emission tomography scan (FDG-PET) B Chest radiography C Spiral CT scan of the chest D Sputum cytology E No screening

E Results from single-arm observational studies of spiral CT screening for lung cancer show that survival results with screening are improved over historical controls, but there is no proof of reduction of the mortality rate. Screening for early-stage lung cancer is not now recommended by any methodology, including spiral CT scan, chest radiography, sputum cytology, or 18F-fluorodeoxyglucose (FDG)-PET scan. No method has been shown to reduce death from lung cancer. Screening for lung cancer with spiral CT scanning detects 60% or more of incident cancers in stage I. However, the false-positive rate (number of benign nodules detected) for spiral CT lung cancer screening is high and may result in patient anxiety and unnecessary invasive testing. Recent large, single-arm observational studies of spiral CT screening show that about 60% to 85% of incident cancers (detected after the baseline scan) are stage I and that the survival rate is better than that of historical unscreened cohorts. However, this finding alone does not prove that lung cancer screening is effective. Proof of efficacy for a lung cancer screening test would be a reduction in the mortality rate among those screened compared with a comparable group at risk who were not screened. Although survival results may be provocative in an observational study, they are subject to bias. The survival rate may be dramatically improved without actually resulting in a reduction in deaths from lung cancer. Because of lead-time bias, length-time bias, and overdiagnosis, a randomized, controlled trial is generally accepted as the definitive means of establishing efficacy for a screening test. Various screening studies now under way are randomized trials that might answer the question of the efficacy of screening for lung cancer. The American Cancer Society does not recommend testing for early lung cancer detection in asymptomatic persons. However, the Society historically has recognized that patients at high risk of lung cancer because of significant exposure to tobacco smoke or occupational carcinogens may decide to undergo testing for early lung cancer on an individual basis after consultation with their physicians.

Pneumo 93 A 27-year-old man is evaluated for a 6-month history of cough, which is worse at night and after exposure to cold air. Often his cough is brought on by taking a deep breath or by laughter. He does not have postnasal drip, wheezing, or heartburn. He has a strong family history of allergies. Physical examination, chest radiograph, and spirometry are normal. He receives no benefit from a 3-month trial of gastric acid suppression therapy, intranasal corticosteroids, and an antihistamine-decongestant combination. Which of the following would likely provide the diagnosis of this patient's chronic cough? A 24-Hour esophageal pH monitoring BBronchoscopy C CT scan of the chest D CT scan of the sinuses E Trial of inhaled albuterol

E The diagnosis of cough-variant asthma is suggested by the presence of airway hyperresponsiveness and confirmed when cough resolves with a trial of inhaled albuterol. The patient has cough-variant asthma. A trial of inhaled albuterol could help control the patient's symptoms and confirm the diagnosis. The most common causes of chronic cough are asthma, postnasal drip syndrome (chronic sinusitis-rhinitis), and gastroesophageal reflux disease (GERD). Bronchoscopy and chest CT play no role in diagnosing cough due to these three causes. The diagnosis of cough-variant asthma is suggested by the presence of airway hyperresponsiveness and confirmed when cough resolves with asthma therapy. Sensitivity to cold air is a clinical marker of airway hyperresponsiveness that can be confirmed with a methacholine challenge test. The methacholine challenge test has a negative predictive value of nearly 100% in the context of cough; this test is extremely useful in ruling out asthma, but because it has a poor positive predictive value, it is not very useful in patients with high prior probability of airway hyperresponsiveness. There is little about the character and timing of chronic cough due to GERD that distinguishes it from other conditions; in addition, it often can be "silent" from a gastrointestinal standpoint. However, the patient failed to benefit from 3 months of empiric gastric acid suppression therapy for GERD; therefore it is reasonable to rule out cough-variant asthma before pursuing 24-hour esophageal pH monitoring. The patient does not have postnasal drip, purulent nasal secretions, sinus congestion, or other symptoms to suggest chronic or recurrent sinusitis and has not responded to treatment. Therefore, CT scan of the sinuses is not necessary. If the patient does not respond to albuterol, eosinophilic bronchitis should be considered as the cause of chronic cough, and bronchoscopy should be done to confirm that diagnosis.

Pneumo 64 A 67-year-old woman is evaluated in the hospital for acute abdominal pain. She was admitted for community-acquired pneumonia, and her condition was improving on therapy with antibiotics and fluids. However, on day 3 she developed acute abdominal pain associated with nausea. Her medical history includes a 7-year history of peripheral arterial disease, type 2 diabetes mellitus, hyperlipidemia, and an episode of diverticulitis 2 years ago. Her medications are pravastatin, aspirin, dipyridamole, and metformin. On physical examination, she appears anxious and in distress; temperature is 38.3 °C (101.0 °F), the blood pressure is 170/100 mm Hg, the pulse rate is 120/min, and the respiration rate is 25/min. She rates her pain as 10 out of 10. She has diffuse abdominal tenderness, more pronounced in the left lower quadrant with rebound tenderness. Which of the following is the most appropriate next diagnostic test in this patient? A Colonoscopy B CT scan of the abdomen C Left pelvic ultrasonography D Leukocyte count and differential E Supine and upright abdominal radiographs

E The term acute abdomen refers to sudden and severe abdominal pain less than 24 hours in duration. Rebound tenderness and severe diffuse abdominal pain are suggestive of an acute abdomen with peritonitis. Pain that is acute in onset generally points to acute inflammatory, infectious, or ischemic causes. Upper abdominal pain is usually of gastric, hepatobiliary, or pancreatic origin, whereas pain in the lower abdomen originates from the hindgut and genitourinary organs. All patients with abdominal pain should have measurements of serum amylase and lipase to evaluate for acute pancreatitis. Although an abdominal CT scan is usually necessary for a definitive diagnosis of acute abdominal pain, initial screening with supine and upright abdominal radiographs to look for air-fluid levels, suggestive of a perforated viscus, should be done. Chest radiograph and supine and upright abdominal radiographs should be obtained in every patient with significant acute abdominal pain to exclude bowel obstruction or perforation or intrathoracic processes (for example, pneumonia, pneumothorax, or aortic dissection) that can present as abdominal pain. This patient's history of diverticulitis suggests possible diverticular rupture. An elevated leukocyte count may suggest an intra-abdominal infection or abscess, but the onset is usually not abrupt. Based on its relatively low cost, convenience, and noninvasive nature, ultrasonography has been utilized as a diagnostic tool for acute diverticulitis. However, the examination remains operator-dependent and in the absence of well-designed prospective comparative studies, it remains a second-line diagnostic tool. Colonoscopy is not indicated in a patient with acute peritoneal signs and has the potential to worsen the situation by causing a perforation of inflamed bowel wall.

Pneumo 67 A 54-year-old man is evaluated in the emergency department for a 1-hour history of chest pain with mild dyspnea. The patient had been hospitalized 1 week ago for a colectomy for colon cancer. His medical history also includes hypertension and nephrotic syndrome secondary to membranous glomerulonephritis, and his medications are furosemide, ramipril, and pravastatin. On physical examination, the temperature is 37.5 °C (99.5 °F), the blood pressure is 110/60 mm Hg, the pulse rate is 120/min, the respiration rate is 24/min, and the BMI is 30. Oxygen saturation is 89% with the patient breathing ambient air and 97% on oxygen, 4 L/min. Cardiac examination shows tachycardia and an S4. Breath sounds are normal. Serum creatinine concentration is 2.1 mg/dL (185.6 µmol/L). Chest radiograph is negative for infiltrates, widened mediastinum, and pneumothorax. Empiric unfractionated heparin therapy is begun. Which of the following is the best test to confirm the diagnosis in this patient? A Assay for plasma D-dimer B CT pulmonary angiography C Lower extremity ultrasonography D Measurement of antithrombin III E Ventilation/perfusion scan

E Ventilation/perfusion scanning is an appropriate noninvasive test to diagnose acute pulmonary embolism. This patient is at high risk for pulmonary embolism because of his recent hospitalization, cancer, and nephrotic syndrome. A positive ventilation/perfusion scan would confirm the diagnosis of pulmonary embolism in this patient with a high pretest probability for the condition, especially in the absence of parenchymal lung defects on chest radiograph. The probability of pulmonary embolism is very high based on this patient's presentation that included chest pain, dyspnea, recent hospitalization and surgery, and active cancer and a protein-losing nephropathy. A negative D-dimer test would not be sufficient evidence to rule out a pulmonary embolism under these circumstances, and a high D-dimer level would add little to the diagnostic work-up. Decreased antithrombin III levels may result from nephrotic syndrome, and levels are lowered during acute thrombosis, especially during treatment with heparin. Therefore, measuring antithrombin III would add little to the accuracy of the diagnosis of pulmonary embolism or have any implication for immediate management decisions. Lower extremity ultrasonography can disclose asymptomatic deep venous thrombosis in a small percentage of patients presenting with symptoms of pulmonary embolism. However, the yield is relatively low and ventilation/perfusion scanning would have a much higher degree of accuracy. CT angiography is an acceptable modality to diagnose acute pulmonary embolism but requires a significant amount of contrast infusion, which would be contraindicated in a patient with an elevated serum creatinine level.


Conjuntos de estudio relacionados

NUR 108 Ch 12: Collaborative Practice and Care Coordination across Settings

View Set

Philosophy 101 Final: Fill in the blank questions

View Set

HESI & Final: Comprehensive Review

View Set

Chapter 13 Real Estate Principles

View Set

CHS 712- Epidemiology in Public Health Quizzes 1-12

View Set